Vous êtes sur la page 1sur 90

1

Inhaltsverzeichnis
Ablaufplan der Lehrveranstaltung 4

Einführung in die Medizinische Psychologie 5


anhand einiger ausgewählter medizinpsychologischer Themata
Maga. Irene Ritter, Maga. Dorothea Wuchse
Stressmanagement und Ressourcenaktivierung in der klinisch-psychologischen und psychothe- 20
rapeutischen Behandlung
Maga. Johanna Granegger, Maga. Kristina Koiner, Maga. Elisabeth Wallner
Klinische Neuropsychologie 25
Mag. Rene Hackstock
Psychotraumatologie 31
Mag . Gerda Greinz, Mag . Dorothea Wuchse
a a

Klinische Psychologie im Universitätsklinikum für Kinder- und Jugendheilkunde Salzburg 37


Mag . Esther Wagenhofer
a

Biofeedback 41
Dr. Andreas K. Kaiser, MSc
ELC. Early Life Care. Eine Einführung. 42
Maga. Silvia Désirée Pernter
Medizinische Psychologie in der Behandlung von Abhängigkeitserkrankungen 47
Maga. Sandra Amesberger, Maga. Dr in. Beatrix Höllbacher-Kneip
Einführung in die Gerontopsychologie 52
Mag . Petra Kröll, Mag. Wolfgang Treschnitzer
a

Psychoonkologie 58
Mag. Franz Wendtner
Supervision als Präventions- bzw. Interventionsmaßnahme 62
bei Belastungen am Arbeitsplatz Krankenhaus
Dr in. Brigitte Matschi, Mag. Lars Larsen
Sexuelle Gewalt bei Kindern und Jugendlichen – 66
Erkennen, anzeigen und vorbeugen
Maga. Dr in. Corinna Fritz, Maga. Sigrun Eder
Suizidrisikoeinschätzung und Risikomanagement 73
Mag. Dr. Clemens Fartacek, Priv.-Doz. Dr. Martin Plöderl
Schreiben an Studierende mit Hinweis auf USMLE-Prüfung 77

Medical psychology examples as preparation for the USMLE exam 78

2
Impressum:
© Institut für Klinische Psychologie der UKPPP
Uniklinikum Salzburg - Christian-Doppler-Klinik
Leitung: Mag. Dr. Andreas K. Kaiser, MSc
Koordination: PD Mag. Dr. Martin Plöderl, Mag. Dr. Clemens Fartacek
Lektorat: Barbara Jamnik

3
LV Medizinische Psychologie
1. Studienjahr 2018/19 (25.2.-28.2.2019) Mittwoch, 27. Februar 2019
Höllbacher-Kneip A1 Abhängigkeitserkrankungen (AUD Mitte)
08:30-10:00
Koordination: Dr. A. Kaiser, Dr. C. Fartacek und PD Dr. M. Plöderl Amesberger A2 Abhängigkeitserkrankungen (AUD Süd)
Institut für Klinische Psychologie Kröll, Treschnitzer B Geriatrie (Aud Nord)
10:00-10:30 Pause Raumwechsel für Studierende A  B
Montag, 25. Februar 2019 10:30-12:00 Höllbacher-Kneip B1 Abhängigkeitserkrankungen (AUD Mitte)
Uhrzeit Vortragende Inhalt Amesberger B2 Abhängigkeitserkrankungen (AUD Süd)
Kröll, Treschnitzer A Geriatrie (AUD Nord)
08:30-10:00 Wuchse A Einführungsveranstaltung 1 (AUD Süd) 12:00-13:00 Mittagspause
Ritter B Einführungsveranstaltung 2 (AUD Mitte) 13:00-14:30 Wendtner A Psychoonkologie (AUD Süd)
Matschi B1 Supervision (AUD Nord)
10:00-10:30 Pause Raumwechsel für Studierende A B
Larsen B2 Supervision (AUD Mitte)
10:30-12:00 Wuchse B Einführungsveranstaltung 1 (AUD Süd)
Ritter A Einführungsveranstaltung 2 (AUD Mitte) 14:30-15:00 Pause Raumwechsel für Studierende A B
15:00-16:30 Wendtner B Psychoonkologie (AUD Süd)
12:00-13:00 Mittagspause
Matschi A1 Supervision (AUD Nord)
13:00-14:30 Koiner BB1 Stressmanagement (Besp 22) Larsen A2 Supervision (AUD Mitte)
Granegger BB2 Stressmanagement (AUD Nord)
Wallner BB3 Stressmanagement (AUD Mitte)
Hackstock A Klinische Neuropsychologie (Aud Süd) Donnerstag, 28. Februar 2019
14:30-15:00 Pause Raumwechsel für Studierende A B 08:30-10:00 Greinz, Wuchse A Psychotraumatologie (AUD Süd)
15:00-16:30 Koiner AA1 Stressmanagement (Besp 22) Wagenhofer B Kinderpsychologie (AUD Mitte)
Granegger AA2 Stressmanagement (AUD Nord) 10:00-10:30 Pause
Wallner AA3 Stressmanagement (AUD Mitte)
Hackstock B Klinische Neuropsychologie (Aud Süd) 10:30-12:00 Greinz, Wuchse B Psychotraumatologie (AUD Süd)
Wagenhofer A Kinderpsychologie (AUD Mitte)
Dienstag, 26. Februar 2019 12:00-13:00 Mittagspause Raumwechsel für Studierende A  B
08:30-10:00 Eder A Sexuelle Gewalt (AUD Süd) 13:00-14:30 Kaiser A Biofeedback (AUD Süd)
Fartacek, Plöderl B Suizidprävention (AUD Mitte) Pernter B Early Life Care (AUD Mitte)
10:00-10:30 Pause 14:30-15:00 Pause Raumwechsel für Studierende A  B
10:30-12:00 Eder A Sexuelle Gewalt (AUD Süd) 15:00-16:30 Kaiser B Biofeedback (AUD Süd)
Fartacek, Plöderl B Suizidprävention (AUD Mitte) Pernter A Early Life Care (AUD Mitte)
12:00-13:00 Mittagspause Raumwechsel für Studierende A B
ö
13:00-14:30 Plöderl, Fartacek A Suizidprävention (AUD Mitte) Prüfungen:
Fritz B Sexuelle Gewalt (AUD Süd)
FR, 1.3.2019 08:30-12:00 Selbststudium
14:30-15:00 Pause
15:00-16:30 Plöderl, Fartacek A Suizidprävention (AUD Mitte) DI, 5.3.2019 08:30-12:00 Moodle Prüfung
Fritz B Sexuelle Gewalt (AUD Süd) MO, 11.3.2019 12:00-13:00 Wiederholungsprüfung

4
Einführung in die Medizinische Psychologie anhand einiger ausgewählter
medizinpsychologischer Themata
Maga. Irene Ritter, Maga. Dorothea Wuchse

Medizinische Psychologie, ein Spezialfach der Psychologie, ist in Österreich, aber auch international
gesehen, ein relativ junges Fach mit der kurzen Geschichte von ca. 100 Jahren, und wird erst seit
ca. 25 Jahren an den österreichischen Medizinischen Fakultäten gelehrt. Der Begriff geht nach ver-
breiteter Ansicht auf den Arzt Rudolph Hermann Lotze zurück und hat sich aus den Fächern Medizin
und Psychologie entwickelt. Das Anliegen der Medizinischen Psychologie ist die Einbeziehung psy-
chologischer und soziologischer Aspekte von Krankheit und Gesundheit in das ärztliche Denken und
Handeln.

1. Womit befasst sich die Medizinische Psychologie?


Die Medizinische Psychologie befasst sich mit:
- der Erforschung und Vermittlung des Zusammenwirkens biologischer, psychologischer
und soziokultureller Gegebenheiten für die Aufrechterhaltung oder Entstehung von Ge-
sundheit und Krankheit
- der Beziehung und Kommunikation zwischen ÄrztInnen und PatientInnen der Arzt oder
die Ärztin-Patient-Beziehung und Arzt-Patient-Kommunikation (Ärztliche Gesprächsfüh-
rung)
- dem Erleben und Verhalten von Menschen in der Situation des Krankseins
Die Medizinische Psychologie versucht weiters das Verständnis und die Einsicht in die psychologi-
schen Maßnahmen zur Krankheitsvorbeugung und zur Gesundheitsförderung zu wecken.
Neben den medizinisch-biologischen Fragestellungen sollen nämlich auch psychische und soziale
Situationen einbezogen werden. Die Medizinische Psychologie stellt somit im weitesten Sinn die An-
wendung der Klinischen Psychologie in der Organmedizin dar.

1.1. Wie unterscheiden sich Berufsgruppen, die sich mit der „Psyche“ befassen im
Hinblick auf Ausbildung und fachliche Zuständigkeiten?
In der Organisation Krankenhaus gilt es verschiedene Berufsgruppen, die unterschiedliche Aufgaben
wahrzunehmen haben, zu unterscheiden. Mit der Psyche beschäftigen sich in Österreich im speziellen
drei Berufsgruppen (Klinische PsychologInnen, PsychotherapeutInnen, PsychiaterInnen) wodurch es
oftmals zu Verwirrungen kommt, worin diese sich unterscheiden. Obwohl jede Berufsgruppe fachli-
che Schwerpunkte hat, so gibt es in der praktischen Ausübung inhaltlich viele Überschneidungen.
Klare Unterschiede gibt es jedoch im Ausbildungsweg und in der Kostenübernahme der Kranken-
kassen.
Alle aufgezählten Berufsgruppen sind freie Berufe mit Berufsbild. Sie sind einander gleichgestellt,
arbeiten eigenverantwortlich und sind weisungsfrei.

1. PsychologInnen/Klinische- und GesundheitspsychologInnen


PsychologInnen haben das Studium der Psychologie absolviert (Master oder Magister-Abschluss –
ein Bachelor Studium alleine reicht nicht aus zur Berufsbezeichnung). Sie beschäftigen sich vorwie-
gend mit dem Erleben und Verhalten von Menschen und in geringerem Ausmaß mit dem Körper und
den Organfunktionen.

5
Die selbständige Ausübung des psychologischen Berufs im Bereich des Gesundheitswesens setzt je-
doch noch zusätzliche Qualifikationen – wie den Erwerb der fachlichen Kompetenz nach § 4 PG und
den Erwerb praktisch-fachlicher Kompetenz, § 6 PG, voraus. Diese wird im Rahmen des postgradu-
ellen Curriculums zur „Klinischen Psychologie“ erworben, bei Beendigung ist dann die Berufsbe-
zeichnung „Klinische/r und/oder Gesundheitspsychologe/in“ zu führen.
Klinische- und/oder GesundheitspsychologInnen haben also auch das Studium der Psychologie und
im Anschluss daran eine ca. 2-jährige praktische und theoretische Ausbildung, in einer dafür aner-
kannten Ausbildungseinrichtung, absolviert. In Organisationen des Gesundheitswesens sind über-
wiegend Klinische- und GesundheitspsychologInnen tätig. Für diese Berufsgruppe gilt neben dem
Psychologengesetz unter anderem auch das Krankenanstaltengesetz, Bundesgesetzblatt Nr.
801/1993, § 11 b: „Die Landesgesetzgebung hat sicherzustellen, dass in den aufgrund des Anstalts-
zwecks und des Leistungsangebots in Betracht kommenden Krankenanstalten eine ausreichende kli-
nisch-psychologische und gesundheitspsychologische Betreuung und eine ausreichende Versorgung
auf dem Gebiet der Psychotherapie angeboten wird.“
Die Umsetzung des Krankenanstaltengesetzes im Salzburger Krankenanstaltengesetz, LGBl.
76/1995, § 10 c lautet: „In bettenführenden Krankenanstalten sind psychologische und psychothe-
rapeutische Dienste in einem solchen Umfang einzurichten, dass jenen PatientInnen, die aufgrund
ihrer Erkrankung besonders schweren psychischen Belastungen ausgesetzt sind, eine ausreichende
psychologische Betreuung und eine ausreichende psychotherapeutische Versorgung angeboten wer-
den kann.“
Die Krankenkasse übernimmt in Österreich die Kosten für klinisch-psychologische Diagnostik, nicht
jedoch für klinisch-psychologische Behandlungen.

2. PsychotherapeutInnen
PsychotherapeutInnen haben ihre Ausbildung in einer in Österreich anerkannten psychotherapeuti-
schen Ausbildungseinrichtung absolviert. Selbstverständlich ist auch die Ausbildung zur Psychothera-
peutIn im Psychotherapiegesetz geregelt. Nach dem ca. zweijährigen psychotherapeutischen Propä-
deutikum, das aus einem theoretischen Teil (765 Stunden) und einem praktischen Teil (550 Stunden)
besteht, schließt sich das Fachspezifikum mit einer Dauer von 4 bis 8 Jahren an.
Psychotherapie wird in Österreich zu einem Teil (ca. 1/3) von der Krankenkassa übernommen.
In Österreich sind 23 Therapierichtungen anerkannt, wobei diese in vier psychotherapeutische Ori-
entierungen untergliedert werden können: Tiefenpsychologische Orientierung, Humanistisch-exis-
tenzielle Orientierung, Systemische Orientierung, Verhaltenstherapeutische Orientierung.
Weitere Informationen finden Sie unter www.bmgf.gv.at.

3. FachärztInnen für Psychiatrie und Psychotherapeutische Medizin/ÄrztInnen mit PSY-Diplom


PsychiaterInnen sind FachärztInnen für Psychiatrie und Psychotherapeutische Medizin. Sie haben
nach dem Abschluss des Studiums der Medizin eine 6-jährige FachäztInnenausbildung absolviert.
Für diese Berufsgruppe gilt neben vielen anderen gesetzlichen Regelungen das Ärztegesetz.
ÄrztInnen mit Berechtigung zur selbständigen Berufsausübung (AllgemeinärztInnen oder Fachärztin-
nen) können neben der Psychotherapieausbildung nach dem Psychotherapiegesetz (BGBl. Nr
361/1990) auch über Diplome der Österreichischen Ärztekammer (ÖÄK) psychotherapeutische
Kompetenz erwerben. Es sind dies die Diplome für psychosoziale, psychosomatische und psychothe-
rapeutische Medizin.
Detaillierte Informationen dazu finden Sie unter www.arztakademie.at.

6
1.2. Leistungen der Klinischen und GesundheitspsychologInnen im Kranken-
haus
Die zentralen Aufgaben der Klinischen und GesundheitspsychologInnen im KH sind die Diagnostik,
die Beratung und die Behandlung von PatientInnen und deren Angehörigen. Während im stationären
psychiatrischen Setting meist therapeutische Ziele definiert werden, geht es bei der Akutbehandlung
meist um Krankheitsbewältigung und psychische Stabilisierung.

1.2.1. Klinisch-psychologische Diagnostik

Die klinisch-psychologische Diagnostik ist ein wesentlicher Teil klinisch-psychologischer Tätigkeit.


In der Klinischen- und Gesundheitspsychologie gibt es eine Vielzahl von Fragestellungen und dem-
entsprechend eine große Auswahl an psychologischen Testverfahren. Psychodiagnostische Frage-
stellungen können zum Beispiel die Beurteilung der Fahrtauglichkeit nach Beeinträchtigungen durch
eine Erkrankung sein, oder die Einschätzung der Suizidalität, der Lebensqualität, die Abklärung psy-
chosomatischer Symptome usw.

Methoden klinisch-psychologischer Diagnostik

Je nach Art der Datengewinnung wird in der psychologischen Diagnostik zwischen Papier-Bleistift-
Tests, projektiven Verfahren, computergestützten Testverfahren, Interviewverfahren, Checklisten und
Verhaltensbeobachtung unterschieden.

Anwendungsfelder klinisch-psychologischer Diagnostik

• Klinisch-psychologische Anamnese:
Familienanamnese
Eigenanamnese
Außenanamnese
Krankheits- bzw. Schmerzanamnese
Analyse von Verarbeitungsprozessen und vorhandenen Ressourcen

• Leistungsdiagnostik/ Intelligenzdiagnostik:
Hier kommen bevorzugt Testverfahren zur Erfassung von Aufmerksamkeit und Konzentration,
schulischen Fertigkeiten, Intelligenz oder Gedächtnisfunktionen zum Einsatz.

• Entwicklungsdiagnostik:
Beurteilung des Reifegrades bei der kindlichen Entwicklung. Z.B: Feststellung von frühkindli-
chen Entwicklungsstörungen.

• Neuropsychologische Funktionsdiagnostik:
Die neuropsychologische Funktionsdiagnostik wird bei der Erfassung von Funktionseinbußen
aufgrund von Hirnschädigungen und der Einschätzung der kognitiven Leistungsfähigkeit ein-
gesetzt. Z.B: Beeinträchtigungen in Konzentration, Gedächtnis, Sprache, Exekutivfunktionen,
Beurteilung einer Minderung der Erwerbsfähigkeit, etc.

• Persönlichkeitsdiagnostik:
Die Persönlichkeitsdiagnostik dient der Beschreibung und Klassifikation von PatientInnen hin-
stichlich ihrer Persönlichkeit und Persönlichkeitsstile (z.B: Introversion-Extroversion, Risikobe-
reitschaft, Offenheit für Neues…). Diese wird oftmals auch bei nicht störungsbezogenen Fra-
gestellungen untersucht (Berufsorientierung, etc…).

7
• Diagnostik von psychischen Störungen, Verhaltensauffälligkeiten, personaler und sozialer
Ressourcen, Belastungen und Belastungsbewältigung

Zur Klassifikation psychischer Störungen stehen mit der Internationalen statistischen Klassifikation der
Krankheiten und verwandter Gesundheitsprobleme der WHO (International Classification of Dise-
ases) ICD-10, Kapitel V (F) und dem Diagnostischen und Statistischen Handbuch Psychischer Stö-
rungen (Diagnostic and Statistical Manual of Mental Disorders) DSM-V zwei internationale Klassifi-
kationssysteme zur Verfügung. In Kapitel V (F) der internationalen Klassifikation psychischer Störun-
gen (ICD-10) werden psychische- und Verhaltensstörungen unter dem Kapitel F (F0-F9) aufgelistet.

2. Die Medizinische Psychologie in der Krankheitsbewältigung

2.1. Welche Möglichkeiten stehen zur Bewältigung schwieriger Situationen im


Krankheitserleben zur Verfügung?
Zur Bewältigung von schwierigen Situationen (Coping) stehen PatientInnen verschiedene Mechanis-
men zur Verfügung:
1. Abwehrmechanismen
2. Bewältigungsformen
3. Psychologische Unterstützung (z.B. Stabilität der therapeutischen Beziehung)
4. Medizinische Hilfe
5. Soziale Unterstützung (z.B. Selbsthilfegruppen; Familie, Freunde)

2.2. Welche Rolle spielt die ÄrztInnen-/ PatientInnenbeziehung auf die Gene-
sung?
Der Beruf des Arztes und der Ärztin ist grundlegend beziehungsorientiert und beinhaltet den täglichen
Umgang mit Menschen. Wie auch in privaten Beziehungen beeinflusst die Interaktion und Kommu-
nikation zwischen Arzt/Ärztin und PatientIn oftmals unbemerkt das weitere Verhalten sowohl von ärzt-
licher Seite als auch von Seite der PatientInnen. Die Beziehung zwischen Arzt/Ärztin und PatientIn hat
einen Einfluss darauf welche Informationen erhoben werden, welche Untersuchungen in Folge ge-
macht werden, welche Behandlungen durchgeführt werden und inwieweit PatientInnen sich an die
Empfehlungen des Arztes/der Ärztin halten. All diese Faktoren beeinflussen die Zusammenarbeit und
wirken sich somit auf die Genesung aus. Fühlt ein Patient/eine Patientin sich wohl, ist er oder sie
bereit Informationen über sich und körperliche Symptome dem Arzt oder der Ärztin mitzuteilen. Fühlt
ein Patient oder eine Patientin sich verstanden und ausreichend informiert über Behandlung und
Erkrankung, ist die Wahrscheinlichkeit erhöht, dass medizinische Anweisungen und Empfehlungen
befolgt werden und eine gesunde Erwartungshaltung bezüglich der Genesung besteht. Anders herum
besteht die Gefahr, dass bei PatientInnen eine Unsicherheit und Angst entsteht, wenn sie sich nicht
ausreichend informiert und verstanden fühlen. Dies kann in weiterer Folge dazu führen, dass Patien-
tInnen im Stationsalltag unverhältnismäßig häufig nach Pflegepersonal oder Ärzt/Innen fragen, im
ambulanten Bereich Doctor-Shopping betreiben und/oder zusätzlich Folgeerkrankungen wie De-
pressionen entstehen oder schlechte Nachrede entsteht, obwohl ÄrztInnen „rein medizinisch“ die
richtigen Behandlungsschritte gewählt hatten.

Dabei ist es auch wichtig zu erwähnen, dass bei einem ÄrztInnenbesuch unterschiedliche Erwartun-
gen und Wünsche vorherrschen können. Ob ein ÄrztInnenbesuch als „Erfolg“ bewertet wird, hängt
ab von den Zielen der einzelnen Personen. Nicht immer ist „Genesung“, „Heilung“, „körperliche
Besserung“ das Ziel eines ÄrztInnenbesuches aus Sicht der PatientInnen.

8
Des Weiteren spielen Erwartungen über Rollenbilder (ideale ÄrztInnen und PatientInnen) in der Be-
ziehung und Interaktion eine Rolle. Welche Erwartung habe ich, als Arzt oder Ärztin an meine Pati-
entInnen? Welche Erwartungen haben PatientInnen an ihre behandelnden ÄrztInnen?
Befragungen über das ideale ÄrztInnenbild in der Bevölkerung ergeben überregional übereinstim-
mende Bilder: Der oder die „ideale“ Arzt bzw. Ärztin behält in jeder Situation seine/ihre Nerven,
handelt fachlich kompetent, mit hoher ethischer Verantwortung, ist selbstlos und tatkräftig, weiß stets
Abhilfe, ist geduldig, hilfsbereit und nie krank. Das Realbild von ÄrztInnen - die Selbsteinschätzung
ihres Verhaltens - schwankt sowohl in vertikaler (hierarchischer) wie auch in horizontaler (fachspezi-
fischer) Richtung; zudem bestehen regionale und altersmäßige Differenzen.
In Analogie zum Positivstereotyp des/der „idealen“ Arztes/Ärztin gibt es auch – aus der Sicht von
ÄrztInnen - die Rolle der „idealen“ PatientInnen. Zahlreiche durchgeführte Befragungen von nieder-
gelassenen ÄrztInnen ergaben folgende Rollenattribute der „idealen“ PatientInnen: klar, geordnet,
bereit sich helfen zu lassen, gelöst, konzentriert, frisch, fröhlich und vergnügt (YAVIS-Patient: young,
attractive, verbal, intelligent, successful). In weiteren Untersuchungen wurden auch die Merkmale
der „heutigen“ PatientInnen eruiert, die folgendermaßen definiert wurden: schwach, ernst, nachgie-
big, müde, krank, redselig, weitschweifig, nüchtern, unsympathisch, braucht Anregung (HOUND-
Patient: humble, old, unattractive, nonverbal, dumb).

Um sich über die Beziehung zwischen ÄrztInnen und PatientInnen reflektiert Gedanken machen zu
können, ist es eine Voraussetzung, einen Zugang zur eigenen Persönlichkeit zu haben. Inwieweit der
Arzt oder die Ärztin zu sich selbst einen emotionalen Bezug hat und sich differenziert wahrnehmen
und nach außen vermitteln kann, wird richtungsweisend für die adäquate Begegnung mit den Kran-
ken sein.

Für die Ärztinnen-PatientInnen-Beziehung ist das Gespräch ein zentrales Element. Wesentliche ärzt-
liche Aufgaben für eine gelungene Kommunikation sind dabei: Anamneseerhebung, Erkundung der
Befindlichkeit, Mitteilung von Information über die Krankheit und ihre Behandlung, Beratung, Moti-
vation zur Mitarbeit (Compliance), Aussprechen von Trost und Mitgefühl.

2.3. Sensibilisierung für unbewusste Vorgänge in der ÄrztInnen-PatientInnen-


Interaktion: Übertragung und Gegenübertragung
Das Ziel dieses Teils der Einführungsveranstaltung ist es, für unbewusst ablaufende Vorgänge sensi-
bilisiert zu werden, um somit in weiterer Folge sowohl das Verhalten von PatientInnen, als auch die
eigenen Reaktionen (auf PatientInnen) besser verstehen zu können, um anschließend auch adäquater
zu reagieren.

In der ÄrztInnen-PatientInnen-Beziehung spielt das psychologische Phänomen von Übertragung und


Gegenübertragung stets eine Rolle. Das Bewusstwerden von solchen unbewusst ablaufenden Pro-
zessen kann dabei helfen, die Beziehung und Kommunikation zwischen Ärztinnen und PatientInnen
zu verbessern.

 Übertragung / Transference = Das Übertragen von bestimmten Gefühlen und Eigen-


schaften, die durch Erfahrungen mit prägenden Personen gemacht wurden, auf das Gegen-
über. Im Kontext der Ärztinnen-PatientInnen Beziehung, Eigenschaften die die PatientIn der
ÄrztIn/TherapeutIn zuschreibt. Z.B.: Die ÄrztIn erinnert die PatientIn an ihren autoritären Va-
ter und der Ärztin wird somit unbewusst gleich die Eigenschaft „autoritär“ zugeschrieben.

 Gegenübertragung / Countertransference = Unter Gegenübertragung können zwei,


leicht voneinander abweichende, Begriffe gemeint sein:

9
- Das Gleiche wie Übertragung nur diesmal geschieht die Übertragung in die andere Richtung:
von ÄrztIn auf die PatientIn (Version USMLE Prüfungsstoff). Z.B: Die PatientIn erinnert die
Ärztin an ihren Großvater, sie schreibt der PatientIn unbewusst eine negative Einstellung in
Bezug zu Medikamenten zu (ohne dass dies stimmen muss), da ihr Großvater nie Medika-
mente einnehmen wollte.
- Das Gegenüber spürt die Übertragung, reagiert unbewusst auf die Übertragung mit einer
verhaltenskonformen Gegenübertragung. Z.B: Patient überträgt auf den Arzt die autoritäre
Vaterrolle. Der Arzt spürt unbewusst, dass der andere ihn als autoritär empfindet und reagiert
unbewusst der Rolle entsprechend autoritär. Dies erklärt auch, warum Personen von ihrer
Umwelt, auch wenn es unterschiedliche und neue Leute sind, immer wieder gleiche Verhal-
tensreaktionen bekommen.

2.4. Die Abwehrmechanismen


 Abwehr (Sigmund Freud, Begriff erstmals 1894 verwendet in „Die Abwehr-Neuropsychosen“)
 Abwehrmechanismen (Anna Freud 1936: erste systematische Beschreibung)

„Unter Abwehr verstehen wir alle intrapsychischen Operationen, die darauf abzielen, unlustvolle Ge-
fühle, Affekte, Wahrnehmungen etc. vom Bewusstsein fernzuhalten bzw. sie >in Schach zu halten<“
(Mentzos, 1984, S. 60). Mithilfe der (unbewusst ablaufenden) „Abwehr“ bzw. der Abwehrmechanis-
men sollen also die Psyche betreffende Vorgänge (Gefühle, Affekte, Erinnerungen, kognitive Ein-
sicht…), aber auch von außen kommende Informationen, Reize oder Wahrnehmungen, die als un-
angenehm, peinlich, unerträglich, heikel oder gar gefährlich eingestuft werden, vom Bewusstsein
ferngehalten werden. Abwehrmechanismen sind für den seelischen Apparat wichtig und unverzicht-
bar, es besteht allerdings ein fließender Übergang zur Krankhaftigkeit. Ein Abwehrvorgang kann
beispielsweise krankhaft sein, wenn ein Mensch dadurch negativ in seiner Lebensführung beeinflusst
wird (zB.: Verfolgungswahn) und/oder keine realitätsgerechte Wahrnehmung mehr möglich ist (zB.:
Größenwahn, Liebeswahn).

Abwehrmechanismen haben wichtige Aufgaben:

 Bewältigungsaufgaben: z.B.: Bewältigung von schwierigen Situationen wie Trennung,


schwere Krankheit, Tod von Nahestehenden. z.B.: Verleugnung, Intellektualisieren

 Schutzaufgaben: z.B.: unlustvolle Gefühle fernhalten wie bei der Affektisolierung; ein Schutz
vor der Aufnahme von weiteren Reizen (Ohnmacht); aber auch Schutz vor: körperlicher Ge-
fährdung, Gefahr, dass tabuisierter-triebhafter Impulse (Es-Impulse) bewusstwerden, Bestra-
fung durch das Gewissen.

Abwehrmechanismus Beschreibung Beispiele


Verdrängung / Zurückhalten von stark be- Missbrauchserfahrungen können für Jahr-
Repression drohlichen Impulsen oder Er- zehnte verdrängt werden (nicht in Erinne-
innerungen vom Bewusst- rung sein), und dann plötzlich wieder in
sein, weil sie Angst oder an- Erinnerung kommen.
dere negative Konsequenzen
hervorrufen würden. Verläuft
unbewusst.
Verleugnung / Denial Leidvolle oder bedrohliche Mann verleugnet, dass seine Frau ihn seit
Tatsachen sind bewusst nicht Jahren betrügt, obwohl er es mitbe-
im Denken des Betroffenen kommt.
vorhanden.

10
Rationalisieren Verhaltensweisen werden se- Eine Person kann sich die Finanzierung
kundär durch Scheingründe eines Autos nicht mehr leisten und ver-
gerechtfertigt. Man versucht kauft dieses. Als Scheinmotiv wird der
das Verhalten durch (sozial Umweltaspekt (Auto= Umweltverschmut-
akzeptierte) Motive zu recht- zung) oder die fehlende Notwendigkeit
fertigen. eines Autos angegeben.

Projektion Eigene unerwünschte Im- Mann verliebt sich in eine neue Frau, und
pulse werden jemandem beschuldigt die eigene Ehefrau der Un-
oder etwas Anderem zuge- treue.
schrieben.
Fixierung / Fixation Bestimmte Teile d. Entwick- Als Erwachsener einen Teddybären zum
lung bleiben auf einer Ent- Einschlafen benötigen.
wicklungsstufe (kindlich) hän-
gen.
Regression Rückentwicklung in einen Ein Kind beginnt wieder zum Bettnässen,
früheren Entwicklungszu- wenn ein neues Geschwisterkind kommt,
stand. obwohl es davor schon zum Bettnässen
aufgehört hat.
Ohnmacht Als Abwehr (Reaktion) auf Bei einem traumatischen Ereignis ohn-
ein psychisches Trauma, da mächtig werden (z.B. Unfall, Gewaltver-
die intensiven Reize einen brechen), um nichts mehr Wahrnehmen
überwältigen. zu müssen.
Identifikation mit dem Um die unerträgliche Angst Das bekannteste und makaberste Beispiel
Aggressor erträglicher zu gestalten, hierfür ist das Phänomen des Antisemitis-
stellt sich der Bedrohte mus unter Juden. Diese Identifikation mit
gleichsam emotional auf die den Unterdrückern dient der Angstreduk-
Seite des Angreifers. tion.

Affektisolierung Die Isolierung vom Affekt ist Ich beschreibe einen beobachteten Mord
die Trennung von Inhalt und völlig emotionslos und rein faktenbezo-
begleitender affektiver bzw. gen.
emotionaler Tönung.
Dissoziation Kurzfristige drastische Verän- Nicht mehr reagieren auf die unmittel-
derung in Wahrnehmung, bare Umwelt, da man gedanklich sich in
Persönlichkeit, Gedächtnis, einer anderen „Welt“ befindet (anderer
Bewusstsein oder Motorik Ort, Situation, Zeitpunkt).
um emotionalen Stress zu Oder z.B. Lähmung eines Körperteiles
entgehen. ohne organische Ursache.
Ausleben / Acting out Ausleben von unakzeptieren Einen Wutanfall bekommen.
Gefühlen und Gedanken.
Reaktionsbildung / Unerlaubte, unerwünschte Zwangshandlungen als Reaktion auf eine
Reaction formation oder angstauslösende Im- übertriebene Ordnungsliebe oder Ängste.
pulse werden durch die Ent- Homophobie bei eigener nicht erwünsch-
wicklung entgegengesetzter ter Homosexualität.
Handlungen oder Einstellun-
gen abgewehrt. Man denkt
und handelt auf Dauer ent-
gegengesetzt von unbewuss-
ten nicht erwünschten Impul-
sen.

11
Verlagerung / Ein unerwünschter Impuls Die Wut auf Vorgesetzte wird auf den/die
Displacement (meist Aggression) wird auf PartnerIn oder andere MitarbeiterInnen
ein anderes als das eigentli- abgeladen.
che Objekt gerichtet
Spaltung / Splitting Spaltung in „nur gut“ oder Extremes schwarz-weiß denken: „ÄrztIn ist
„nur böse“ da man mit Am- nur gut“, „Pflege ist nur böse“…
biguität nicht umgehen
kann.

Positive/Reife Abwehrmechanismen – „Mature Defenses“

Sublimierung Triebregung wird von ihrem Kulturleistungen: Künstlerische Schöpfun-


ursprünglichen Ziel weg zu gen und geistige Leistungen.
einem kulturell höherwerti-
gen Ziel hin gerichtet.
Altruismus Sich von negativen Gefühlen Ein Teil von zwielichtig erworbenem Geld
erleichtern indem man groß- für wohltätige Zwecke spenden.
zügig ist oder etwas für an-
dere tut.
Humor Den humorvollen Anteil einer Scherze über eine bevorstehende Prüfung
schwierigen Situation sehen machen.
Unterdrückung / Bewusst temporär Gedanken Bewusst sich im Vorhinein keine Sorgen
Suppression oder Gefühle fernhalten. machen bis es so weit ist. Z.B. vor einer
OP, einem Wettkampf nicht daran den-
ken.

3. Ausgewählte psychologische Grundlagen

So wie es in der Medizin vielzählige spezifische Fachgebiete gibt, lässt sich auch die Psychologie in
zahlreiche Fachgebiete untergliedern. In dieser Vorlesung werden einige davon näher erläutert. Zum
besseren Verständnis der Entstehung von psychischen Störungen werden nun im allgemeinen Teil
Grundlagen aus der Entwicklungspsychologie und Klinischen Psychologie vermittelt.

3.1. Entwicklungspsychologie: Bindungstheorie


In der Bindungstheorie geht man davon aus, dass Beziehungserfahrungen im Säuglingsalter mit pri-
mären Bezugspersonen zu einem Bindungsstil beitragen, welcher im weiteren Verlauf des Lebens
maßgeblich für das Verhalten und Erleben in menschlichen Interaktionen ist. Gegründet wurde die
Bindungstheorie in den 1950er Jahren vom englischen Kinderpsychiater und Psychoanalytiker John
Bowlby. Entgegen Freuds Triebtheorie, dass sich ein Säugling durch die orale Triebbefriedigung
während des Stillens an seine Mutter binde, postulierte Bowlby ein biologisch angelegtes Bindungs-
system. Neugeborene seien genetisch programmiert, eine Bindung zu ihrer Mutter einzugehen, um
ihr Überleben zu sichern. Der Säugling baut sich so im ersten Lebensjahr ein interaktives Bindungs-
system auf (durch Festklammern, Weinen, Ärger, Protest, emotionalem Rückzug, Resignation, Aufsu-
chen der Bezugsperson), das die Nähe zur Bezugsperson sichern soll. Für Bowlby war die primäre
Bezugsperson immer nur die Mutter, heute geht man jedoch davon aus, dass es zwar eine primäre
Bezugsperson braucht, diese muss jedoch nicht die biologische Mutter sein.
Mary Ainsworth etablierte in den 70iger Jahren zur Beurteilung des Bindungsstils die mittlerweile
bewährte Untersuchungsmethode der „fremden Situation“. In der Methode wird untersucht wie sich
Kinder im Alter von 1 bis 2 Jahren verhalten, wenn sie mit einer fremden Person alleine gelassen
werden und wie sie dann bei Rückkehr der primären Bezugsperson auf diese reagieren. Aus dem

12
Verhalten konnten vier Bindungsstile identifiziert werden, ein sicher- gebundener Stil und drei unsi-
chere Bindungsstile:

1.) Sicher gebundene Bindung: diese Kinder nutzen die Bezugsperson als sichere Basis, lassen
sich von ihr schnell trösten und sind so in der Lage aktiv die Welt zu erkunden. Sie bevorzugen
die Bezugsperson über einer fremden Person, und können deswegen auch weinen, wenn die
Bezugsperson den Raum verlässt.

2.) Unsicher – vermeidende Bindung: diese Kinder scheinen auf die anwesende Bezugsperson
nicht zu reagieren, auch nicht wenn diese den Raum verlässt. Sie interagieren mit der frem-
den Person ähnlich wie mit der Bezugsperson, bei der Wiedervereinigung mit der Bezugs-
person sind sie eher zögerlich oder ablehnend. Sie klammern nicht und wollen auch nicht in
den Arm genommen bzw. getröstet werden.

3.) Unsicher – ambivalente (resistente) Bindung: Diese Kinder klammern häufig an der Bezugs-
person, eine aktive Exploration der Umwelt gelingt ihnen deswegen eher nicht. Nach der
Trennung von der Bezugsperson zeigen diese Kinder den größten Stress und weinen stark.
Bei der Wiedervereinigung zeigen sie ein ambivalentes Verhalten: sie drücken einerseits den
Wunsch nach Körperkontakt und Nähe zur Bezugsperson aus, reagieren aber auch wütend,
aggressiv und ablehnend (Strampeln, Schlagen, Stoßen, …). Sie brauchen sehr lange um
sich in dann wieder zu beruhigen bzw. einen emotional stabilen Zustand zu erreichen.

4.) Desorientiert – desorganisierte Bindung: Dieses Muster reflektiert die größte Unsicherheit.
Bei der Wiedervereinigung zeigen diese Kinder eine Reihe konfuser, sich widersprechender
Verhaltensweisen. Z.B. wenden sie sich ab, während sie von der Bezugsperson auf den Arm
genommen werden, stoßen Schreie aus nachdem sie sich schon beruhigt haben, nähern sich
der Bezugsperson mit depressivem Gesichtsausdruck oder erstarren in ihren Bewegungsab-
läufen.
Die Frage wie nun ein Bindungsmuster entsteht hängt von der Bindungsqualität ab. Diese wird be-
einflusst von: 1.) die Gelegenheit eine enge Beziehung einzugehen, 2.) die Qualität der Fürsorge,
3.) die Persönlichkeitseigenschaften des Säuglings und 4.) der familiäre Kontext. Aktiviert wird das
Bindungsverhalten durch die Trennung von der Bezugsperson, äußere und innere Bedrohung und
Schmerz. An der Bezugsperson liegt es nun dem Säugling emotionale und reale Sicherheit zu geben
und es vor Bedrohung zu schützen. Nur wenn das Kind sich sicher und geborgen fühlt, ist es im
späterem Verlauf dazu in der Lage, die Lebenswelt aktiv zu erkunden. Für Bowlby ist die erste sichere
Bindung so wichtig, weil sie die Voraussetzung schafft, ein „inneres Arbeitsmodell“ zu entwickeln,
das einen Bezugsrahmen dafür schafft sich selbst, andere und die Welt zu verstehen. Dieses Arbeits-
modell steuert wie wir unsere Gedanken, Gefühle und Erwartungen in unsere Beziehungen einbrin-
gen, auch im Erwachsenenalter noch. Insofern ist der Bindungsstil maßgeblich an unserer psychoso-
zialen Entwicklung beteiligt und kann so auch mitverantwortlich für psychische Störungen sein.

3.2. Lerntheorien (USMLE-Prüfungsstoff)


Lerntheorien versuchen zu erklären, unter welchen Bedingungen ein bestimmtes Verhalten öfter auf-
tritt und wann es unterlassen wird. Die folgenden „klassischen“ Lerntheorien stammen aus dem be-
havioristischen Ansatz (vorherrschender Denkansatz in der Psychologie von 1920-1960), indem da-
von ausgegangen wird, das bestimmte Stimuli (=Reize) auf den Organismus einwirken und eine
bestimmte Reaktion bzw. Verhaltensweisen auslösen. Was dazwischen an inneren Prozessen im Or-
ganismus vor sich geht, ist nicht von Bedeutung. Dabei gibt es zwei große Theoriegruppen: Die
Theorie des klassischen Konditionierens und die des operanten Konditionierens.

13
 Klassisches Konditionieren: Beim klassischen Konditionieren wurden Reflexe, die durch be-
stimmte Reize zwangsläufig (unwillkürlich) ausgelöst wurden, mit bis dahin neutralen (nicht
reflexauslösenden) Reizen gekoppelt. Durch Lernerfahrungen werden neutrale Reize zu kon-
ditionierten Reizen. Das berühmteste Beispiel dafür gibt es von den Experimenten mit Hunden
von dem russischen Physiologen Ivan Pawlow: Bei einem Hund tritt bei Wahrnehmung des
Futters (unkonditionierter Reiz) ein Speichelfluss ein (unkonditionierte automatische Reak-
tion). Wird jetzt das Futter mehrmals gleichzeitig mit einem Glockenton (neutraler Reiz) prä-
sentiert, reicht nach einiger Zeit schon der Glockenton (dann ein konditionierter Reiz) aus,
um einen Speichelfluss beim Hund auszulösen. Der Hund hat gelernt, dass der Glockenton
mit dem Futter einhergeht und ein anfänglicher neutraler Reiz (Glockenton) wird zu einem
konditionierten Reiz.

 Operantes Konditionieren: Willkürlich gesetztes Verhalten (Operanten) wird konditioniert.


Durch Bestrafung und Belohnung sollen erwünschte Verhaltensweisen erlernt werden.

o Positive Verstärkung: Die Häufigkeit eines Verhaltens soll durch Belohnung erhöht
werden. Z.B. Maus drückt Knopf um Futter zu bekommen.
o Negative Verstärkung: Eine Verhaltensweise wird „verstärkt“ indem ein unangeneh-
mer Reiz entfernt wird. Z.B. Maus drückt Knopf um unangenehme laute Geräusche
auszuschalten.
o Bestrafung: Durch regelmäßige Bestrafung soll ein ungewolltes Verhalten reduziert
werden. Z.B: Kind wird Spielzeug weggenommen, wenn es sich schlimm verhält.
o Löschung: Konditionierte Reaktionen können mit der Zeit auch wieder gelöscht wer-
den, sowohl bei operanten oder klassischen Reaktionen. Wenn ein Verhalten mehr-
malig nicht mehr bestraft oder belohnt wird oder Reize nicht mehr mit Reflexen ein-
hergehen, kann sich die Konditionierung auch wieder rückbilden. Z.B. Ein miss-
brauchter Hund hat gelernt, dass er von seinem Besitzer geschlagen wird, wenn er
sich nähert und meidet deswegen Menschen. Bei Besitzerwechsel wird der Hund nicht
mehr geschlagen, mit der Zeit wird die Konditionierung (sich einem Menschen nä-
hern verursacht Schläge) gelöscht und der Hund traut sich langsam wieder sich Men-
schen zu nähern.

3.3. Die Entstehung von psychologischen Störungen: Das Diathese Stress-Mo-


dell
Bei der Erklärung von psychischen Störungen besteht die Frage, wie es zum Vollbild einer behand-
lungsbedürftigen Störung kommt. Bei der Entstehung spielen sowohl personenzentrierte, interne als
auch externe Faktoren eine Rolle und beide Faktoren stehen in einem dynamischen Prozess zueinan-
der. So kommt es zu komplexen Wechselwirkungen zwischen den von außen auf die Person wirkende
Faktoren und dem Einwirken der Person auf die äußeren Faktoren.

Ein etabliertes Modell, das auf diese Wechselwirkungen Rücksicht nimmt, ist das Diathese-Stress
Modell oder auch Vulnerabilitäts-Stress Modell. Es erklärt sowohl die Entstehung als auch die Auf-
rechterhaltung von Störungen. Das Modell geht davon aus, dass psychische Störungen durch aktu-
elle chronische Belastungen (= Stress) ausgelöst werden, die auf individuell unterschiedliche Veran-
lagungen einer Person (= Diathese, Vulnerabilität, Prädisposition) treffen. Dabei spielt die zeitliche
Interaktion zwischen prädisponierenden, auslösenden und aufrechterhaltenden Faktoren den ent-
scheidenden Faktor für das Auftreten einer Störung.

14
So können zum Beispiel finanzielle Sorgen bei Person A eine Depression auslösen, bei Person B mit
einer unterschiedlichen Veranlagung (Diathese) keine.

DIATHESE-STRESS-MODELL

DIATHESE
STRESS Veranlagung d. Betroffenen
aktuelle, chron. Belastung • Biologisch
• Sozial • Genetisch
• Psychologisch • Kognitiv
• Biologisch • umweltbezogen

3.3.1. Die Auswirkungen von Stressoren auf Gesundheit, Verhalten, Erleben


und Denken
Im Alltag sind wir alle Belastungen ausgesetzt. Erleben wir diese Belastungen als Stress, wirkt sich
das auf körperlicher und psychischer Ebene aus, wobei beide Ebenen stark miteinander verknüpft
und somit auch nicht entkoppelt voneinander betrachtet werden sollten. Die Psychoneuroimmuno-
logie beschäftigt sich damit, wie diese körperlichen und psychischen Faktoren zusammenspielen.
Obwohl man früher davon ausging, dass das Immunsystem unabhängig vom Nervensystem operiert,
gibt es mittlerweile zahlreiche Studien die einen engen Zusammenhang zwischen Immunsystem, Zent-
ralnervensystem und Psyche aufzeigen. Das Forschungsgebiet der Psychoneuroimmunologie wurde
etabliert nachdem Robert Ader und Nicolas Cohen 1975 experimentell nachweisen konnten, dass
das Immunsystem mit dem zentralen Nervensystem zusammenarbeitet und lernen kann. Dazu paar-
ten sie bei Mäusen die Präsentation von gesüßtem Wasser (neutraler Reiz) mit einer Injektion der
immunsuppressiven Substanz Zyklophosphamid (unkonditionierter Reiz). Die erneute Präsentation
von gesüßtem Wasser führte bereits nach einmaliger Paarung zu einer deutlich geringeren Antikör-
perproduktion nach Injektion von Schaferythrozyten.

Auf körperlicher Ebene kommt es bei Stress zu einer neuroendokrinen Stressreaktion. Ausgehend
vom Hypothalamus wird die Hypophysenvorderlappen-Nebennierenrinde-Achse und das sympathi-
sche Nervensystem aktiviert, wodurch Cortisol, Noradrenalin und Adrenalin produziert werden. Kurz-
fristig bewirkt dies eine Ausschwemmung von immunkompetenten Leukozyten in der Blutbahn,
wodurch systemisch die Abwehrkraft zunächst gestärkt wird. Langfristig, wenn der Stress anhält,
kommt es zu immunsuppressiven Effekten und zahlreichen gesundheitsschädliche Folgen. Praktisch
erleben wir das zum Beispiel im Alltag, wenn wir im Urlaub, nach einer stressigen Phase, krank
werden. In der Psychosomatik wird dies sichtbar, wenn psychologische Ursachen körperliche Symp-
tomen bzw. Beschwerden auslösen.

Die Psychologie beschäftigt sich deswegen stark mit der Frage wie Stress entsteht und wie damit gut
umgegangen werden kann durch die Anwendung von Copingstrategien bzw. Bewältigungsstrate-
gien. Ein etabliertes Modell, das genau auf diese Frage eingeht, ist das im Jahre 1974 veröffentlichte
„transaktionale Stressmodell“ von Richard Lazarus, einen amerikanischer Emotionsforscher.

15
Literaturverzeichnis:
Berk, L. E. (2005). Geschichtliche Hintergründe, Theorien und Forschungsstrategien. Entwicklungspsychologie (3., aktua-
lisierte Aufl.), S.22. Pearson Studium: München.
Berk, L. E. (2005). Bindungsentwicklung. Entwicklungspsychologie (3., aktualisierte Aufl.), S.255-259. Pearson Studium:
München.
Dilling, H., Mombour, W. & Schmidt, M. H. (Hrsg.) (2008). Internationale Klassifikation psychischer Störungen ICD-10
Kapitel V (F), Klinisch-diagnostische Leitlinien (6., vollständig überarbeitete Auflage). Bern: Hans Huber.
Kaluza, G. (2015). Stressbewältigung. Trainingsmanual zur psychologischen Gesundheitsförderung. Berlin: Springer.
Kryspin-Exner, I. (2007). Klinische-Psychologie. In Kastner-Koller, U., & Deimann, P.(Hg.), Psychologie als Wissenschaft
(S.170-171). Facultas: Wien.
Kryspin-Exner, I. (2011). Klinische Psychologie II. Facultas: Wien.
Kirschbaum, C., & Heinrichs, M. (2011). Biopsychologische Grundlagen. In H.-U. Wittchen, & J. Hoyer, Klinische
Psychologie & Pschotherapie (S. 217-219). Berlin Heidelberg: Springer.
Laireiter, A.-R. (Hrsg.) (2000). Diagnostik in der Psychotherapie. Wien: Springer.
Pinel, J. P., & Pauli, P. (2007). Stress und Gesundheit. Biopsychologie (6. aktualisierte Aufl.), S.583-585. Pearson Stu-
dium: München
Stieglitz, R.-D., Baumann, U. & Freyberger, H. J. (Hrsg.) (2001). Psychodiagnostik in Klinischer Psychologie, Psychiatrie,
Psychotherapie (2. Auflage). Stuttgart: Thieme.
Tao Le et al. (2015). First aid for the USMLE Step 1 2015. New York: McGraw Hill

16
1. Bio-psycho-soziales Schmerzmodell
Schmerz wird heute als bio-psycho-soziales Gesamtphänomen aufgefasst, an dessen Entstehung und
Aufrechterhaltung neben der biologisch-physiologischen (sensorischen) Ebene auch affektiv-emotio-
nale, motivationale und evaluativ-kognitive Faktoren beteiligt sind. Darüber hinaus sind interaktio-
nelle und soziokulturelle Faktoren des Schmerzerlebens und Schmerzausdrucks als Determinanten
von Chronifizierungsprozessen wirksam. Die klassische Dichotomisierung von „somatischen“ und
„psychischen“ (oder „psychogenen“) Schmerzen kommt zwar dem Bedürfnis nach klar abgrenzbaren
klinischen Entitäten nahe, ist aber vor dem Hintergrund der modernen klinischen und neurobiologi-
schen Forschung heute nicht mehr sinnvoll. An Stelle eines „entweder-oder“ stellt sich nun die Frage,
wie biologische und psychosoziale Faktoren individuell pathogenetisch bzw. pathoplastisch wirksam
werden können.
Geht man nun von einem mehrdimensionalen Schmerzverständnis aus, so ist in der Diagnostik und
Therapie von Schmerzkranken auf eine ausgewogene Berücksichtigung biologischer, psychologi-
scher und soziokultureller Aspekte zu achten. Die Integration dieses ganzheitlichen Denkens in den
klinischen Alltag im Sinne einer interdisziplinären Schmerztherapie ist vielerorts noch Vision. Ein erster
wichtiger Schritt in diese Richtung war die Definition von Schmerz durch die Internationalen Schmerz-
gesellschaft (IASP): „Schmerz ist ein unangenehmes Sinnes- und Gefühlserlebnis, das mit aktueller
oder potentieller Gewebsschädigung verknüpft ist oder mit Begriffen einer solchen Schädigung be-
schrieben wird“ (Merskey & Bogduk 1994). Diese Definition enthält einige relevante Aspekte einer
modernen Schmerzkonzeption:
- Schmerz ist immer gleichzeitig ein sensorisch-nozizeptives Erlebnis (damit vergleichbar anderen
Sinnesmodalitäten, wie sehen, hören, riechen, schmecken oder fühlen) und ein emotionales Er-
lebnis (damit vergleichbar anderen Primäremotionen wie Angst, Trauer, Wut, Neid, Ekel oder
Zufriedenheit). Beide Aspekte gelten als gleichberechtigte, untrennbare Komponenten der
Schmerzerfahrung. Hierdurch wird die Multidimensionalität des Schmerzes betont.
- Die Betonung des Erlebnisaspektes von Schmerz unterstreicht die Subjektivität der Schmerzwahr-
nehmung. Schmerz hat damit mehr Ähnlichkeit mit der Erfassung innerer Zuständen wie Hunger
oder Müdigkeit und weniger mit sensorischen Sinnesqualitäten wie Sehen oder Hören.
- Eine Gewebsschädigung ist zwar oft Auslöser der Schmerzempfindung, die Schmerzerfahrung
selbst beruht aber nicht notwendigerweise auf einer Gewebsschädigung. Schmerzzustände ohne
ausreichende medizinische Erklärung fallen daher unter den gleichen Schmerzbegriff.

Schmerz ist ein unangenehmes bio-psycho-soziales Erlebnis, bei dem sensorische (biologische) und
emotionale (psychosoziale) Aspekte untrennbar miteinander verbunden sind. Gewebsschädigungen
sind zwar oft Auslöser der Schmerzempfindung, aber nicht zwingend notwendig für eine Schmerzer-
fahrung. Die Unterscheidung zwischen „körperlichen“ und „seelischen“ Schmerzen ist nicht trenn-
scharf und ist heute nicht mehr sinnvoll.

2. Chronifizierung von Schmerz

Akuter Schmerz ist in der Regel kurz andauernd und üblicherweise somatisch-nozizeptiv ausgelöst
(durch Verletzungen, Entzündungen, körperliche Fehlfunktionen etc.). Akuter Schmerz führt üblicher-
weise zu akuten vegetativen Reaktionen (z.B. Schwitzen, Muskeltonuserhöhung) und akuten psychi-
schen Veränderungen (z.B. Ängstlichkeit, Betroffenheit, Schlafstörung). Der akute Schmerz besitzt
eine biologische Warnfunktion, indem er auf zugrundeliegende (organ-)pathologische Prozesse hin-
weist und zu einer unmittelbaren Begrenzung der potentiellen Gewebsschädigung führt (z.B. Entfer-
nen der Hand von der heißen Herdplatte). Gleichzeitig hat akuter Schmerz eine rehabilitative Funk-
tion, indem er (z.B. bei Unfällen oder Entzündungen) zur Ruhe und Schonung zwingt.

17
Abbildung 1: Chronifizierung von Schmerz: Bio-psycho-soziale Modell

Mit zunehmender Schmerzdauer finden auf somatischer und psychosozialer Ebene Chronifizierungs-
vorgänge statt, die eine sekundäre Kausalkette für die weitere Aufrechterhaltung des Schmerzes dar-
stellen. Der chronische Schmerz „verselbständigt“ sich zunehmend von seiner ursprünglich auslösen-
den Ursache und verliert somit seine biologische Warnfunktion; er ist nicht mehr Hinweis auf eine
zugrundeliegende Verletzung oder Erkrankung, sondern ist selbst zu einer eigenständigen Erkran-
kung geworden (ab einer Schmerzdauer von mehr als 3 Monaten). Während bei akuten Schmerzzu-
ständen häufig den somatischen Faktoren eine zentrale Rolle zukommt, gewinnen mit zunehmender
Chronifizierung die psychosozialen Aspekte des Schmerzerlebens und der Schmerzverarbeitung an
Bedeutung. Bei vielen Betroffenen ist das Ausmaß erlebter Schmerzen und die subjektive Beeinträch-
tigung bzw. Behinderung durch die Schmerzen (sog. „pain disability“) nicht progressiv linear zum
organmedizinischen Befund. Die fehlende Berücksichtigung psychosozialer Aspekte bei der Schmerz-
diagnostik und Therapieplanung führt fälschlicherweise zu einer einseitig medizinischen Sichtweise
von subjektivem Leiden.
Folgende Faktoren spielen bei der Schmerzchronifizierung im Sinne eines Wechselwirkungsmodells
(gegenseitige Verstärkerprozesse, siehe auch Abbildung 1) eine Rolle:
Tab. 1: Faktoren der Schmerzchronifizierung
Biologische Ebene Psychosoziale Ebene
Prädisponierende Fak- Genetische Vulnerabilität: Opiatre- Persönlichkeitsmerkmale: Schwierigkeiten
toren zeptor-Polymorphismus, Neuropeptid- in der Emotionswahrnehmung und -regu-
Metabolismus lation („Alexithymie“, „repression/sen-
sitization“, „negative affectivity“), Selbst-
Körperliche Vorerkrankungen: körper-
wertmangel, übermäßiger Leistungsan-
liche Fehlhaltung, postinfektiöse neu-
spruch
ronale Sensibilisierung, etc.
Psychische Traumatisierungen in der Vor-
geschichte: Misshandlung, Missbrauch,
Verwahrlosung, Verlust einer zentralen Be-
zugsperson, Verfolgung, Krieg, Folter, etc.

18
Schmerzauslösende Verletzungen, Entzündungen, auto- „life events“: häufig Verlusterfahrungen,
Faktoren nome Übererregbarkeit, erhöhtes An- Beziehungskrisen, aber auch Migration o-
spannungsniveau, Schmerzschwellen- der akute Traumatisierungen
änderung durch Schlafstörungen und
Chronische Belastungen und Konflikte (am
Inaktivität
Arbeitsplatz und/oder in der Partnerschaft
bzw. Familie)
Aggressive Störung/ Angststörung: negati-
ver Prädiktor für eine Schmerzentwick-
lung/- chronifizierung
Aufrechterhaltende Neuronale Sensibilisierung: Erlernen von Schonhaltungen durch
Faktoren NMDA-Rezeptor-Sensibilität, Substanz- (angstassoziierte) Schmerzvermeidung
P-Metabolismus etc.
Aufmerksamkeitsfokussierung auf Schmerz
Wechselwirkung Muskelspannung -
Abhängigkeit des Schmerzausdrucks von
Schmerz – Schonhaltung - muskuläre
Umgebungsfaktoren („sekundärer Krank-
Dysbalance
heitsgewinn“)

Weiterführende Literatur:
Bach M, Aigner M, Bankier B (Hrsg.) Schmerzen ohne Ursache – Schmerzen ohne Ende. Konzepte – Diagnostik – Therapie.
Facultas-Verlag, Wien 2001.
Egle UT, Hoffmann SO, Lehmann KA, Nix WA (Hrsg.) Handbuch Chronischer Schmerz. Grundlagen, Pathogenese, Klinik
und Therapie aus bio-psycho-sozialer Sicht. Schattauer Verlag: Stuttgart (2003).
Flor H. (2003): Chronische Schmerzsyndrome. In: Ehlert, Ulrike (Hrsg.) Verhaltensmedizin. Berlin: Springer (Springer-Lehr-
buch), S. 183–221.
Fritsche G., Gaul Ch. (2013). Multimodale Schmerztherapie bei chronischem Kopfschmerz – Interdisziplinäre Behand-
lungskonzepte. Thieme: Stuttgart.
Kröner-Herwig B. (2004). Die Wirksamkeit von Verhaltenstherapie bei psychischen Störungen von Erwachsenen sowie
Kindern und Jugendlichen. Expertise zur empirischen Evidenz des Psychotherapieverfahrens Verhaltenstherapie. Tübingen:
dgvt-Verlag.
Kröner-Herwig B., Frettlöh J., Klinger R., Nilges P. (Hrsg.) (2007). Schmerzpsychotherapie. Grundlagen – Diagnostik –
Krankheitsbilder – Behandlung, 6. Auflage. Springer Verlag.
Merskey H, Bogduk N (Eds.) (1994). Classification of Chronic Pain: Descriptions of Chronic Pain Syndromes and Defini-
tions of Pain Terms. 2nd ed., IASP Press, Seattle, p. 210

19
Stressmanagement und Ressourcenaktivierung in der klinisch-psychologischen
und psychotherapeutischen Behandlung
Maga. Johanna Granegger, Maga. Kristina Koiner, Maga. Elisabeth Wallner

1. Was ist Stress - eine kurze Definition


Aus biologischer Sicht bezeichnet der Stressbegriff einen psychophysischen Zustand, bei dem
Abweichungen von der Homöostase vorliegen, die durch die verfügbaren, routinemäßigen
Reaktionen nicht kompensiert werden können (Kaluza, 2018). Die Reize, die die Abweichungen von
dieser Homöostase erzeugen, nennt man Stressoren.
Jede Stressreaktion verbraucht erhebliche Anpassungsenergien – können diese wegen anhaltender
Belastungen und Bedrohungen in Ruhephasen nicht wieder angefüllt werden, setzt die negative
Wirkung der Stressoren ein. (Hurrelmann, 2010).

1.2 Folgen von Stress (Eckert & Tarnowski, 2017; Kaluza, 2018)

1.2.1 Kurzfristige Folgen von Stress:


Gesundheitlich scheint kurzfristiger Stress unbedenklich zu sein. Es erfolgt eine erhöhte Anspannung
und Nervosität. Dabei ist eine Zunahme der Immunkompetenz zu beobachten. Kurzfristiger Stress
kann sich förderlich auf Leistungen auswirken (vgl. Eckert & Tarnowski, 2017): bei
leistungsbezogenen Aufgaben, die eine geringere Komplexität aufweisen und mit Bewegung
verbunden sind. Komplexe Aufgaben werden allerdings am besten bei einem mittleren
Anspannungsniveau gelöst. Wenn die stressbedingte Anspannung jedoch über dieses Niveau
hinaussteigt, nimmt die Leistung ab und mehr Fehler schleichen sich ein. Ebenso sinken Konzentration
und Erinnerungsleistung ab einer gewissen Anspannung. Die gestiegene Anspannung kann vermehrt
zu interpersonellen Konflikten führen, da Stress als erhöhte Kampf- oder Fluchtbereitschaft aufgefasst
wird.

1.2.2 Mittelfristige Folgen von Stress:


Wird die stressbedingte Aktivierung über eine längere Zeit aufrechterhalten, verkehrt sie sich in ihr
Gegenteil. Aus Aktivierung und Wachheit wird bei unzureichender Regeneration mittelfristig
emotionale Erschöpfung. Die zunächst erhöhte Immunkompetenz kann sich in eine Schwächung des
Immunsystems verkehren, entzündliche Prozesse und Krankheiten (z.B. Erkältungen) nehmen zu. Es
können sich oftmals dysfunktionale Regulationsversuche (wie ungesunde Essensgewohnheiten oder
Alkoholkonsum) etablieren, die sich längerfristig negativ auf die Gesundheit auswirken.

1.2.3 Langfristige Folgen von Stress:


Können die oben beschriebenen Teufelskreise nicht durchbrochen bzw. verlassen werden, besteht die
Gefahr der Chronifizierung möglicher physischer und psychischer Störungen.
Eine häufige physische Folge ist stressassoziierter Bluthochdruck. Arteriosklerose und entzündliche
Prozesse (wie Arthritis) werden durch Stress mitverursacht. Längerfristige stressbedingte Erhöhungen
des Blutzuckers, der Blutfette, des Körpergewichts und des Blutdrucks sind Risikofaktoren für einen
Herzinfarkt, einen Schlaganfall, einen Tinnitus, einen Hörsturz oder einen Typ-II-Diabetes.
Psychische Störungen können ebenfalls durch Stress ausgelöst, aufrechterhalten oder zumindest
mitbedingt werden. Eine starke Beeinträchtigung des sozialen Netzes, der wahrgenommenen
Leistungsfähigkeit, der wahrgenommenen Handlungskontrolle und der Lebenszufriedenheit können
das Risiko für depressive Störungen oder Angststörungen erhöhen. Durch langanhaltenden Stress
werden die Cortisol-Regelkreise gestört, was bei einer Depression zu einem chronisch erhöhten
Cortisolspiegel führen kann. Durch langanhaltenden Stress kann ebenso das Belohnungssystem mit
seiner Dopamin- und Serotoninregulation so beeinträchtigt werden, dass die Fähigkeit zum positiven
Empfinden und Genuss gestört ist.
Aus chronifizierten, dysfunktionalen Bewältigungsversuchen wie Alkohol- oder Substanz-missbrauch
resultieren häufig Abhängigkeitserkrankungen.

20
1.3. Das Transaktionale Stressmodell nach Lazarus (Drexler, 2006; Kaluza, 2018)
Die ersten Forschungsansätze zur Entwicklung einer Theorie psychologischer Stressreaktionen führte
Lazarus in den 1960ern durch. Im Rahmen dieser Theorie wird Stress als dynamischer, relationaler
Prozess zwischen Individuum und Umwelt aufgefasst. Wesentliche individuelle Variablen sind dabei
die kognitive Bewertung durch den Betroffenen („cognitive appraisal“) und die unterschiedlichen
Bewältigungsformen („coping“).

Lazarus beschreibt die kognitive Bewertung in drei Schritten: Geschehnisse in der Umgebung werden
prinzipiell daraufhin überprüft, ob sie bedeutsam für das persönliche Wohlbefinden sind („primary
appraisal“). Dabei stellt man sich Fragen wie: „Was passiert überhaupt? Ist das Ereignis relevant für
mich? Ist es herausfordernd oder bedrohlich?
Wenn das Ergebnis dieser Einschätzung stressrelevant ist, dann wird beurteilt, ob die verfügbaren
externen oder internen Ressourcen hilfreich sind, um das Problem zu bewältigen („secondary
appraisal“). Ob sich jemand durch ein Problem (positiv) herausgefordert oder bedroht fühlt, hängt
wesentlich von der Einschätzung der persönlichen Ressourcen (Bewältigungsfähigkeiten und
Bewältigungsmöglichkeiten) ab. Im Bewertungsprozess der belastenden Situation kann es durch neue
Erfahrungen und Informationen auch zu einer Neubewertung des Problems kommen („reappraisal“).
Es gibt formale und inhaltliche Parameter, die den Stressprozess zusätzlich beeinflussen. Formale
Parameter sind unter anderem Kontrollierbarkeit, Vorhersagbarkeit, zeitliche Nähe und Dauer des
Stressors. Bei den inhaltlichen Parametern handelt es sich um physische, soziale und
selbstwertrelevante Parameter. Es sind die sogenannten „daily hassles“ (alltägliche, anhaltende
Stressoren), die laut Lazarus gesundheitsrelevanter als sogenannte kritische Lebensereignisse sind.
Eine Stressreaktion entsteht gemäß dem transaktionalen Stressmodell immer dann, wenn eine
subjektiv wahrgenommene Diskrepanz zwischen den Anforderungen einer Situation und den
verfügbaren Ressourcen zur Bewältigung dieser Situation besteht. Die Einschätzung erfolgt
automatisiert und ohne bewusste Überlegung.
Bei der Stressbewältigung („Coping“) wird im Rahmen des transaktionalen Stressmodells zwischen
problemzentrierten (instrumentellen) und emotionsbezogenen (palliative) Bewältigungsstrategien
unterschieden. Instrumentelle Bewältigungsstrategien zielen auf die Veränderung des Problems durch
Handlungen oder Aktivitäten wie Auseinandersetzung, Kontrollverhalten oder Ausbau von
Kompetenzen ab. Palliative Bewältigungsstrategien setzen sich mit der Veränderung des Befindens
der betroffenen Person auseinander, ohne dabei den Stressor selbst zu beeinflussen (z.B.
Entspannung, kognitive Umstrukturierung, Ablenkung, etc.). Diese Strategien eignen sich vor allem
für Stressoren, die nicht kontrollierbar oder veränderbar sind.

Abb. 1: Transaktionales Stressmodell nach Lazarus

Das Modell von Lazarus hat zwar mittlerweile einige berechtigte Kritik erfahren, da es inzwischen ein
differenzierteres Verständnis der Bewertungsprozesse gibt, dennoch hat das Modell auch heute noch
einen hohen heuristischen Wert im Rahmen der klinisch-psychologischen Arbeit (Eckert & Tarnowski,
2010).

21
1.4. Die Theorie der Ressourcenerhaltung von Hobfoll (Drexler, 2006; Eckert & Tarnowski, 2017)

Das Stressmodell von Hobfoll, genauer bezeichnet als „Die Theorie der Ressourcenerhaltung“ befasst
sich alternativ zur Theorie von Lazarus nicht mit individuellen Bewertungsprozessen, sondern
konzentriert sich auf den Erhalt und den Verlust von Ressourcen (Eckert & Tarnowski, 2017). Die
Ressourcenerhaltungstheorie basiert auf der Annahme, dass Menschen danach streben und eine
grundlegende Motivation haben, für sie wertvolle Ressourcen zu erwerben, zu erhalten und zu
schützen (Drexler, 2006). Wobei in Hinsicht auf die Ressourcenerhaltung das Vermeiden von
Ressourcenverlusten ein stärkeres Motiv darstellt, als neue Ressourcen zu gewinnen (Drexler, 2006;
Eckert & Tarnowski, 2017).
Ressourcen können dabei materieller Natur (z.B. Nahrungsmittel, Kleidung, Wertsachen, etc.), nicht
materieller Natur (z.B. Familienstand, soziales Netz, Arbeitsplatzsicherheit, etc.), persönlich-
keitsbezogen (z.B. Überzeugungen/Werthaltungen, Selbstwertgefühl und Fähigkeiten, etc.) und
Energieressourcen (Zeit, Geld, Wissen, körperliche, mentale Fitness, etc.) sein (Drexler, 2006).

Der Ressourcenerhalt und/oder die Ressourcenvermehrung kann durch kritische Lebensereignisse


gefährdet und verhindert werden. Daraus erschließen sich im Rahmen des Stressmodells von Hobfoll
weitere Erklärungen für die Entstehung von Stress. Stress tritt dann auf, wenn in Bezug auf die eigenen
Ressourcen Verluste erlebt und Instabilität oder Gefährdung wahrgenommen werden, wenn
Handlungsmöglichkeiten zur Förderung oder zum Schutz vorhandener Ressourcen fehlen oder wenn
Ressourcen nach Investition nicht wieder erneuert werden können (Drexler, 2006).

Geht es in weiterer Folge um den Umgang und die Bewältigung von Stress, lässt sich die Annahme
ableiten, dass Menschen, die über viele und gute Ressourcen verfügen, sich gegenüber möglichen
Schicksalsschlägen besser behaupten können und auch eher ihren Ressourcenpool über Investitionen
erweitern können. Stressbewältigung erfolgt, indem Menschen versuchen durch Investitionen von
Ressourcen einem drohenden oder tatsächlichen Verlust von Ressourcen entgegen zu wirken. Dieser
Prozess beinhaltet die Möglichkeit der Ressourcenersetzung, des Ressourcenaustausches, der
Selektion, der Optimierung und der Kompensation von Ressourcen. Die Selektion von Ressourcen
meint den Versuch, sich auf das Wesentliche und das Machbare in der neuen Situation zu
konzentrieren. Bei der Optimierung geht es darum, bestimmte Ressourcen gezielt zu trainieren und
diese somit zu stärken und die Kompensation als Bewältigungsmechanismus von Stress hat das Ziel,
die fehlende Passung zwischen Anforderung und Bewältigung auszugleichen (Drexler, 2006).

Abb. 2: Theorie der Ressourcenerhaltung nach Hobfoll

2. Ressourcenaktivierung
Damit Psychotherapie bzw. klinisch-psychologische Behandlung als Gesundungs- und
Heilungsprozess verstanden werden kann, muss neben der Betrachtung der problematischen
Faktoren (pathologische Sichtweise) der Fokus vor allem auch auf das Wohlbefinden/ auf die
Ressourcen (salutogenetische Sichtweise) gelegt werden (Frank, 2010; Kanfer, 2012).

22
Im Rahmen der klinisch-psychologischen bzw. psychothrapeutischen Therapie geht es unter anderem
auch um diese Aktivierung von Ressourcen und um das (Wieder-)Herstellen von Wohlbefinden.
Ressourcenaktivierung umfasst hierbei einerseits das „Erkennen und Stützen der bereits vorhandenen
positiven Seiten und Fähigkeiten“ eines Menschen und andererseits auch die Erweiterung von
Ressourcen „durch gezielte Vermittlung neuer Informationen und durch Einübung neuer und bis
dahin ungewohnter Bewältigungskompetenzen“ (Fiedler, 2011, S. 20).
In der klinisch-psychologischen bzw. psychotherapeutischen Grundlagenforschung, die sich mit der
Aufklärung von Wirkfaktoren beschäftigt, nimmt die Erforschung der Bedeutsamkeit von protektiven
Faktoren, z.B. im Sinne von schützenden Eigenschaften oder allgemeinen Ressourcen der Person und
ihrer Umwelt, eine zunehmend wichtigere Rolle ein (Beesdo-Baum, 2011).
Ressourcenaktivierung hat mittlerweile in jeder psychotherapeutischen Richtung (z.B. Ver-
haltenstherapie, Systemische Familientherapie, Existenzanalyse, Psychodrama, ...) einen hohen
Stellenwert und ist laut Grawe (1998) – neben der Problemaktualisierung, Problembewältigung und
motivationalen Klärung – einer der vier bedeutendsten „schulenübergreifenden“ Wirkfaktoren der
Psychotherapie.

2.1 Arten von Ressourcen (Beesdo-Baum, 2011; Willutzki & Teismann, 2013)
Ressourcen können subjektiv (vom Handelnden selbst) und/oder objektiv (von anderen Personen
beobachtet und zugeschrieben) wahrgenommen werden. In der klinisch-psychologischen und
psychotherapeutischen Behandlung sind vor allem die subjektiv wahrgenommenen Ressourcen von
Bedeutung bzw. deren Aufbau und Entwicklung können zum Ziel der Behandlung werden.
Weiters werden folgende Kategorien von Ressourcen unterschieden:
- extern: natürliche, soziale und technische Hilfsmittel bzw. HelferInnen in der Umwelt (z.B. soziale
Netzwerke, sozioökonomischer Status, kulturelle Güter, der in einer Situation gegebene
Handlungs- und Kontrollspielraum, ...)
- interpersonell: Beziehungsmuster und -charakteristika, die sich positiv auf soziale Beziehungen
auswirken (z.B. gegenseitiger Respekt, Verlässlichkeit, Fähigkeit zur Wiedergutmachung, …)
- intrapersonell: Persönlichkeitsvariablen, persönliche Fähigkeiten, Fertigkeiten und Kräfte der
Person (z.B. Selbstwertgefühl, Optimismus, Kontrollüberzeugung, Flexibilität, ...)

2.2 Mögliche Methoden/Strategien zur Ressourcenaktivierung im Rahmen der klinisch-psycho-


logischen bzw. psychotherapeutischen Behandlung

- Training von sozialen Fertigkeiten: z.B. Durchsetzen von eigenen Bedürfnissen lernen, Nein
sagen üben, Verhalten in Konfliktsituationen, etc.
- Übungen zur Selbstwertstärkung: z.B. sich selbst loben, Übungen zum fairen Blick auf sich
selbst, Selbstfürsorgeprotokolle, Aktivitätsaufbau, Methoden zur Wahrnehmung eigener
Stärken, Tankmodell, etc.
- Training von Fertigkeiten zur Emotionsregulation: z.B. Skillstraining
- Achtsamkeitstraining: Übungen mit den 5 Sinnen (Fokus auf Beschreibung ohne Bewertung)
- Genusstraining: Genussübungen, Genussregeln, Liste angenehmer Tätigkeiten, etc.
- Entspannungsverfahren: z.B. Progressive Muskelrelaxation, Autogenes Training, z.T. auch mit
Einsatz von Biofeedback, etc.
- stabilisierende Imaginationen: z.B. Innerer sicherer Ort, Phantasiereisen, ...
- Skalierende Fragen: mithilfe von Skalen (z.B. 0-10) werden der Ist- und Soll-Zustand eines
aktuellen Problems bzw. der aktuelle Stand der Problembewältigung bewertet (z.B. Woran
merken Sie, dass Sie jetzt bei 4 sind? Wie würden Sie handeln/denken, wenn Sie bei 5 sind?
Was müssten Sie probieren, um auf 5 zu kommen?)
- Suche nach Ausnahmen des umschriebenen Problems: Bewusstmachung von Ausnahmen
des Problems und auf eigene Handlungsanteile hin analysieren
- Wunderfrage: „Wenn das Problem über Nacht verschwunden wäre …“ als Ausgangspunkt
für konkrete Lösungsansätze

23
- Psychoedukation (Wissensvermittlung) zum Wohlbefinden: z.B. gesunder Schlaf, Möglich-
keiten zur Stressreduktion, etc.
- Tagebuch „der kleinen Schritte“: z.B. Selbstbeobachtung mit Fokus auf (minimale) positive
und/oder neutrale Ereignisse/Emotionen im Alltag mittels Protokoll
- Bezugnahme auf erfolgreiche Bewältigung von konkreten schwierigen und krisenhaften
Lebenssituationen in der Vergangenheit
- Evaluation des Zeit-Managements: z.B. durchschnittliche Woche protokollieren lassen und
im Hinblick auf Aktivitäts- und Ruhezeiten reflektieren
- Einsatz von therapeutischen Geschichten und Weisheiten

Literatur:
Beesdo-Baum, K. (2011). Ressourcenaktivierung. In Wittchen, H.-U. & Hoyer, J. (Hrsg.). Klinische Psychologie und
Psychotherapie (S. 492 – 502). Berlin: Springer-Verlag.
Drexler D. (2006). Das Integrierte Stressbewältigungsprogramm ISP. Leben Lernen 187. Stuttgart: Klett-Cotta.
Eckert M. & Tarnowski T. (2017). Stress- und Emotionsregulation. Trainingsmanual zum Programm Stark im Stress.
Weinheim: Beltz.
Fiedler, P. (2011). Ressourcenorientierte Psychotherapie. In Frank, R. (Hrsg.), Therapieziel Wohlbefinden (2. Aufl.). Berlin:
Springer-Verlag.
Frank, R. (2010). Wohlbefinden fördern. Stuttgart: Klett-Cotta.
Grawe, K. (1998). Psychologische Therapie. Göttingen: Hogrefe.
Hurrelmann K. (2010). Gesundheitssoziologie. Eine Einführung in sozialwissenschaftliche Theorien von Krankheits-
prävention und Gesundheitsförderung. (7. Aufl.) Weinheim, München: Juventa-Verlag.
Kanfer, F., Reinecker, H. & Schmelzer, D. (2012). Selbstmanagement-Therapie (5. Aufl.). Berlin: Springer-Verlag.
Kaluza, G. (2018). Stressbewältigung. Trainingsmanual zur psychologischen Gesundheitsförderung (4.Aufl.). Berlin:
Springer-Verlag.
Willutzki, U. & Teismann, T. (2013). Ressourcenaktivierung in der Psychotherapie. Göttingen: Hogrefe Verlag.

24
Klinische Neuropsychologie
Mag. Rene Hackstock

Begriffsbestimmung
‚Die klinische Neuropsychologie beschäftigt sich mit den zentralnervösen Grundlagen des mensch-
lichen Verhaltens, Denkens und Empfindens und verwendet die Erkenntnisse ihrer Grundlagenwis-
senschaft zusammen mit den Methoden der allgemeinen und klinischen Psychologie bei der Diag-
nostik und Therapie von Patienten mit Hirnfunktionsstörungen.‘ (Sturm et al. 2000). Der Zugang der
klinischen Neuropsychologie zu PatientInnen erfolgt somit primär von der Funktionsseite her, weniger
von der neuroanatomischen.
Das wesentlichste Ziel der klinischen Neuropsychologie ist die differenzierte Erfassung von kognitiven
Funktionen und –Dysfunktionen im Sinne einer Diagnostik, als Grundlage einer gezielten Behand-
lung. Ebenso nimmt die psychologische Behandlung eine wichtige Rolle ein, da PatientInnen häufig
eine Unterstützung zur Verarbeitung der Erkrankung / Verletzung und deren Folgen bedürfen, bzw.
neurologische Schädigungen auch Psyche und Persönlichkeit verändern können. ‚Im Mittelpunkt der
klinisch-psychologischen Behandlung steht die Ressourcenarbeit, wobei es um die Akzeptanz und die
Integration der Erkrankung im Leben nach dem Ereignis oder der Diagnosestellung geht’ (Lehrner,
J. et al. 2011).

Kognitive Funktionen
Angeborene oder erworbene Schädigungen des Gehirns (z. B. vaskuläre Erkrankungen, raumfor-
dernde Prozesse, Schädel-Hirn-Traumata, entzündliche und/oder degenerative Prozesse) wirken sich
je nach Umfang und Lokalisation in unterschiedlichem Ausmaß auf kognitive-, aber auch emotio-
nale- und Persönlichkeitsaspekte aus. Daraus können unterschiedlichste Beeinträchtigungen im Le-
bensalltag der PatientInnen resultieren. Störungen können dabei grundsätzlich schleichend (z. B. bei
Hirntumoren, degenerative Prozessen) oder plötzlich (z. B. bei Schlaganfall, Blutungen) auftreten, sie
können progredient sein, persistieren oder sich wieder zurückbilden.
Unter dem Begriff kognitive Funktionen werden hier sämtliche geistige Leistungen subsummiert. Jede
Funktion ist neuroanatomisch mit dem Zusammenspiel spezifischer neuronaler Netzwerke verbun-
den, wobei dies aber nicht als strikte Lokalisationszuordnung zu verstehen ist. Im Folgenden sollen
grob die wichtigsten kognitiven Domänen, die mit ihnen verbundenen neuroanatomischen Strukturen
sowie mögliche Störungsbilder dargestellt werden. Eine ausführlichere Beschreibung würde den Um-
fang des Skriptes sprengen, weswegen nur das Wesentlichste angeführt werden soll.

• Aufmerksamkeit:
‚Aufmerksamkeit ist die klare und lebhafte Inbesitznahme des Verstandes von einem Objekt oder
Gedanken aus einer Menge anderer gleichzeitig möglicher Objekte und Gedanken. Ausrichtung
und Konzentration des Bewusstseins sind ihr Wesen.‘ (William James 1890). Die Fähigkeit, seine
Aufmerksamkeit auszurichten und aufrechtzuerhalten ist Grundlage für menschliches Denken und
Handeln und gehört zu den wichtigsten Grundfunktionen des Gehirns. Aufmerksamkeitsprozesse
lassen sich unterteilen nach Intensität (Aktivierung, Daueraufmerksamkeit und Vigilanz) und Selekti-
vität (fokussierte / selektive Aufmerksamkeit, geteilte Aufmerksamkeit, kognitive Flexibilität).
Wichtige neuroanatomische Strukturen sind das aufsteigende retikuläre Aktivierungssystem (ARAS),
(prä)frontaler, parietaler und cingulärer Kortex, Thalamus und noch weitere (wide spred, quasi 1x
quer durch den Schädel).
Störungen von Aufmerksamkeitsfunktionen sind die häufigsten (kognitiven) Folgeerscheinungen neu-
rologischer Schädigungen. Sie können einzelne, mehrere oder alle Subdimensionen betreffen. Ein
Sonderfall ist der sogenannte Neglect, eine Störung der Aufmerksamkeitsverlagerung (supramodal)
in die kontraläsionelle Raumhälfte, der zumeist nach rechts parietalen Schädigungen auftritt. Be-
troffene Patienten vernachlässigen die linke Raum- und Körperhälfte und zeigen häufig ein reduzier-
tes Störungsbewusstsein.

25
• Gedächtnis:
Es stellt unseren Informationsspeicher dar. Die Voraussetzung dafür, vorhandenes Wissen zu nutzen
und laufend neues Wissen hinzuzulernen, ist ein intaktes Gedächtnis. Die Unterteilung der Subsys-
teme erfolgt entlang der Zeitachse in sensorisches (Millisekunden), Kurzzeit- / Arbeits- (Sekunden bis
wenige Minuten) - und Langzeitgedächtnis. Die Inhalte des Langzeitgedächtnisses können wiederrum
unterteilt werden in unbewusst / implizit (prozedual, Konditionierung, Priming und Habituation) und
bewusst / explizit (episodisch / biographisch und semantisch / ‚Weltwissen‘). Als prospektives Ge-
dächtnis wird die Fähigkeit bezeichnet, sich zukünftig an Vorhaben zu erinnern (z. B.: Heute Abend
muss ich noch den Müll nach draußen bringen.)
Wichtige neuroanatomische Strukturen sind der Papez’sche Schaltkreis (Hippocampus, Mamillarkör-
per, anteriorer Thalamus und cingulärer Kortex) und der basolaterale Schaltkreis (Amygdala, medi-
odorsaler Thalamus und Teile des basalen Vorderhirns) sowie der temporale Kortex. Faktisches Wis-
sen und verbale Informationen werden primär linkshemisphärisch verarbeitet, episodische Erinne-
rungen und nonverbale Informationen primär rechtshemisphärisch.
Störungen des Gedächtnisses werden als Amnesien bezeichnet und können retrograd (betrifft Inhalte
vor der Schädigung) oder anterograd (betrifft den Erwerb neuer Information nach einer Schädigung)
ausfallen. Dabei können sie sich inhaltsspezifisch (verbal vs. nonverbal, in Einzelfällen auch hoch-
spezifisch z. B. autobiographisch oder Gesichter) oder auch global auswirken. Auch können unter-
schiedliche Prozesse selektiv betroffen sein (Enkodierung / Aufnahme / Arbeitsgedächtnis – Transfer
in das Langzeitgedächtnis – Abruf aus dem Langzeitgedächtnis / prospektives Gedächtnis). Vor allem
bei Schädigungen des orbitofrontalen Kortex kann es auch zu so genannten Konfabulationen kom-
men. Darunter versteht man episodische Fehlerinnerungen, die durch das (unbewusste) Ausfüllen
von Gedächtnislücken entstehen. Dabei kann es sich um (teils bizarre) Assoziationen handeln, die
jedoch eine gewisse innere Logik beibehalten und von PatientInnen als authentisch wahrgenommen
werden.

• exekutive Funktionen:
Ist ein Sammelbegriff für Funktionen, die steuernd und modulierend auf elementare oder routinierte
kognitive Fähigkeiten einwirken. Man könnte sie auch als Instanz für kognitive Kontrolle bezeichnen.
Sie dienen dazu, Handlungen über mehrere Teilschritte hinweg auf ein übergeordnetes Ziel zu pla-
nen, die Aufmerksamkeit auf hierfür relevante Informationen zu fokussieren und ungeeignete Hand-
lungen zu unterdrücken (Lehrner, J. et al., 2011). Beispielhaft genannt seien die Fähigkeiten zu:
Inhibition, Antizipation, Aufmerksamkeitsmodulation, Wechsel zwischen attentionalen Einstellungen,
Problemlösen, strategisches Abwägen, Monitoring und Evaluation. Auch Flexibilität und Arbeitsge-
dächtnis zählen zu diesem Funktionsbereich.
Als wichtige neuroanatomische Strukturen werden häufig primär Areale des präfrontalen Kortex an-
geführt, aber auch Schädigungen in anderen Bereichen (Basalganglien, Kleinhirn) können eine ‚dy-
sexekutive Symptomatik‘ zur Folge haben.
Störungen in diesen Funktionen werden oft als dysexekutives Syndrom umschrieben und können ent-
sprechend vorausgegangener Beschreibung sehr heterogen ausfallen. Betroffene, die an Störungen
dieser „höheren kognitiven Fähigkeiten“ leiden, zeigen sich oft interessenslos und gleichgültig. Hand-
lungen scheinen nicht zielorientiert, außerdem können Teilschritte meist nicht definiert werden. Au-
ßerdem fällt es Patienten mit beeinträchtigten Exekutivfunktionen schwer, bereits gefasste Pläne zu
modifizieren oder Störreize auszublenden (zu inhibieren). Gemeint sind demnach die Steuerung und
Kontrolle des eigenen Verhaltens. Betroffene weisen häufig eine fehlende oder eingeschränkte Krank-
heitseinsicht auf (Anosognosie/Anosodiaphorie), was die Behandlung von Defiziten der Exekutivfunk-
tionen erschwert. Außerdem können die PatientInnen auch Wesensveränderungen zeigen, die vor
allem die Emotionalität, den Antrieb und das Sozialverhalten betreffen. Die mangelnde interne Ver-
haltenskontrolle kann zu einem raschen Stimmungswechsel, heftigen Gefühlsausbrüchen oder vor-
schnellem Agieren führen, was meist für die Angehörigen ein massives Problem im Alltag bedeutet.

26
• Sprache:
Die Sprache ist unser geläufigstes ‚Kommunikationswerkzeug‘. Unterschieden wird hierbei zwischen
Sprachproduktion (Wortschatz, Syntax, Grammatik) und –verständnis.
Relevante neuroanatmoische Strukturen liegen beim gesunden Rechtshänder klassisch links fronto-
parietal (hier v. a. die perisylvische Sprachregion mit Broca- und Wernickeareal sowie gyrus angu-
laris).
Sprachstörungen werden als Aphasien bezeichnet und je nach Symptomatik grob unterteilt in Broca-
, Wernicke-, Amnestische- und Globale Aphasie, wobei auch Sonderformen existieren. Beurteilt wer-
den jeweils die Spontansprache (flüssig / nicht flüssig), Benennen, Nachsprechen und Sprachver-
ständnis. Als Paraphasien bezeichnet man Fehlbenennungen, die entweder semantisch (‚Messer‘ statt
‚Schere‘) oder phonematisch (‚Schelle‘ statt ‚Schere‘) einen Bezug zum Zielwort haben. Als Neolo-
gismen werden Wortneuschöpfungen bezeichnet, die ebenfalls einen semantischen (‚Blattschneider‘
statt ‚Schere‘) oder phonematischen (‚Schese‘ statt ‚Schere‘) Bezug zum Zielwort haben können. Bei
der Broca Aphasie (‚motorische‘ Aphasie) sind Spontansprache und Benennen schwer, Nachspre-
chen und Sprachverstänis leicht bis mäßig beeinträchtigt. Im Gespräch vermitteln diese Patienten viel
Information mit wenig Wörtern. Bei der Wernicke Aphasie (‚sensorische‘ Aphasie) ist die Spontan-
sprache flüssig, aber von Paraphasien und Neologismen geprägt. Das Sprachverständnis ist deutlich
beeinträchtigt, ebenso Benennen und Nachsprechen. Diese Patienten vermitteln wenig Information
mit vielen Wörtern. Bei der amnestischen Aphasie ist die Spontansprache flüssig, jedoch mit deutli-
chen Wortfindungsstörungen. Das Benennen ist beeinträchtigt, es können semantische Paraphasien
auftreten. Nachsprechen und Sprachverständnis sind nicht bis gering beeinträchtigt. Bei der globalen
Aphasie sind alle Sprachkomponenten schwer beeinträchtigt. Von Sprachstörungen abzugrenzen
sind Sprechstörungen, die sich auf Stimmgebung, Artikulation und Sprechrhythmus beziehen (z. B.
Dysarthrie, Sprechapraxie).

• visuelle Wahrnehmung:
Aus neuropsychologischer Sicht ist von den unterschiedlichen Sinneskanälen vor allem das Sehen
relevant. Unter visueller Wahrnehmung versteht man die zentrale Verarbeitung von visuellen Reizen.
Visuoperzeptive Funktionen spielen eine wichtige Rolle für den Gewinn von Informationen über un-
sere Umwelt sowie für die Steuerung unseres Verhaltens. Neuroanatomisch verläuft die Sehbahn von
der Netzhaut über den Nervus Opticus zum Thalamus (dort nucleus geniculatus lateralis) und dann
weiter zum occipitalen Kortex, wo die primäre Verarbeitung stattfindet. Das ‚semantische Erkennen‘
des Gesehenen sowie die räumliche Zuordnung finden im temporalen bzw. parietalen Kortex statt.
Störungen können je nach Schädigungsebene unterschiedlich ausfallen und entweder als Doppel-
bilder durch fehlende Augenkoordination, Gesichtsfeldausfall (Anopsie), mangelnde Gesichter- oder
Objekterkennung (Agnosie) sowie visuell- räumliche oder visuo-konstruktive Defizite in Erscheinung
treten. Auch hier sei wieder kurz auf den Neglect verwiesen, bei dem es zur Vernachlässigung von
Reizen aus der kontraläsionellen Seite kommt.

Grundsätze neuropsychologischer Diagnostik


Im klinischen Alltag sollten diese kognitiven Funktionen hypothesengeleitet durch eine standardisierte
(d. h. Vergleich mit den Leistungen einer gesunden Normstichprobe) Diagnostik überprüft werden.
Dabei sollte bereits vorab eine Fragestellung festgelegt werden, für deren Beantwortung aus einer
Vielzahl an Testverfahren (sowohl Papier-Bleistift als auch computergestützt) die geeignetsten ausge-
wählt werden (u. a. sollten dabei Validität, Anforderung und Normierung berücksichtigt werden).
Zusätzlich liefern eine ausführliche Eigen- und/oder (wenn möglich) Außenanamnese sowie eine
genaue Verhaltensbeobachtung in der Testsituation wertvolle Informationen.

Neuropsychologische Behandlung
Bei der Behandlung neuropsychischer Defizite werden je nach Art und Schweregrad der Störung
verschiedene Ansätze verfolgt (Gauggel, S. 2003).

27
• restitutiv:
Dabei sollen geschädigte neuronale Netzwerke durch eine intensive und wiederholte Stimulation der
beeinträchtigten Funktion teilweise oder vollständig wieder reaktiviert werden. An Arbeitsmaterialien
stehen hierfür sowohl computerbasierte, als auch Papier-Bleistift Verfahren, die sehr individuell an
die PatientInnen angepasst werden, zur Verfügung. Dieser Ansatz ist vor allem bei Aufmerksamkeits-
und leichten Sprachstörungen sinnvoll. Auch in den Bereichen Gedächtnis und exekutive Funktionen
existieren diverse restitutive Behandlungsansätze, allerdings sind diese nur bei umschriebenen bzw.
leichteren Einschränkungen sinnvoll, da sich hierbei selten generalisierende Fortschritte erzielen las-
sen.
• kompensatorisch:
Der kompensatorische Behandlungsansatz, bei dem der Ausgleich der Funktionsdefizite durch die
Verwendung von Hilfsmitteln, den Einsatz verbliebener Fähigkeiten bzw. durch das Lernen neuer
Fertigkeiten und Strategien, aber auch der emotionale Umgang mit den Folgen der Schädigung im
Mittelpunkt stehen, spielt insbesondere in der postakuten Phase eine sehr wichtige Rolle. Dabei geht
es auch um die Modifikation von Erwartungen und Anforderungen (internen und externen).
• integrierte Verfahren:
Darunter werden Methoden anderer psychotherapeutischer Richtungen verstanden, die ebenfalls in
der neuropsychologischen Behandlung zum Einsatz kommen. So etwa operante Techniken, Token-
Programme, Selbstinstruktionstechniken, Zielsetzungstechniken, Feedback-Interventionen oder auch
therapeutische Interventionen mit Angehörigen.

Organisch bedingte Veränderungen von Psyche und Persönlichkeit


Verletzungen oder Erkrankungen des Gehirns können auch zu organisch bedingten Veränderungen
der psychischen Funktionen und der Persönlichkeit des Patienten führen. Diese sind von reinen psy-
chischen Reaktionen zu unterscheiden und werden im ICD-10 unter den Kategorien F06 (sonstige
psychische Störungen aufgrund einer Schädigung oder Funktionsstörung des Gehirns oder einer
körperlichen Krankheit) und F07 (Persönlichkeits- und Verhaltensstörungen aufgrund einer Schädi-
gung oder Funktionsstörung des Gehirns) geführt. Derartige Störungen treten zumeist bei Schädi-
gungen des (prä-)frontalen Kortex bzw. des limbischen Systems auf. Unter F06 fallen dabei Störungen
wie Organisch affektive- oder wahnhafte Störung, aber auch die leichte kognitive Störung. Unter
F07 werden auffällige Veränderungen gegenüber dem prämorbiden Verhalten beschrieben. Solche
Veränderungen betreffen besonders tiefgreifend die Äußerungen der Affekte, Bedürfnisse und Im-
pulse sowie die Fähigkeit eigene Handlungen zu planen und ihre wahrscheinlichen persönlichen und
sozialen Konsequenzen vorauszusehen (z. B. Organisches Psychosyndrom nach Schädelhirntrauma).

Psychische Reaktionen auf neurologische Erkrankungen


Neurologische Erkrankungen / Schädigungen und ihre Folgen stellen für PatientInnen und ihre An-
gehörigen auch eine psychische Belastungssituation dar, die es zu bewältigen gilt. PatientInnen füh-
len sich oft mit ihrer körperlichen Verwundbarkeit und Endlichkeit konfrontiert, erleben Kontrollverlust
und Hilflosigkeit, fallen aus gewohnten Rollen etc. Hier ist es wichtig, ‚normale’ Copingstrategien
von pathologischen zu unterscheiden, sprich festzustellen, ob es sich bei dem beobachtbaren Ver-
halten um ein krankheitswertiges handelt oder nicht. Der Übergang ist dabei zumeist fließend. Eine
leichte depressive Verstimmung oder gelegentliches Weinen in Reaktion auf eine neurologische Er-
krankung muss nicht zwangsweise pathologisch sein, sondern kann durchaus auch als ‚Katharsis‘
erlebt werden. Prinzipiell empfiehlt es sich bei Verdacht auf psychischen Leidensdruck immer, Pati-
entInnen oder auch die Angehörigen direkt empathisch anzusprechen und Unterstützung anzubieten.
Die begründete Erwartungshaltung die dahinter steht ist, neben dem ethischen Grundrecht auf psy-
chische Gesundheit, das psychisch ‚gesunde’ PatientInnen in höherem Ausmaß von den therapeuti-
schen und medizinischen Anwendungen profitieren können, sowie auch mit Einschränkungen eher
wieder ihren ‚Platz im Leben’ finden.

28
Gerade leichtere kognitive Einschränkungen bleiben ohne neuropsychologische Abklärung oft uner-
kannt bzw. unbemerkt. Nach längerer Zeit fallen solche Patienten jedoch häufig durch diffuse Be-
schwerden wie z. B. vegetative und emotionale Störungen (Kopfschmerzen, Schwitzen, Übelkeit,
Müdigkeit / Erschöpfung, ‚Reizüberflutung‘, etc.) auf, die durch Überbeanspruchung auf Grund von
vermehrten kompensatorischen Anstrengungen entstehen (sogenannte Coping-Hypothese).

Neurorehabilitation
Die Neurorehabilitation ist ein Teilgebiet der Neurologie, in der die Diagnostik und Therapie von
individuellen Funktionen im Vordergrund stehen. Ziel ist es, den Betroffenen eine Wiedereingliede-
rung in sein soziales System zu ermöglichen. Das Behandlungskonzept wird als interdisziplinär auf
jeden Patienten individuell ausgerichteter Prozess angesehen. Das Wiedererlernen verlorener Funk-
tionen und somit die größtmögliche Selbstständigkeit von Betroffenen wird demnach von einem mul-
tiprofessionellen Team (bestehend aus ÄrztInnen, Pflegepersonal, Klinische NeuropsychologInnen/
Klinische PsychologInnen, PhysiotherapeutInnen, ErgotherapeutInnen, LogopädInnen, Sozialarbeite-
rInnen/ Care- und CasemanagerInnen) begleitet.

Die Österreichische Gesellschaft für Neurorehabilitation (GNR) hat die Notwendigkeit der Einteilung
der Neurorehabilitation in Phasen dargelegt. Es soll angelehnt an die europäischen Phasenmodelle
(insbesondere der deutschsprachigen Länder) die Vergleichbarkeit der Betreuungsinstitutionen sowie
die Evaluation der Behandlungsergebnisse ermöglichen. Der Übergang von einer Phase in eine an-
dere ist dabei aber variabel und fließend. Nicht jede/r PatientIn durchläuft zwingend jede Phase,
außerdem sind Rückfälle in frühere Phasen möglich.

In die Akutphase (Phase A) werden PatientInnen mit jeder akuten neurologischen Erkrankung oder
auch akuten Verschlechterung einer chronischen Erkrankung in den ersten Behandlungstagen ein-
gereiht. Eine intensivmedizinische Versorgung und akuttherapeutische Maßnahmen stehen im Vor-
dergrund, weshalb die Infrastruktur eines Akutkrankenhauses erforderlich ist.

PatientInnen in der Frühneurorehabilitation (Phase B) sind bewusstseinsgestört und die Kooperati-


onsfähigkeit ist nicht oder nur in einem geringen Ausmaß gegeben. Bezogen auf die Aktivitäten des
täglichen Lebens (ADLs, activitys of daily living) ist der/die Betroffene vollkommen oder weitgehend
unselbständig. Diese Aktivitäten inkludieren beispielsweise sich bewegen, sich selbst waschen, essen
und trinken oder kommunizieren. In dieser Phase sind Verschlechterungen des Zustandes oder Kom-
plikationen häufig, aber der/die PatientIn ist nicht (mehr) dauernd beatmungspflichtig.
In der Weiterführenden Rehabilitation (Phase C) ist der/die Patientin wach, bewusstseinsklar und
zumindest teilorientiert. Er/sie ist in der Lage, über den Tag verteilt zumindest an 4 Therapieeinheiten
teilzunehmen. Er/sie ist zwar noch zu einem meist erheblichen Teil von pflegerischer Hilfe abhängig,
kann aber zumindest einige Aktivitäten des täglichen Lebens (ADLs) alleine bewältigen.
Die Langzeitrehabilitation (Phase D) ist geeignet für Betroffene, die die Aktivitäten des täglichen Le-
bens (ADLs) vollkommen oder weitgehend selbstständig mit Hilfsmittel durchführen können. Weiters
sind sie in der Lage, mehrere Stunden am Tag an einem Therapieprogramm teilzunehmen und zei-
gen sich kooperativ, eine gewisse Zeit ohne professioneller Aufsicht zu verbringen.
Eine Teilstationäre bzw. Ambulante Reha (Phase E) ist induziert, wenn PatientInnen ihren Alltag und
ihre Freizeit auch über mehrere Tage alleine planen, organisieren und verbringen können. Schwer-
punktmäßig werden spezielle neurorehabilitative Maßnahmen zum Ausbau, zur Stabilisierung und
Erhaltung bereits erreichter Therapieerfolge durchgeführt. Darunter zählen auch die berufliche Wie-
dereingliederung und der Erwerb umfassender sozialer Kompetenzen.

29
Literatur:
Cicerone , K. et al. (2011). Evidence-Based Cognitive Rehabilitation: Updated Review of the Literature From 2003
Through 2008. Archives of physical medicine and rehabilitation, 92, 519 – 530
Gauggel, S. (2003). Grundlagen und Empirie der neuropsychologischen Therapie: Neuropsychotherapie oder Hirnjog-
ging?. Zeitschrift für Neuropsychologie, 14, 217 – 246
Goldenberg, G. (2007) Neuropsychologie. München: Elsevier
Hartje, P. & Pöck, K. (2006) Klinische Neuropsychologie. Stuttgart: Thieme
Harvey, P. D. (2012). Clinical applications of neuropsychological assessment. Dialogues in clinical neuroscience, 14, 91
– 99
Lehrner, J., Pusswald, G., Fertl, E., Strubreither, W., Kryspin-Exner, I. (Hrsg.). (2011). Klinische Neuropsychologie.
Grundlagen – Diagnostik – Rehabilitation. Wien: Springer

Prigatano, G. P. (2013). Challenges and opportunities facing holistic approaches to neuropsychological rehabilitation.
NeuroRehabilitation. 32, 751 – 759
Sturm, W., Hermann, M., Wallesch C.-W. (Hrsg.). (2000). Lehrbuch der klinischen Neuropsychologie. Lisse: Swets &
Zeitlinger Publishers
ICF, International Classification of Functioning, Disability and Health, WHO, 2005.

30
Psychotraumatologie
Mag.a Gerda Greinz u. Mag.a Dorothea Wuchse

Psychotraumatologie ist die Lehre, die sich mit seelischen Traumafolgen und mit der Erforschung
und Behandlung der Auswirkungen von traumatischem Stress auf das Erleben und Verhalten von
Individuen und Systemen, beschäftigt. Psychotraumatologie setzt sich mit den Vorbedingungen, dem
Situationsgeschehen, dessen Folgen und den therapeutischen Hilfen für traumatisierte Menschen
auseinander (Fischer und Riedesser, 1998).
Das Wort „Trauma“ stammt aus dem Griechischen und bedeutet Verletzung und wird sowohl im
medizinischen Bereich (z.B. für die Auswirkungen eines Schlages) aber auch im Zusammenhang mit
unserer Psyche verwendet. Ein psychisches Trauma, „Psychotrauma“ entsteht durch eine, mehrere
oder lang andauernde psychische Belastung(en) und ist eine seelische Verletzung.
Trauma - Begriffsklärung
Ein Trauma ist
• ein vitales Diskrepanzerlebnis zwischen (lebens)bedrohlichen Situa-
tionsfaktoren und den individuellen Bewältigungsmöglichkeiten,
• das mit Gefühlen von Hilflosigkeit und schutzloser Preisgabe einher-
geht
• und so eine dauerhafte Erschütterung von Selbst- und Weltverständ-
nis bewirkt (Fischer/ Riedesser 2009, S. 84).
Das bedeutet, dass ein Trauma aus einem Ereignis im Leben eines Menschen unter folgenden Fak-
toren entstehen kann:
• das Ereignis wird vom individuellen Organismus als lebensbedrohlich bewertet – für das
eigene Leben oder das Leben einer anderen Person und/oder
• das Ereignis ist mit überwältigenden Gefühlen von Ohnmacht, Hilflosigkeit und Angst ver-
bunden und/oder für die Verarbeitung des Ereignisses sind keine ausreichenden Ressourcen
(Sicherheit, Geborgenheit, andere Personen, Geld, Nahrung etc.) vorhanden

Definition nach ICD-10:


„.. ist definiert als ein kurz oder langanhaltendes Ereignis oder Geschehen von außergewöhnlicher
Bedrohung mit katastrophalem Ausmaß, das nahezu bei jedem eine tiefgreifende Verzweiflung aus-
lösen würde“

Definition nach DSM-IV: unterscheidet zwischen A1 und A2-Kriterium


A1: (Objektives Erleben): „… ein Ereignis oder besteht aus mehreren Ereignissen, die eine Konfron-
tation mit tatsächlichem oder drohendem Tod oder ernsthafter Verletzung oder Gefahr für die eigene
oder fremde körperliche Unversehrtheit beinhalten“
A2: (Subjektives Erleben): „… die Reaktion der Person umfasst intensive Furcht, Hilflosigkeit oder
Entsetzen.“

Erschütterung von Selbst- und Weltverständnis (Janoff-Bulman, 2002)


1. Erwartung eines grundlegenden Wohlwollens der Welt und der Menschen (benevolence) –
Grundvertrauen ist verloren
2. Sinn- und Bedeutungshaftigkeit der Welt umfasst Annahmen über die Kontrollierbarkeit
(Strukturprinzip) und Gerechtigkeit (Verteilungsprinzip) (meaningfulness) – „es wirkt alles sinn-
los“
3. Konzepte über das Selbst wie den Selbstwert, die Investition der Person in „richtiges“ Verhal-
ten“ (eigene Moralität) und Annahmen zum Glück versus Unglück – „ich bin nichts wert“

31
Damit verbunden sind
• die Erschütterung der Vorstellung der persönlichen Unverwundbarkeit
• die Erschütterung der positiven Selbstsicht (z.B. eine „gewisse“ Kontrolle über den Lauf der
Dinge zu haben)
• die Zerstörung unserer Überzeugungen
• der Zusammenbruch der Vorstellung einer geordneten sinnhaften gerechten Welt (Lemke,
2008)
• der Verlust von Hoffnung
• der Verlust des Sinngefühls
• der Verlust des Gefühls von Eingebundenheit

Die Bedeutung der Traumaerfahrung:


> Was ist das „Traumatische“ am Trauma?

eine traumatische Situation ist eine


• überwältigende,
• lebensbedrohliche,
• furchtbare und ängstigende Erfahrung,
die außerhalb des „normalen“ menschlichen Erfahrungsbereichs liegt und
• mit enormen seelischen und/oder körperlichen Schmerzen
• sowie dem Gefühl der Ohnmacht, des Ausgeliefertseins
• und des Kontrollverlustes
verbunden ist.

Dies geschieht häufig durch Unfälle, Verkehrs- oder Naturkatastrophen, Gewalterlebnisse, durch
plötzlichen Verlust vertrauter Menschen, lebensbedrohlichen Erkrankungen und Erfahrungen von Ty-
rannei, Folter oder Vertreibung. Traumatische Erlebnisse werden in unserem Gedächtnis nicht als
zusammenhängende und vergangene Geschichte abgespeichert, sondern es werden nur Bruchstü-
cke bzw. Fragmente des Erlebens unvollständig und in unterschiedlichen Wahrnehmungsqualitäten
abgespeichert – zum Beispiel als einzelne Bilder, Gerüche, Geräusche, Wörter, Satzteile etc. Das
traumatische Erlebnis wird somit unvollständig ins Gedächtnis integriert, womit gemeint ist, dass die
zeitliche und räumliche Einordnung des Ereignisses fehlt und dieses Ereignis als „vergangenes Ereig-
nis“ abgespeichert werden konnte. Es erfolgt eine unzusammenhängende Speicherung des Ereignis-
ses auf psychischer und somatischer Ebene. Die betroffenen Personen können sich meist nur bruch-
stückhaft oder gar nicht an das traumatische Erlebnis erinnern. Die Erinnerungen an das Ereignis
überschwemmen die betroffenen Personen völlig unvorbereitet und derart massiv, dass diese das
Gefühl haben, wieder mitten in dieser traumatischen Situation zu sein, also diese noch einmal zu
erleben, obwohl objektiv keine Gefahrensituation besteht. Dieses „plötzliche und nicht steuerbare
Überschwemmtwerden“ von der Erinnerung an die traumatische Situation wird als „Flashback“ be-
zeichnet und durch sog. „Trigger“ (Reize), wie zum Beispiel Gerüche, Bilder, Geräusche etc. ausge-
löst. Die betroffenen Personen erleben sich in diesen Flashbacks hilflos und ohnmächtig. Flashbacks
treten in der Regel Wochen bzw. Monate nach dem traumatischen Erlebnis auf, werden jedoch in
der Regel mit dem Abstand zum Ereignis weniger. Je nach Schwere der Traumatisierung können
Flashbacks jedoch auch Jahre oder ein Leben lang auftreten bzw. auch erst Jahre nach einer Trau-
matisierung durch einen Trigger ausgelöst werden.

Formen von Traumata:

Trauma Typ I: findet einmalig und kurzfristig statt, meist besteht akute Lebensgefahr; das Ereignis
kommt plötzlich und überraschend

32
Trauma Typ II: Mehrfachtraumatisierungen –mehrere traumatische Einzelereignisse und/oder häus-
liche Gewalt oder Folter. Von Entwicklungstraumata spricht man, wenn eine Person als Kind lang-
andauernden, traumatisierenden Situationen ausgesetzt gewesen ist - hier unterscheidet man zwi-
schen Vernachlässigung und Missbrauch.

Sequentielle Traumatisierung (Keilson, 1979; Becker, 2006, S.190): ist definiert als eine Aufeinan-
derfolge von unterschiedlichen traumatischen Sequenzen. Damit ist gemeint, dass das Trauma mit
dem Ereignis nicht zu Ende ist, sondern dass das Trauma als Prozess auch die Erfahrungen „davor“
und „danach“ mitberücksichtigt:
• prä-traumatische Sequenz
• traumatische Sequenz
• post-traumatische Sequenz

Dadurch erhält die zeitliche Dimension des Traumatisierungsprozesses eine neue Bedeutung. Beson-
dere Aktualität erfährt das Modell der sequentiellen Traumatisierung im Kontext der Flucht, in dem
Keilsons Theorie verdeutlicht, dass das Trauma nicht auf eine Situation bezogen werden kann, son-
dern einen Prozess darstellt, der in mehreren Sequenzen abläuft und sich im Aufnahmeland fortsetzt
(Hargasser, 2014 und Becker, 2006).

Sekundäre oder indirekte Traumatisierung: darunter versteht man die „Ansteckung“ mit typischen
posttraumatischen Symptomen im Verlauf der Arbeit mit traumatisierten Menschen. Es handelt sich
dabei um eine übertragene Traumatisierung, die zustande kommt, obwohl der/die HelferIn nicht
selbst mit dem traumatischen Ereignis konfrontiert ist. (Daniels J., 2006)

Sekundäre oder indirekte Traumatisierung kann entstehen durch

• Begleitung
• Begegnung
• Konfrontation
• das Mitbekommen – „Zeug/in“ sein von direkter, unmittelbarer Traumatisierung

Daniels (2003) beschreibt die ST (Sekundäre Traumatisierung) als eine „Ansteckung“ mit den typi-
schen posttraumatischen Symptomen der KlientInnen und definiert sie als eine Traumatisierung, die
ohne direkte sensorische Eindrücke des Ausgangstraumas sowie mit zeitlicher Distanz zum Aus-
gangstrauma entsteht. Lemke (2010) vertritt die Ansicht, dass Personen, die traumatisierten Men-
schen helfen, durch diese Begegnung und den Kontakt zu diesen Menschen, durch die Auseinan-
dersetzung mit deren Erleben und der Behandlung von deren Leid, auch traumatisiert werden kön-
nen, ohne dass diese Personen dem Trauma selbst unmittelbar ausgesetzt gewesen sein müssen.
Nach Lemke kann damit auch bei Helfern die Erkrankung der Sekundären Traumatischen Belas-
tungsstörung (STBS) auslöst werden.

Folgen von Traumatisierungen

Häufig auftretende Reaktionen auf „Extrem-Stress“

• Nervosität, Angst, andauerndes Auf-der-Hut sein – „Überwachsamkeit“, Schreckhaftigkeit


• Konzentrationsschwierigkeiten, Erinnerungslücken
• erneutes Durchleben des Ereignisses
• unkontrollierte Bilder und Gedanken
• Isolations- und Taubheitsgefühle
• psychische oder physische Reaktionen, wenn man an das Ereignis denkt
• Dissoziationen – von sich selbst abgeschnitten sein – körperlich und emotional
• Schlafstörungen

33
• Traurigkeit, emotionale Gefühllosigkeit
• sozialer Rückzug, Resignation – Schwierigkeiten im Kontakt mit anderen Menschen
• Vermeiden von Orten, Menschen, Gedanken, die an das Ereignis erinnern
• Reizbarkeit
• Schuldgefühle, Scham

Auswirkungen früher Stresserfahrungen

Frühe Stresserfahrungen sind bleibende „biologische Narben“, die die Anpassungsfähigkeit des Or-
ganismus über die gesamte Lebensspanne hinweg beeinträchtigen und daher klare Risikofaktoren
für das Auftreten schwerer psychischer und körperlicher Erkrankungen im Erwachsenenalter.
Die ACE (Adverse Childhood Experiences) – Studie, Felliti et al. 1998 zeigt eine lineare Abhängigkeit
zwischen dem Ausmaß frühkindlicher Traumatisierung und dem Risiko, körperlich (KHK, Krebs, chro-
nische Lungenerkrankungen) und psychisch (Alkohol/Drogen, Depression, kPTSD) schwer zu erkran-
ken.

Intrusionen - Flashbacks, Albträume: ausgelöst durch Schlüsselreize (Trigger)


• Wiedererleben der traumatischen Situation
• hohe emotionale Beteiligung, Überschwemmung starker Gefühle (Wut, Angst, Ohnmacht,
Hilflosigkeit)
• Erschütterung des „Ich-Verständnisses“
• Innere Unruhe, Schweißausbrüche, Panikattacken
• Folgebeschwerden von Intrusionen: Reizbarkeit, Müdigkeit, Unwohlsein

Dissoziationen
• Alltagsdissoziation – Vergesslichkeit, Zerstreutheit
• Amnesie – keine Erinnerung an das traumatische Ereignis
• Derealisation – sich fremd in der Welt fühlen – „wie in Watte gepackt“, kein Zugang zu
bestimmten Sinnesreizen, anderes Raum-Zeit-Gefühl
• Depersonalisation – „neben sich stehen“ – sich nicht mehr mit sich selbst identifizieren kön-
nen – um
• häufige Dissoziation: kein Zugang zu den Empfindungen des eigenen Körpers
• Affektarmut – wenig oder kein Zugang zu eigenen Gefühlen und Benennung von Gefühlen
nicht möglich (Alexithymie)
• Fugue (franz: Flucht): „Filmriss“ – nicht mehr wissen, wie man an einem Ort gekommen ist
• starker Präindikator für Auftreten einer späteren PTBS (Posttraumatischen Belastungsstörung)
• Dissoziationen führen zu Schutz vor Schmerzen (Analgesie) und Angst- oder Panikgefühlen
(Gefühlstaubheit)
• dissoziative Störungen

Häufigste Diagnosen nach psychischen Traumata

ICD 10 (World Health Organisation, 2011)


• F43.0 akute Belastungsreaktion
• F43.1 posttraumatische Belastungsreaktion
• F43.2 Anpassungsstörungen
• F43.8 sonstige Reaktionen auf schwere Belastungen
• F43.9 Reaktion auf schwere Belastung, nicht näher bezeichnet

34
DSM V (American Psychiatric Association, 2013)
• Trauma- and Stressor-Related Disorders (> kPTSD)
• Dissociative Disorders
• Somatic Symptom Disorders

F43.0 akute Belastungsreaktion


Symptome:
• „Betäubung“, Bewußtseinseinengung
• eingeschränkte Aufmerksamkeit - Unfähigkeit, Reize zu verarbeiten
• Desorientiertheit
• sich zurückziehen aus der Situation oder Unruhezustand und Überaktivität
• vegetative Symptome panischer Angst: Tachykardie, Schwitzen, Erröten, u.a.

F43.1 Posttraumatische Belastungsreaktion


Leitsymptome:
• Wiedererleben (Intrusion) des Traumas in sich aufdrängenden Erinnerungen oder in Träumen
• Vermeidung (Avoidance) von Aktivitäten oder Situationen, die Erinnerungen an das Trauma
wachrufen könnten
• Übererregtheit (Hyperarousal) – übermäßige Schreckhaftigkeit, Schlaflosigkeit, Vigilanzstei-
gerung

Im DSM-IV wurde die PTBS (Posttraumatische Belastungsstörung) noch den Angststörungen zugeord-
net, seit dem DSM-V findet sie sich unter den „trauma- and stressor-related disorders“, im ICD-10
den „Reaktionen auf schwere Belastungen und Anpassungsstörungen“.

Psychische Erste Hilfe in oder unmittelbar bzw. zeitnah nach traumatischen Situationen
• Kontakt herstellen
• Sicherheit gewährleisten
• Ruhe und Privatsphäre bieten
• Zuhören
• Beruhigen und Gefühle normalisieren
• Gefühle priorisieren helfen
• Verbindung mit nahestehenden Personen herstellen
• Informieren
• praktische Information

Die Betroffenen über ihre körperlichen/psychischen Reaktionen aufklären und informieren - Psycho-
edukation
• Normalisierung: „Ihre Reaktion ist eine normale Reaktion in einer außergewöhnlichen Situ-
ation“
• Notfallreaktionen/Symptome erklären: „Was ihr Gehirn hier macht ist sehr hilfreich,…“
• Pathologisierung vermeiden!
• individuelle Reaktionen akzeptieren
 dadurch erfahren die Betroffenen Entlastung, Stärkung, Wiederermächtigung sowie die Reduk-
tion von Ohnmacht und Hilflosigkeit

Keinesfalls sollte man Personen unterbrechen, oder dazu drängen, die Geschichte zu erzählen. Die
Betroffenen sollen nicht dazu aufgefordert werden, Einzelheiten zu erzählen und es sollte seitens der
Helferseite keine Meinung zu der Situation geäußert werden.
Lehnen Betroffene die Hilfe ab, sollen dieses darüber informiert werden, dass Hilfe auch zu einem
späteren Zeitpunkt möglich sei.

35
Literatur:
Daniels, J. (2003). Sekundäre Traumatisierung - kritische Prüfung eines Konstruktes anhand einer explorativen Studie.
Unpublished manuscript, Universität Bielefeld.
Figley, C. R. E. (1995). Compassion Fatigue: coping with secondary traumatic stress disorder in those who treat the trau-
matized. New York: Brunner/Mazel.
Figley, C. R. E. (2002). Treating compassion fatigue. New York: Brunner-Routledge.
Fischer, G. & Riedesser, P. (2009): Lehrbuch der Psychotraumatologie. München.
Becker, D. (2006): Die Erfindung des Traumas. Verflochtene Geschichten. Berlin.
Hargasser, B. (2014): Unbegleitete minderjährige Flüchtlinge. Sequentielle Traumatisierungsprozesse und die Aufgaben
der Jugendhilfe. Frankfurt/M.
Keilson, H. (1979): Sequentielle Traumatisierung bei Kindern. Stuttgart.
Lemke, J. (2006): Sekundäre Traumatisierung. Klärung von Begriffen und Konzepten der Mittraumatisierung. Kröning.
Pearlman, L. A., & Saakvitne, K. W. (1995). Trauma and the therapist: countertransference and vicarious traumatization
in psychotherapy with incest survivors. London: W. W. Norton.
Porges, S.W. (2017): Die Polyvagaltheorie und die Suche nach Sicherheit. Traumabehandlung, soziales Engagement und
Bindung. Lichtenau.
Reddemann, L. (2011): Psychodynamisch Imaginative Traumatherapie. Ein resilienzorientierter Ansatz in der Psychotrau-
matologie. Stuttgart.
Salston, M., & Figley, C. R. (2003). Secondary traumatic stress effects of working with survivors of criminal victimization.
Journal of Traumatic Stress, 16(2), 167-174.
Shapiro, F. (1998a): EMDR – Grundlagen und Praxis: Handbuch zur Behandlung traumatisierter Menschen. Paderborn.
Watkins, J.G. & Watkins, H. (2003): Ego-States. Theorie und Therapie. Heidelberg.

36
Klinische Psychologie im Universitätsklinikum für Kinder- und Jugend-
heilkunde Salzburg
Maga. Esther Wagenhofer

Im Feld der Kinder und Jugendlichenpsychotherapie ist ein methodenintegrierendes oder –über-
greifendes Vorgehen besonders wichtig. Bei der Kinder- und Jugendlichenpsychotherapie handelt
es sich um ein hochkomplexes Geschehen, da sich Kinder (und zum Teil auch Jugendliche) weni-
ger direkt über Worte, sondern vor allem über Spielhandlungen ausdrücken (Reinelt et.al, 1997).
Das Spiel selbst hat den Charakter prozessualer Aktivierung. Im Spiel können Kinder und auch Ju-
gendliche ihre Lebensthemen und Ressourcen zeigen, aktivieren und weiterentwickeln. Es geht da-
rum, die im Spiel ausgedrückten Inhalte und Themen zu verstehen und auf die Lebensthemen des
Kindes zu beziehen und dann auf der Basis einer reflektierten Beziehungsgestaltung verbal oder in
der Spielhandlung zu beantworten.
Psychotherapie mit Kindern und Jugendlichen bedeutet immer auch Zusammenarbeit mit den Be-
zugspersonen.

Wer braucht Klinische Psychologie im Kinderspital?

Kinder = PatientInnen Eltern/Angehörige/Bezugsper- Team


son
•Probleme im Umgang mit •Probleme im Umgang mit •Entlastung des Teams bei
der Erkrankung der Erkrankung Familien mit einer schwierigen
•Psychische Auffälligkeiten der •Psychische Auffälligkeiten der Situation (Krankheit, innerfa-
Kinder Eltern miliär, ...)
•Familiäre Konflikte •Familiäre Konflikte •Intervision (Austausch) über
•Krisensituationen •Krisensituationen besonders belastende Fallge-
•Mitteilung der Diagnose •Mitteilung der Diagnose schichten
•Sterbebegleitung •Vernachlässigung, Miss-
•Vernachlässigung, Miss- handlung, Missbrauch > Kin-
handlung, Missbrauch > Kin- derschutzgruppe
derschutzgruppe

Klinisch psychologische Arbeit mit Kindern und Jugendlichen im Kinderspital


• Diagnostik
• Verhaltensbeobachtung
• Interaktion
• Spieltherapie
• Gespräch
• Beziehung
• Konkrete Planung von Veränderungsschritten
• Neues Verhalten üben
• Helfer einladen

Gelingender Kontakt
• Erstkontakt 
• Faktor Zeit
• Achtsamkeit
• Verlangsamung
• Freundlichkeit
• Humor
• Flexibilität
• Information

37
• Psychoedukation: z.B. von Gefühlen (Benennung, Bedeutung, Nutzen, Auslöser) sonst Verunsi-
cherung und Kontrollverlust (276 positive und liebenswerte Eigenschaften, ein Dino zeigt Gefühle,
…)
• Atmosphäre
• Fehlerfreundlichkeit
• Krankenhaus als sicherer Ort

Eltern in Ausnahmesituationen
• Vorerfahrungen der Eltern mit Krankenhaussituationen > Ängste
• Viele verschiedene Menschen > Orientierung
• Privatsphäre im Krankenzimmer
• Dolmetschgespräche

Systemische Fragetechniken/Zirkuläres Fragen


• Fragen nach Unterschieden: „Ist das Problem in jeder Situation gleich?“
• Fragen nach Ressourcen: „Was soll in deinem Leben so bleiben, wie es ist?“, „Was ist gut daran?“
• Wunderfrage: „Angenommen heute Nacht kommt eine Fee und zaubert dein Problem weg, was
ist dann morgen anders?“, „Wer wird es als erstes bemerken, dass das Wunder über Nacht ge-
schehen ist, und woran?“, „Was werden die Menschen um dich herum anders machen?“
• Verbesserungsfragen: „Wie oft (wie lange, wo, wann) ist das Problem nicht aufgetreten?“
• Verschlimmerungsfragen: „Was kannst du tun – angenommen, du würdest es dir vornehmen –
um dein Problem absichtlich zu verschlimmern, zu verewigen, zu behalten?“ (ACHTUNG! Nur
humorvoll mit Patienten besprechen, die gerade gutgelaunt sind!)
• Prozentfragen
• Klassifikationsfragen: „Wer würde als erster…, wer zuletzt?!, „Wer freut sich am meisten..., wer
am wenigsten…?“
• Übereinstimmungsfragen: „Wer stimmt mit wem überein/nicht überein?“

Reframing
Reframing zählt zu den sanftesten und zugleich effektivsten Interventionen. Es wird von der Annahme
ausgegangen, dass alle menschlichen Verhaltensweisen ihre Bedeutung erst durch den Rahmen, den
Kontext erhalten, innerhalb dessen sie wahrgenommen, erlebt oder beurteilt werden. Wird der Rah-
men durch einen anderen ersetzt, ändert sich die Gesamtbedeutung der Handlung, obwohl diese
an und für sich gleichbleibt.

Möglichkeiten des Reframings (Weeks, 1977)


Negative Bewertung Reframing
Umherirren Alle vorhandenen Möglichkeiten ausschöpfen
Kontrollieren wollen Struktur und Überblick in seine Umwelt bringen
Gefühllos sein Sich vor Verletzungen schützen
Unterwürfig sein Autorität und Führung suchen, um sich selbst
zu finden
Auf Distanz gehen Sich um sich selbst kümmern
Abgeschieden leben und kontaktarm sein Sein eigenes Bewusstsein genau erforschen
Überempfindlich sein Sehr lebendig, bewusst und intensiv auf andere
Menschen und die Umwelt eingestellt sein
Passiv sein Die Fähigkeit, Dinge zu akzeptieren, wie sie
sind

38
Methoden mit Kindern/Kreative Zugänge
Der Einsatz analoger Methoden ist in der Arbeit mit Kindern und auch mit Jugendlichen hilfreich, da
bei Kindern die sprachliche Entwicklung noch nicht abgeschlossen ist und sie nicht in der Lage sind,
im selben Ausmaß, wie Erwachsene, abstrakt zu denken (Reinelt et.al, 1997).
Zu viele Fragen bringen Kinder in eine Überforderungssituation, aus der sie versuchen auszusteigen,
in dem sie „Weiß nicht!“ antworten oder ganz spontan zu „spielen“ beginnen > perfekter Zeitpunkt
für Psychologinnen einzusteigen und mitzumachen.

Analoge Methoden
Handpuppen Zeichnungen Geschichten und Märchen
Rollenspiele Verzauberte Familie Auswählen lassen
Interviews, wie z.B. Steckbrief Baum oder Bäume Lesen/Vorlesen
Begleiter nach Hause Inneres Haus Erzählen
Sicherer Ort Spielen
Szenen oder Comics Erfinden
Kritzeleien
Name
Gemeinsames Zeichnen
Symbole Genogramme Entspannungstechniken
Timeline Zeichnen Angenehmer Ort
Knöpfe Knöpfe Fantasiereisen
Bänder
Steine

Wirkfaktoren der Psychotherapie nach Klaus Grawe (2005)

• Therapeutische Beziehung: Die Qualität der Beziehung zwischen dem Psychotherapeuten


und dem Patienten trägt signifikant zum Therapieergebnis bei.
• Ressourcenaktivierung: Die Besonderheiten und Eigenarten, die die PatientInnen in die The-
rapie mitbringen, werden als positive Ressource für das therapeutische Vorgehen genutzt.
Das betrifft vorhandene Motivation, Fähigkeiten und Interessen der PatientInnen.
• Problemaktualisierung: Die Probleme, die in der Therapie verändert werden sollen, werden
unmittelbar erfahrbar. Das kann z.B. dadurch geschehen, dass TherapeutInnen und Patien-
tInnen reale Situationen aufsuchen, in denen die Probleme auftreten, oder dass sie durch
besondere therapeutische Techniken wie intensives Erzählen, Imaginationsübungen, Rollen-
spiele o.ä. die Probleme aktualisieren.
• Motivationale Klärung: Die Therapie fördert mit geeigneten Maßnahmen, dass PatientInnen
ein klareres Bewusstsein der Determinanten (Ursprünge, Hintergründe, aufrechterhaltende
Faktoren) ihres problematischen Erlebens und Verhaltens gewinnen.
• Problembewältigung: Die Behandlung unterstützt PatientInnen mit bewährten problemspezi-
fischen Maßnahmen (direkt oder indirekt) darin, positive Bewältigungserfahrungen im Um-
gang mit ihren Problemen zu machen.

Ziel der Klinischen Psychologie im Kinderspital


Ziel ist die multiprofessionelle, ganzheitliche und familienorientierte Versorgung von Kindern, Ju-
gendlichen und ihren Familien.

39
Literatur
Grönemeyer, D. (2007). Der kleine Medicus. Reinbek bei Hamburg: Rowohlt Taschenbuch Verlag.
Hüther, G. & Michels, I. ((2009). Gehirnforschung für Kinder. München: Kösel Verlag.
Kleinstäuber, M. et.al (2018). Therapie-Tools. Somatoforme Störungen. Weinheim: Beltz
Lutz, W. & Grawe, K. (2005). Psychotherapieforschung. In F. Petermann & H. Reinecker (Hrsg.), Handbuch der Klini-
schen Psychologie und Psychotherapie (S. 92-100). Göttingen: Hogrefe.
Petermann, F. (2017). Therapie-Tools. Eltern- und Familienarbeit. Weinheim: Beltz.
Reinelt, T. et.al (1997). Lehrbuch der Kinderpsychotherapie. München: Ernst Reinhardt, Verlag.
Siegel, D. (2015). Aufruhr im Kopf. München: mvg Verlag.
Weeks, G. Toward a dialectical approach to intervention. Human Development, 20, 277-292, 1977.

40
Biofeedback
Dr. Andreas K. Kaiser, MSc

Bei der Biofeedback-Behandlung werden den PatientInnen körperliche Funktionen optisch oder akus-
tisch zurückgemeldet. Positive Änderungen dieser Körperfunktionen werden dadurch verstärkt, so
dass die PatientInnen lernen, diese physiologischen Prozesse selbst zu beeinflussen (Rief, Birbaumer
2000).
Das Hauptziel der Biofeedback-Behandlung ist die Entwicklung von Selbstkontrolle über körperliche
Vorgänge. Biofeedback baut also auf der Fähigkeit des Menschen auf, durch Lernprozesse körperli-
che Funktionen zu verändern. Bis in die 60er Jahre des 20. Jahrhunderts wurde angenommen, dass
das „autonome“ Nervensystem nicht willentlich oder durch Lernprozesse beeinflussbar ist. Die
Biofeedback-Therapie ist ein Verfahren, das Lernprozesse ermöglicht und verbessert. Mittels einer
spezifischen Apparatur, dem Biofeedbackgerät, lernen PatientInnen die willentliche Kontrolle über
diese, dem Bewusstsein sonst wenig zugänglichen Funktionen, besser und präziser wahrzunehmen.
Über akustische Signale (Variation der Tonhöhe oder Lautstärke) oder optische Signale (Linien oder
Balken auf dem Bildschirm, Veränderung der Helligkeit oder des Farbspektrums) erkennt der/die
PatientIn kleinste Veränderungen des abgeleiteten physiologischen Prozesses.
Durch Biofeedback beeinflussbare Körperfunktionen sind u. a. Muskelaktivität, Herzrate, Blutdruck,
Schweißdrüsenaktivität als allgemeines Maß für autonome Erregung, Haut- und Körpertemperatur,
elektrophysiologische Prozesse des Gehirns, periphere Änderung der Durchblutung durch Verände-
rung des Durchmessers der Blutgefäße sowie Änderung des Atemrhythmus.
In der Therapie sollen PatientInnen lernen, kognitive Prozesse, Emotionen und Körpersignale zu ei-
nem Gesamtbild zu integrieren und zu verändern. Die Behandlung ermöglicht den Betroffenen
psychophysiologische Zusammenhänge besser zu verstehen.
Zu den grundlegendsten Wirkmechanismen des Biofeedbackverfahrens zählen das Erlernen der Kon-
trolle über Körperfunktionen, die Verbesserung der Wahrnehmung körpereigener Prozesse (body a-
wareness), das Erlernen der allgemeinen Entspannungsreaktion sowie Veränderungen von kognitiven
Überzeugungen.
Biofeedback bezieht sich nicht nur auf eine spezifische Interventionsform, einen Körperbereich, eine
Modalität oder auf einen bestimmten Ablauf. In den Biofeedback-Sitzungen werden verschiedenste
therapeutische Interventionen eingesetzt wie u. a. verbale Instruktionen, Aufmerksamkeitsfokussie-
rung, Entspannungsverfahren, Belastungstests und Visualisierungsübungen.
Die Anwendungsbereiche von Biofeedback sind sehr vielfältig. So wird Biofeedback eingesetzt in der
Schmerzbehandlung (Spannungskopfschmerz, Migräne, Chronische Rückenschmerzen), bei psychi-
schen Störungen (Angststörungen, Reaktionen auf schwere Belastungen, somatoforme Störungen,
nicht organische Schlafstörungen, Aufmerksamkeitsdefizitstörung und Hyperaktivität wie ADS,
ADHS), bei essentieller Hypertonie, funktionellen Herzbeschwerden, Morbus Raynaud, chronischen
Tinnitus und zur Atemschulung. Immer größere Bedeutung bekommt die Biofeedbackbehandlung
bei neuromuskulären und neurologischen Störungen (z. B. Tortikollis, Bruxismus, Lähmungen und
anderen neurologischen Erkrankungen, Harn- und Stuhlinkontinenz, epileptischen Anfällen) sowie in
der Sport- und Wettkampfvorbereitung zur Leistungssteigerung und im Entspannungs- und mentalen
Training.

41
ELC. Early Life Care. Eine Einführung
Maga. Silvia Désirée Pernter

Begriffsbestimmung
Early Life Care ist eine sehr junge, neu definierte Disziplin, welche sich mit dem ganz frühen Leben
befasst. Seit 2016 wird an der PMU (in Kooperation mit St. Virgil) der europaweit erste Universitäts-
lehrgang für Early Life Care angeboten. ELC beschäftigt sich mit verschiedensten Themen rund um
Schwangerschaft, Geburt und erstes bzw. erste drei Lebensjahre, wobei die involvierten Fachbereiche
auf ganzheitliche und multiprofessionelle Weise ineinandergreifen. ELC hat den Präventionsgedan-
ken sowie den Menschen vor seinem biopsychosozialen Hintergrund zum Thema.

Demensprechend rückt das Baby nicht erst bei seinen ersten postnatalen Auffälligkeiten und Krank-
heitsanzeichen in den Fokus, sondern ELC setzt schon vorher an. Das pränatal sich entwickelnde
Kind wird ebenso berücksichtigt wie der physische, soziale und emotionale Lebensraum, innerhalb
dessen es entsteht. Die neueren wissenschaftlichen Erkenntnisse aus dem Gebiet der Epigenetik lie-
fern wertvolle Hinweise für die Bedeutung dieser ersten wichtigen und prägenden Zeit.

Die Disziplinen Gynäkologie und Geburtshilfe dürfen sich nicht auf die Begleitung eines reibungslo-
sen Voranschreitens der Schwangerschaft, das intrauterine Wachstum und die körperliche Situation
des Ungeborenen beziehen, sondern müssen auch die psychosoziale Versorgung der werdenden
Familie im Blick haben, sowie natürlich die körperlichen und psychischen Bedingungen der werden-
den Mutter. Die Verläufe von Schwangerschaft und Geburt stellen heute nicht nur aus medizinischer,
sondern auch aus psychologischer Sicht einen wertvollen Indikator für die mögliche weitere Entwick-
lung des Kindes dar, zumal bestimmte Risiken und prädisponierende Faktoren (Prägung) auf die Prä-
und Perinatalzeit zurückgehen.

In der postnatalen Phase von Early Life Care treten insbesondere die Disziplinen Neonatologie und
Pädiatrie in den Vordergrund. Aus psychosozialer Sicht steht hier das Entwicklungsumfeld des Neu-
geborenen und des späteren Kleinkindes im Fokus. Es sollen möglichst optimale Entwicklungsbedin-
gungen im Spielfeld von Bindung und Autonomie für das heranwachsende Kind und die neu ent-
standene Familie gewährleistet werden. Man könnte sagen, ELC befasst sich nicht vordergründig mit
der Behandlung von Krankheiten, Störungen und Belastungen, sondern bemüht sich im klinischen
und außerklinischen Bereich um Gesundheitsförderung und Prävention.
In jenen Situationen, wo keine kurativen Behandlungen mehr greifen, wird das Augenmerk auf eine
gelingende palliative Betreuung gesetzt. Die pädiatrische Palliativpflege gehört in diesem Sinne ge-
nauso zu ELC, wie das Thema „palliative Geburt“. Kranke Neugeborene, Babys und Kleinkinder mit
lebensbegrenzenden Erkrankungen müssen betreut, gepflegt und begleitet werden, wobei dazu die
Betreuung und Begleitung von Eltern und sozialem Umfeld hinzukommen.

Salutogenese, Prävention und Early Life Care


„Gesundheit ist ein Zustand völligen psychischen, physischen und sozialen Wohlbefindens und nicht
nur das Freisein von Krankheit und Gebrechen. Sich des bestmöglichen Gesundheitszustandes zu
erfreuen ist ein Grundrecht jedes Menschen, ohne Unterschied der Rasse, der Religion, der politi-
schen Überzeugung, der wirtschaftlichen oder sozialen Stellung." (Gesundheitsdefinition der WHO
1948)

Mit dieser Definition löste die WHO das Konzept Gesundheit aus einer rein biomedizinischen Sicht-
weise und aus den engen Bezügen des professionellen Krankheitssystems.

42
Im Sinne von Salutogenese (salus =Heil, Gesundheit; genese = Entstehung), einem Konzept, das
der israelisch-amerikanische Medizinsoziologe Aaron Antonovsky in den 70er Jahren entwickelte,
verstehen wir Gesundheit als Prozess und nicht als Zustand. Es handelt sich demnach um keinen
einmal erreichten und dann unveränderlichen „Zustand", sondern um eine lebensgeschichtlich und
alltäglich immer wieder neu und aktiv herzustellende „Balance".
Definiert man Prävention als die Vermeidung des Eintretens von Erkrankungen oder als Vermeidung
der Ausbreitung von Erkrankungen, muss man in erster Linie bei Themen wie der Risikovermeidung,
der Förderung von Gesundheitsressourcen sowie bei der Motivation zu gesundem Verhalten anset-
zen.
Eine großangelegte epidemiologische Studie aus den USA (Adverse Childhood Experiences Study)
untersucht den Zusammenhang zwischen den 10 häufigsten Kategorien traumatischer oder Gewalt-
erfahrungen in der Kindheit mit dem aktuellen Gesundheitszustand von mittlerweile Erwachsenen.
Die daraus gewonnenen Erkenntnisse liefern wichtige Hinweise und tiefgreifende Implikationen für
die medizinische Praxis und für die ärztliche Primärversorgung.
Im Bereich von Early Life Care geht es demnach um die Förderung von physischer und psychischer
Gesundheit von Kindern, von ihren Eltern und von deren sozialem Umfeld, damit möglichst optimale
Entwicklungsbedingungen gewährleistet werden können.

Biopsychosoziales Modell und systemisch-relationale Perspektive


„The dominant model of disease today is biomedical, and it leaves no room within its framework for
the social, psychological and behavioral dimensions of illness. A biopsychosocial model is proposed
that provides a blueprint for research, a framework for teaching, and a design for action in the real
world of health care“. (Georg.L.Engel, 1977)

Early Life Care spielt sich stets vor dem Hintergrund des biopsychosozialen Modells ab, zeigt sich
doch insbesondere beim Kind eindrücklich, dass es ohne erwachsene Fürsorgepersonen, die es er-
nähren und sich um es kümmern, nicht lebensfähig ist.

Der Buchstabe C in ELC steht für „Care“ und wird im Sinne von Pflege, Fürsorge, Halten, Nähren,
aber auch von Beziehung verwendet. Demnach beinhaltet er immer einen interpersonellen Ansatz.
Die Einzigartigkeit eines jeden Menschen spiegelt sich im Zeitraum von Early Life nicht nur in seinem
einzigartigen biologischen, psychischen und sozialen System wider. Wir alle sind bis ins hohe Er-
wachsenenalter immer auch Teil von Beziehungssystemen wie Familie, Freundeskreis, Arbeitsumfeld
und Gesellschaft. Aus systemischer Sicht müssen die Wechselwirkungen zwischen den Systemen er-
kannt und berücksichtigt werden, damit deren Dynamik bewusst wird und man bei Schwierigkeiten
neue Lösungsmöglichkeiten entwickeln kann.

Um sich ein Leben lang im Sinne der oben zitierten WHO-Gesundheitsdefinition über die gesamte
Lebensspanne hinweg gesund zu entwickeln, bedarf es anderer Menschen. Zu den lebenslang wich-
tigsten menschlichen Grundbedürfnissen werden aus psychologischer Sicht die folgenden gezählt:
das Bedürfnis nach Anerkennung und Wertschätzung (gehört, gesehen, verstanden werden), nach
Zugehörigkeit, nach Sicherheit, nach Autonomie und Entwicklung (Selbstwirksamkeit). Nicht nur das
afrikanische Sprichwort behauptet also „Um ein Kind zu erziehen, braucht es ein ganzes Dorf“, son-
dern auch laut modernen neurowissenschaftlichen Erkenntnissen ist das menschliche Wesen in seiner
neurobiologischen Entwicklung von einem sozialen Gegenüber abhängig. Wir sind quasi die inner-
halb von Beziehung entstandene „Summe unserer synaptischen Verbindungen“. Man spricht in die-
sem Zusammenhang auch von einem „relationalen Gehirn“.

Forschungsergebnisse aus dem Bereich der Neurobiologie ermöglichen es uns, eine Brücke zwischen
biologischen Vorgängen des Körpers und dem sozial-psychischen Erleben zu schlagen.

43
Der deutsche Biologe und Hirnforscher Gerhard Roth schreibt bezüglich der Entwicklung von Psyche
und Gehirn des Kindes, dass psychische Zustände und Funktionen aufs Engste mit den Tätigkeiten
der Gehirnzentren verbunden sind: insbesondere des limbischen Systems und dessen Interaktion mit
den kognitiven Zentren des Gehirns. Psyche und Gehirne entwickeln sich in strenger Parallelität zur
Entwicklung dieser Zentren. Dabei greifen genetisch-epigenetische, frühkindlich-psychische und spä-
tere psychosoziale Faktoren eng ineinander.

Bindungsorientierung
Eng an den systemisch-relationalen Ansatz und die zugrundeliegende biopsychosoziale Sichtweise
anknüpfend, nimmt das Konzept der Bindung einen wichtigen Stellenwert im Bereich von Early Life
Care ein.
Bindungsförderung wird bereits in der Pränatalzeit angestrebt. Im Zeitraum von der Zeugung über
die Schwangerschaft und Geburt bis hin zu der frühen postnatalen Zeit wird heute - nicht nur aus
psychologischer Sicht – der Aufbau von elterlichem Bonding und dann von Bindung des Kindes an
seine Eltern als ein wichtiges Ziel angestrebt, welches als ein positiver Prädiktor für eine bestmögliche
Entwicklung des Kindes und der familiären Gesundheit herangezogen werden kann.
„Bindungsförderung braucht einen Körper“, betont der deutsche Psychologe Thomas Harms. In sei-
ner über 25-jährigen Arbeit im Bereich der bindungsbasierten Körperpsychotherapie mit Säuglingen,
Kindern und Erwachsenen kommt er zum Schluss, dass die erste Beziehung zwischen dem Säugling
und seinen Eltern ohne Körperberührung nicht denkbar sei und präventive Ansätze (z.B. Basic Bon-
ding) in dieser Zeit von großer Wirksamkeit seien.
Wie auch der deutsche Bindungsforscher, Kinderarzt und Psychoanalytiker Karl Heinz Brisch betont,
profitiert ein Säugling ein Leben lang von intensiver Zuwendung. Daher seien elterliche Aufmerksam-
keit, Feinfühligkeit und Liebe vor allem in den ersten Lebensmonaten unersetzlich. Diese erste Zeit
hat, wie Forschungsergebnisse zeigen, Auswirkungen auf die Qualität der Bindungsentwicklung, wel-
che einen Menschen lebenslang begleitet. Eine sichere Bindung, die positive Auswirkungen im Sinne
eines Schutzfaktors bei Belastungen, besseren Bewältigungsmöglichkeiten, besseren sozialen Kom-
petenzen, aber auch mehr Flexibilität und Ausdauer, sowie bessere Gedächtnisleistungen und Lernen
zur Folge hat, sollte unbedingt gefördert werden.
Die eng an die körperliche Nähe zur jeweiligen Bindungsperson geknüpfte Bindungsentwicklung wird
insbesondere in der Neonatologie in Form von „Kängurutherapie“ bewusst eingesetzt. Außerdem
legen immer mehr Geburtskliniken Wert auf „primäres Bonding“ nach der Geburt. Gerade im Kran-
kenhaus sollte nach geeigneten Lösungen gesucht werden, um möglichen Traumatisierungen durch
frühe Trennung der Mutter-Kind-Einheit vorzubeugen und eine positive Bindungsentwicklung zu för-
dern und unterstützen, sowie das Entstehen von Bindungsstörungen zu vermeiden.

Familienzentrierung im klinischen Alltag


Ein Kind kann per se nicht ohne seinen familiären Hintergrund (Rückhalt oder eben auch fehlende
und störende Einflussgrößen) gesehen werden. Es muss immer wieder berücksichtigt werden, dass
Kinder vom (mehr oder weniger fördernden bzw. schädigenden) Verhalten ihrer Eltern beeinflusst
werden.
Der englische Pädiater und Psychoanalytiker Donald W. Winnicott wurde mit dem Ausspruch bekannt
„There’s no such thing as a baby“. Insbesondere im Falle von Angst, Unsicherheit und Krankheit
brauchen Kinder ihre Bezugspersonen zur Beruhigung und Regulation von Emotionen. Eltern versu-
chen in der Regel das Beste für ihr Kind zu tun und es in allen Situationen bestmöglich in seiner
Entwicklung zu begleiten und zu unterstützen.
Andererseits kann der Umstand, ein krankes Kind zu haben, Gefühle und Zustände bei den Eltern
zum Vorschein bringen (Angst, Unsicherheit, Ohnmacht, Inkompetenzerleben, Stress, Schock…),
welche ihre üblichen, bisher eingesetzten Strategien und Unterstützungsangebote im Rahmen von
Elternschaft außer Kraft setzen können.

44
Laut EACH Charta Art. 2 1 haben „Kinder im Krankenhaus … das Recht, ihre Eltern oder eine andere
Bezugsperson jederzeit bei sich zu haben.“ Die Eltern-Kind-Bindung wird dadurch gewahrt und ge-
stärkt. Je kleiner unsere Patienten, desto klarer wird das Bild von einer psychischen und biologischen
Einheit von Eltern und Kind.
Die sogenannte „Elternarbeit“, ohne die wir in einer modernen pädiatrischen Versorgung nicht aus-
kommen, betrifft das gesamte Team: Eltern müssen als wichtigste Co-TherapeutInnen und PartnerIn-
nen in der Behandlung des Kindes erkannt und gewürdigt werden. Die elterliche Autonomie und
Verantwortlichkeit für ihr Kind sollten im Stationsalltag alle Teammitglieder vor Augen haben.

Eltern im Blick
Zumal die Rolle der Eltern eine tragende (und haltende) ist, muss den Themen Elternidentität sowie
schwierige Konstellationen in der Elternschaft genügend Beachtung und Verständnis entgegenge-
bracht werden.
Der Übergang zur Elternschaft als sogenanntes kritisches Lebensereignis oder als Entwicklungsauf-
gabe bringt eine große Umstellung im äußeren und inneren Raum von Vätern und Müttern bzw. der
Paarbeziehung einer Frau und eines Mannes mit sich. Es ist eine Umstrukturierung von Lebensplänen
und Handlungsroutinen der werdenden Eltern notwendig, die nicht immer spurlos und einfach von
statten geht. Es handelt sich dabei um einen längeren Prozess, der mindestens 1 Jahr andauert und
sich nach der deutschen Psychologin Gloger-Tippelt phasenartig entwickelt.
Menschen haben angeborene „intuitive elterliche Fähigkeiten“, welche ihnen den Zugang zu ihren
Kindern (oder auch zu Kindern im Allgemeinen) erleichtern. Dies birgt bereits aus evolutionärer Sicht
den Vorteil einer besseren Überlebenschance in sich. Ein feinfühliger Umgang mit Babys ist die
Grundvoraussetzung für Bindungsentwicklung und somit ein Schutzfaktor, der unbedingt gefördert
werden muss.
Eltern mit besonderen Belastungen und psychischen Störungen, sowie jugendliche Mütter bzw. Väter,
Eltern mit Intelligenzminderung und Eltern mit Migrationshintergrund bringen ein erhöhtes Risiko für
schwierige Entwicklungsbedingungen für ihre Kinder und möglichen daraus entstehenden Entwick-
lungsstörungen mit sich. Ein besonderer Fall ist die postpartale Depression, die immerhin ca. 25%
der Mütter betrifft und die frühestmöglich erkannt und aufgefangen werden muss.
Wenn wir uns die systemisch-relationale Perspektive noch einmal kurz vergegenwärtigen, wird es
klar, dass umgekehrt kranke oder frühgeborene Kinder, aber auch Kinder mit Beeinträchtigungen
von vornherein einen schwierigeren Start ins Leben haben als gesunde. Dies kann wiederum bei den
Eltern zusätzlich ein feinfühliges Eingehen auf die kindlichen Bedürfnisse erschweren. In weiterer
Folge bedeutet dies unter Umständen eine veränderte Bindungsentwicklung zwischen Eltern und
Kind. Eltern brauchen hier viel Anleitung und Unterstützung, um nicht zusätzlich von Schuldgefühlen,
Versagensängsten oder einer generellen Überforderung außer Gefecht gesetzt zu werden.
In diesem Sinne werden heute Regulationsstörungen wie exzessives Schreien, Schlafstörungen und
Fütterstörungen sowie deren Entstehung ebenfalls als multifaktorielles Geschehen gesehen, das un-
bedingt durch geeignete Unterstützung begleitet werden muss.

Kommunikation
Nach all dem Gesagten dürfte nun klar sein, dass ELC unbedingt als eine interdisziplinärere und
interprofessionelle Angelegenheit betrachtet werden muss. Es ist daher unumgänglich, die Trennung
zwischen den einzelnen Fachgebieten aufzuheben, was wiederum einen guten professionellen Aus-
tausch zwischen Fachleuten aus unterschiedlichen, mit dem Thema zusammenhängenden Diszipli-
nen voraussetzt. Dazu muss in die Erarbeitung von gelingenden Kommunikationsformen und in eine
gute Kommunikationskultur investiert werden.
Natürlich spielt auch die Kommunikation mit den Eltern im Bereich von Early Life Care eine tragende
Rolle. Die Art und Weise, wie Eltern in die Behandlungspläne unserer kleinen PatientInnen (sprich
ihrer kleinen Töchter und Söhne) einbezogen werden, macht gerade beim Erleben und Verarbeiten
von kindlichen Krankheitsverläufen und Krankenhausaufenthalten wesentliche Unterschiede sichtbar.

1
EACH: European Association for Children in Hospital

45
Es geht dabei nicht nur um ausreichende Information, sondern auch um Begleitung, Unterstützung
und Anleitung, welche als sehr positiv für die subjektive Verarbeitung innerhalb der Familien be-
schrieben werden. Ein ausreichendes Maß an Verständnis, Empathie, sowie Übung in Perspektiven-
übernahme können dazu beitragen, dass die Kommunikation mit den Eltern gelingt. Beispielsweise
ist es bei der Kommunikation mit Eltern im Falle von Krankheit oder Belastung von großer Wichtig-
keit, die veränderte und grundsätzlich unterschiedliche Informationsverarbeitung bei Stress im Hin-
terkopf zu haben und dementsprechend angemessene Kommunikationsformen zu wählen.

Zusammenfassung
Early Life Care stellt einen noch sehr jungen, aber sehr notwendigen biopsychosozialen Fachbereich
dar, der an der Schnittstelle von Gynäkologie, Geburtshilfe und Pädiatrie, sowie Psychologie, Päda-
gogik, Prävention und Sozialarbeit immer mehr an Bedeutung gewinnen wird und dem aus gesund-
heitspolitischen Überlegungen heraus unbedingt Einzug in die grundlegende medizinische Ausbil-
dung gewährt werden sollte.
„Was während der Schwangerschaft und den ersten drei Lebensjahren eines Kindes massiv falsch
läuft, wirkt das ganze Leben fort. Es kann, wenn überhaupt, nur unter großen Mühen und hohem
finanziellen Aufwand wieder wettgemacht werden. Deshalb liegt in der Prävention der Schlüssel, je
früher die richtigen Weichen gestellt werden, desto besser“, sagt Klaus Vavrik, Kinderarzt, Kinderpsy-
chiater und Präsident der Liga für Kindergesundheit: "Jeder hier investierte Euro kommt 18-fach zu-
rück."

Literatur
Brisch, K.H.; Hellbrügge, T. (2007) Die Anfänge der Eltern-Kind-Bindung. Schwangerschaft, Geburt und Psychotherapie
Klett-Cotta, Stuttgart
Engel GL. The need for a new medical model: A challenge for biomedicine. Science. 1977; 196 (4286): 129–36.
Felitti,V.J.; Fink P.J.; Fishkin,R.E.; Anda, R.F. Ergebnisse der Adverse Childhood Experiences (ACE) – Studie zu Kindheits-
trauma und Gewalt. Epidemiologische Validierung psychoanalytischer Konzepte Mai 2007, 1. Jahrgang, Heft 2,pp 18-32
Gloger-Tippelt, G. (1988) Schwangerschaft und erste Geburt. Kohlhammer
Harms, T.(2016) Hebammenforum 8/2016, pp 857-860
Roth, G. (2001) Fühlen, Denken, Handeln. Wie das Gehirn unser Verhalten steuert. Suhrkamp
Winnicott, D.W. (1965) Reifungsprozesse und fördernde Umwelt. Studien zur Theorie der emotionalen Entwicklung. Psy-
chosozialverlag, Gießen 2002

46
Medizinische Psychologie in der Behandlung von Abhängigkeitserkrankungen
Maga. Sandra Amesberger, Maga. Dr in. Beatrix Höllbacher-Kneip

In der Universitätsklinik für Psychiatrie, Psychotherapie und Psychosomatik werden PatientInnen u.a.
mit suchtbedingten psychischen Störungen behandelt. Insbesondere werden substanzgebundene Ab-
hängigkeiten behandelt, wobei der Schwerpunkt vom Themenbereich Alkohol gebildet wird.
Welche Verfahren zur Diagnose von Alkoholabhängigkeit gibt es?
Zur Diagnostik einer Alkoholabhängigkeit finden einige kurze Skalen bzw. Fragebögen Verwendung.
Zu unterscheiden sind Selbstbeurteilungsfragebögen und Laborparameter als Screening-Instrumente.
Zu den klinisch relevanten Selbstbeurteilungsfragebögen zählen der Lübecker Alkoholabhängigkeits
und –missbrauchs Screening Test (LAST) und der Alcohol Use Disorder Identification Test (AUDIT)
zur Identifizierung von Risikokonsum. Weiters zählen der CAGE Fragebogen (Cut-down, Annoyed,
Guilty, Eye-opener) und der MALT Fragebogen (Müchner Alkoholismustest) zu den validen Scree-
ningfragebögen.
Trotz der Möglichkeit zur Verfälschung haben sich direkte, auf Selbstaussagen beruhende Fragebo-
genverfahren als valide bewährt. Die entsprechenden Verfahren weisen bessere Gütekriterien auf als
klinische Tests, Laborparameter (y-GT, MCV, CDT, EtG und EtS) und indirekte Fragebogenverfahren,
welche den Alkoholkonsum und dessen Folgen nicht thematisieren.
Wenn PatientInnen stationär aufgenommen werden und in der psychologischen Diagnostik ein Al-
koholproblem erkennbar wurde, ist die Frage zu klären, ob bei ihm eine Alkoholabhängigkeit oder
ein Alkoholmissbrauch vorliegt. Operationalisierte Kriterien zur Diagnostik wurden im DSM 5 (unter:
Störungen im Zusammenhang mit psychotropen Substanzen) und ICD-10 zusammengestellt.
Zu unterscheiden im DSM 5 ist:
• Störung durch Alkoholkonsum (Alkoholkonsumstörung)
• Alkoholintoxikation
• Alkoholentzug
• Andere alkoholinduzierte Störungen
• Nicht näher bezeichnete Störung im Zusammenhang mit Alkohol

Zu unterscheiden im ICD-10 ist:


• F1x.0 akute Intoxikation
• F1x.1 schädlicher Gebrauch
• F1x.2 Abhängigkeitssyndrom

Abhängigkeitskriterien (ICD-10):
1. Toleranzentwicklung
2. Entzugserscheinungen
3. Verlangen
4. Kontrollverlust
5. Vernachlässigung anderer Lebensbereiche
6. Fortsetzung des Konsums trotz negativer Folgen

Welche komorbiden psychischen Störungen treten mit einer Alkoholabhängigkeit auf?


Da komorbide psychische Störungen bei alkoholabhängigen PatientInnen häufig vorkommen, ist
eine differenzierte psychologische Diagnostik von großer Bedeutung. Zur Differenzierung werden
Messinstrumente der Befindlichkeit (Symptom-Checklist, SCL), krankhafter Zustände, vor allem in

47
Bezug auf Angst und Depression (z.B. Hamilton Anxiety Scale, Beck Depressions Inventar, BDI-II)
eingesetzt. Die Begleiterkrankungen können Beginn, Verlauf, Behandlung und Prognose der alko-
holbezogenen Störung beeinflussen.
• Angst- und Panikstörungen treten bei bis zu einem Drittel der alkoholabhängigen Männer
und zwei Drittel der alkoholabhängigen Frauen auf. Eine besonders hohe Komorbidität
scheint für soziale Phobien, Panikstörungen und generalisierte Angsterkrankung zu bestehen.
• Klinische Stichproben zeigen, dass unipolare Depression und Alkoholismus eng miteinander
verbunden sind.
• Je nach untersuchter Stichprobe und verwendeten Messinstrumenten schwanken die Häufig-
keiten diagnostizierter Persönlichkeitsstörungen stark. In den angloamerikanischen Ländern
wurde bei bis zu 50% der alkoholabhängigen PatientInnen als Diagnose eine antisoziale
Persönlichkeitsstörung beschrieben.
• Epidemiologische Erhebungen legen enge Zusammenhänge zwischen Schizophrenie und Al-
koholismus nahe. Die Angaben zur Lebenszeitprävalenz für einen Alkoholismus bei schizo-
phrenen PatientInnen schwanken zwischen 10 und 40%.

Wie wird Alkoholabhängigkeit behandelt?


1) Entzugsbehandlung: Bei der Entzugsbehandlung wird zwischen der körperlichen Entzugsbe-
handlung, dabei steht die körperliche Entgiftung im Vordergrund, und der qualifizierten Ent-
zugsbehandlung, welche neben der körperlichen Entgiftung ein umfassenderes Behand-
lungsprogramm mit psychologischer Betreuung beinhaltet, unterschieden. Das Hauptziel der
qualifizierten Entzugsbehandlung ist eine Unterbrechung des bisherigen Alkoholkonsums
bzw. das Erreichen einer vorläufigen Alkoholabstinenz. Zu den Aufgaben Klinischer Psycho-
logInnen zählen die Klärung und Unterstützung der Abstinenzmotivation, die Planung über-
schaubarer Ziele und die Durchführung einzelner Schritte zur Bewältigung der unmittelbar
anliegenden Probleme von Suchtverlangen oder von Trinkimpulsen.
2) Entwöhnungsbehandlung: So gut sich ein Betroffener nach abgeschlossener Entzugsbehand-
lung auch körperlich fühlen mag, das Rückfallrisiko ist ohne eine weiterführende Behandlung
hoch. Die Entwöhnungsbehandlung hat das Ziel die vorläufige Abstinenzphase zu stabilisie-
ren und Abhängigkeitsprozesse langfristig zu unterbrechen. Diese kann ambulant, teilstatio-
när oder stationär durchgeführt werden.
3) Nachsorge: Rückfallfaktoren entfalten ihre Wirkung erst richtig nach Ende einer Behand-
lungsmaßnahme. Umso bedeutsamer sind eine Teilnahme an Selbsthilfegruppen oder an-
deren Nachsorgeangeboten wie Psychotherapie oder die Inanspruchnahme einer ambulan-
ten Suchtberatung. Ziel ist, die bisherige Abstinenz zu stabilisieren, insbesondere den absti-
nenten Lebensstil bei Wiedereingliederung in die sozialen Bezüge der Arbeitsstelle und der
Familie einzuüben. Unter anderem kam die deutsche MEAT-Studie (Küfner et al, 1999) zu
dem Ergebnis, dass von der Personengruppe, die nach einer stationären Therapie regelmä-
ßig an einer Selbsthilfegruppe teilnahm, nach 4 Jahren 71% der Männer und 45% der
Frauen abstinent geblieben waren. Im Gegensatz dazu belief sich der Anteil der Abstinenten
bei den Männern auf 46% und bei den Frauen 35%, welche die Selbsthilfegruppe unregel-
mäßig oder gar nicht besuchten (siehe auch Bachmeier, et. al, 2017).
Insgesamt betrachtet, zeigen empirische Studien (u.a. von Anton et al, 2006), dass in der Behand-
lung einer Alkoholabhängigkeit die Kombination einer Pharmakotherapie mit psychologischen Inter-
ventionen bessere Ergebnisse im Sinne von längeren abstinenten Phasen verzeichnet.

48
Welche Aufgaben nehmen Klinische PsychologInnen im Rahmen der Suchtbehandlung wahr?
Sowohl während des stationären Aufenthalts zur körperlichen Entgiftung aber insbesondere auch
während einer ambulanten oder stationären Entwöhnungstherapie sind die Aufgaben Klinischer Psy-
chologInnen vielfältig. Im Folgenden werden einige mögliche Themeninhalte einer strukturierten Sit-
zung in Anlehnung an Lindenmeyer (2017) vorgestellt:

a) detaillierte Problemanalyse: eine detaillierte Problemanalyse wird bei Alkoholabhängigen


durch folgende Umstände erschwert: Peinlichkeit, soziale Maskierung, beeinträchtigte Selbst-
wahrnehmung, grobe Vorstellung der Alkoholwirkung. Aus diesem Grund ist die Problem-
analyse nicht nur Teil des diagnostischen Prozesses, sondern stellt häufig bereits einen zent-
ralen Gegenstand der Behandlung dar, da sie eine veränderte Sichtweise der eigenen Ab-
hängigkeit und ein erhebliches Maß an kritischer Selbstreflexion impliziert. Inhalte einer Prob-
lemanalyse sind die Beschreibung der häufigsten Trinksituationen, der Situationsmerkmale,
aber auch Gefühle, Gedanken und Wirkungserwartung.
b) Informationsvermittlung und Auseinandersetzung mit der eigenen Abhängigkeitsentwicklung
c) Vier-Felder-Tafel: Diese Intervention ist eine Kosten-Nutzen-Analyse, mit der eine alkoholab-
hängige Person seine persönlichen Vor- und Nachteile durch künftige Alkoholabstinenz ein-
schätzen soll.
d) Notfallplan zur Überwindung von Rückfällen: Da in der Regel ein Rückfall selbst bei bester
Prognose nicht auszuschließen ist, sollten PatientInnen auf die Möglichkeit eines Rückfalls
vorbereitet werden. Es sollten gemeinsam mit den PatientInnen Strategien für persönliche
Ausstiegsmöglichkeiten erarbeitet werden. Ein Hilfsmittel dafür ist der so genannte „Ausrut-
scher-Vertrag“, welcher mit einer Vertrauensperson geschlossen wird.
e) Nachsorge: Wie empirische Studien zeigen, trägt die Nachsorge einen wichtigen Teil zur
Aufrechterhaltung der Abstinenz bei. Auf Grund dessen sollten PatientInnen Informationen
über Nachsorgeangebote erhalten, und es sollte ein Nachsorgeplan erarbeitet werden.

Die hauptsächlichen positiven Auswirkungen in der psychologischen Behandlung der Alkoholabhän-


gigkeit bestehen nach Back (2007) darin, dass PatientInnen darin unterstützt werden, eine Behand-
lung zu beginnen und diese dann auch weiterhin in Anspruch zu nehmen. Die Information der Pati-
entInnen, ihrer Familien und wichtigsten Bezugspersonen über die Ursachen und Konsequenzen der
Alkoholabhängigkeit, über die Ziele der Behandlung sowie der Vermeidung von Rückfällen stellt eine
weitere Aufgabe Klinischer PsychologInnen dar. Die Einbeziehung ihres persönlichen und beruflichen
Umfelds, welches meist durch die Alkoholabhängigkeit beeinträchtigt wurde, ist für eine Verbesse-
rung der Lebensqualität unabdingbar.

Welche Formen von Rückfällen gibt es?


Definition:
Bezüglich der Frage, was ein „Rückfall“ ist, gibt es nach Körkel und Schindler (2003) unterschiedliche
Sichtweisen:
• „enge“ Rückfalldefinition: jeglicher Alkoholkonsum wird als Rückfall verstanden.
• Definition nach Trinkmenge und Trinkmuster: vor allem in wissenschaftlichen Untersuchun-
gen
• „Ausrutscher“ vs. „Rückfall“: Von einem Ausrutscher wird gesprochen, wenn es bei absti-
nenzmotivierten Alkoholabhängigen, die bereits alkoholfrei lebten, zum ersten Mal wieder
zum Alkoholkonsum gekommen ist. Der Begriff „Rückfall“ wird hingegen dann benutzt, wenn

49
Alkohol wieder wie in früheren Zeiten konsumiert wird. Diese Unterscheidung der beiden
Begriffe hat sich als empirisch haltbar und für die Behandlung sinnvoll erwiesen.
• „subjektive Rückfalldefinition“: Jede Person hat ihr eigenes Verständnis davon, was ein Rück-
fall ist. Ein Rückfall liegt in diesem Sinne erst dann vor, wenn eine subjektiv definierte Kon-
sumschwelle überschritten worden ist.
• „trockener Rückfall“: Vor allem in Kreisen der Abstinenzgruppen werden Denk-, Erlebens-
und Verhaltensweisen, die mit dem früheren Trinkverhalten in Verbindung stehen, als „tro-
ckener Rückfall“ bezeichnet.
• „systemischer Rückfall“: Hierbei handelt es sich um eine Sichtweise, die annimmt, dass ein
Rückfall immer auch beziehungsdynamische Aspekte besitzt und nicht nur auf individuelle
Merkmale der abhängigen Person zurückgeht.
• „iatrogener Rückfall“: Dieser Begriff weist daraufhin, dass die ärztliche Verordnung bzw. Ein-
nahme einer pharmakologischen Substanz (v.a. alkoholhaltiger bzw. stimmungsverändern-
der Medikamente) einen Alkoholrückfall auslösen.

Rückfallhäufigkeiten:

Art der Behandlung Katamnesezeitraum Rückfällige


Personen

stationäre Entgiftungs- bzw. Entzugsbehandlung 1 Monat 33 – 50%

1 Jahr 67 – 84%

stationäre Entwöhnungsbehandlung 6 Monate 33 – 46%

1 Jahr 36 – 57%

7 Jahre 65 – 85%

ambulante Entwöhnungsbehandlung 28 Monate 27 – 65%

Zusammenfassend ist anzumerken, dass die ersten drei Monate nach Erreichen der Abstinenz die
Zeit des größten Risikos darstellen, wieder Alkohol zu konsumieren – unabhängig davon, ob eine
Entgiftungs- oder Entwöhnungsbehandlung vorausgegangen ist. Bezüglich einer Rückfallhäufigkeit
in Abhängigkeit vom Geschlecht gibt es keinen eindeutigen empirischen Trend. Ebenso scheint für
die Höhe der Rückfallquoten der angewandte Therapieansatz wenig bedeutsam zu sein.
Die weltweit größte Therapiestudie (Project MATCH, 1997) zeigte, dass die Variation des Behand-
lungserfolges mehr durch Therapeutenmerkmale als durch den spezifischen Behandlungsansatz zu
erklären ist.

50
Literatur:
Altmannsberger, W. (2004). Kognitiv-verhaltenstherapeutische Rückfallprävention bei Alkoholabhängigkeit. Ein Train-
ingsmanual. Göttingen: Hogrefe.
Anton, R., O’Malley, S., Ciraulo, D. et al. (2006). Combined Pharmacotherapies and Behavioral Interventions for Alco-
hol Dependence. JAMA, 295(17) 2003-2017.
Arbeitsgemeinschaft der wissenschaftlichen medizinischen Fachgesellschaft (2003). Leitlinien der deutschen Gesellschaft
für Suchtforschung und Suchttherapie: Akutbehandlung alkoholbezogener Störungen. Sucht, 49(3), 147–167.
Bachmeier, R. et.al (2017). Effektivität der stationären Suchtrehabilitation – FVS-Katamnese des Entlassjahrgangs 2014
von Fachkliniken für Alkohol- und Medikamentenabhängige. Sucht Aktuell, 2017 (1), 53-69.
Brueck, R. & Mann, K. (2007). Alkoholismusspezifische Psychotherapie. Manual mit Behandlungsmodulen. Köln: Deutscher
Ärzteverlag.
Di Nicola, M. et.al (2015). Co-occurrence of alcohol use disorder and behavioral addictions: relevance of impulsivity and
craving. Drug and Alcohol Dependence. 148, 118-125.
Garbutt, J.C. et. al (2014). Pharmacological Treatment of Alcohol Dependence. A Review of Evidence. JAMA, 281, 14.
Johann, M., Lange, K., & Wodarz, N. (2007). Psychiatrische Komorbiditäten bei Alkoholabhängigen. Psychiatrische Praxis,
34(1), 47-48.
Körkel, J., & Schindler, C. (2003). Rückfallprävention mit Alkoholabhängigen: das strukturierte Trainingsprogramm
S.T.A.R. Berlin: Springer Verlag.
Lieb. K., & Frauenknecht, S. (2012). Instensivkurs Psychiatrie und Psychotherapie. (7. Aufl.). München: Urban & Fischer.
Lindenmeyer, (2017). Alkoholabhängigkeit. (Fortschritte der Psychotherapie, Band 6) (2. überarb. Aufl.). Göttingen: Ho-
grefe.
Mann, K.F., Löber, S. et al. (2006). Qualifizierte Entzugsbehandlung von Alkoholabhängigen. Ein Manual zur Pharmako-
und Psychotherapie. Köln: Deutscher Ärzteverlag.
Mann, K. (2015). Können Alkohlabhängige ihre Trinkmengen durch eine Kombination von Medikamenten und Kurzinter-
ventionen dauerhaft reduzieren? Sucht, 61 (1), 29-36.
Schneider, R. (2013). Die Suchtfibel. Hohengrefen: Schneider Verlag.

51
Einführung in die Gerontopsychologie
Maga. Petra Kröll, Mag. Wolfgang Treschnitzer

Der alte Mensch im Krankenhaus


Die häufigsten Funktionsdefizite hochbetagter, multimorbider PatientInnen stellen die Immobilität
(z.B. nach Insult), die Instabilität (z.B. Sturzneigung), die Inkontinenz, der intellektuelle Abbau (z.B.
Demenz) sowie Isolation (Vereinsamung) und iatrogene Störungen (z.B. Medikamenten Interaktio-
nen) dar. Sensorische Beeinträchtigungen, die alle Sinnesmodalitäten betreffen können, sollten im
Umgang mit älteren PatientInnen berücksichtigt werden. Besonders Einschränkungen des Hör- und
Sehvermögens können sich erschwerend auf die Kommunikation auswirken. Im Gespräch mit älteren
PatientInnen ist es wichtig laut, deutlich und in klar formulierten Sätzen zu sprechen. Überhastete
Sprechweise, Negativ-Formulierungen, Schachtelsätze und Fachausdrücke sollten vermieden wer-
den.
Das multiprofessionelle Team
Das Ziel des „geriatrischen Assessments“ ist es den alten Menschen auf seine physischen, geistigen
und sozialen Beeinträchtigungen zu untersuchen und darauf aufbauend sinnvolle therapeutische
Wege einzuschlagen. Dabei steht die Rückführung der älteren PatientInnen zur größtmöglichen
Selbstständigkeit in seinem gewohnten Lebensumfeld, wenn nach einer akuten Erkrankung oder an-
deren Entwicklungen heraus, die Aufnahme in ein Pflegeheim ansteht, im Mittelpunkt. Eine der gro-
ßen Herausforderungen der näheren Zukunft ist in diesem Zusammenhang die Multimorbidität der
alten Menschen, vor allem bei demenzkranken PatientInnen. Erst durch die multiprofessionelle Zu-
sammenarbeit entsteht ein ganzheitliches Bild der PatientInnen, das eine optimale Behandlung er-
möglicht. Das multiprofessionelle Team besteht aus ÄrztInnen, Pflegepersonal, PsychologInnen, Er-
gotherapeutInnen, LogopädInnen, PhysiotherapeutInnen, SozialpädagogInnen und MasseurInnen.
Klinisch-psychologische Diagnostik, Psychotherapie und Beratung
Einen Aufgabenbereich klinischer PsychologInnen stellt die klinisch-psychologische Diagnostik dar,
die auf die Erfassung und Objektivierung von psychischen und kognitiven Beeinträchtigungen sowie
die Identifikation psychischer Störungen („Diagnosestellung“). Sie dient der Entscheidungsfindung
(z.B. Wahl der richtigen Therapieform, Behandlungsnotwendigkeit) sowie der Evaluation der Behand-
lung. Zum Einsatz kommen dabei wissenschaftliche Methoden wie Beobachtung, strukturierte Inter-
views, Anamnesen, Fremdanamnesen, Fragebogen, und psychologische Tests.
Ein weiterer Aufgabenbereich klinischer PsychologInnen ist die Beratung, vor allem die Beratung von
Angehörigen. Diese werden dabei unterstützt, ihre Bedürfnisse, Fähigkeiten und Grenzen wahrzu-
nehmen und persönliche Kompetenzen zu entwickeln. Das Angebot umfasst psychoedukative Inter-
ventionen, die neben Informations- und Wissensvermittlung auch über Veränderungen bei verschie-
denen Krankheitsbildern (z.B. Demenz, Depression) aufklärt. Außerdem wird über den angemesse-
nen Umgang mit Krankheitssymptomen (z.B. Verhaltensauffälligkeiten bei Demenzkranken) infor-
miert. Daneben werden die Angehörigen zu entlastenden Angeboten (z.B. Tagespflege, Selbsthilfe-
gruppen) beraten.
Demenzdiagnostik und Psychotherapie von Demenzkranken
An der Universitätsklinik für Geriatrie werden durchgeführt:
• Frühdiagnostik von Demenzen
• Differentialdiagnostik Demenz/Depression/Delir
• Differentialdiagnostik verschiedener Demenzformen
• Psychodiagnostik (v.a. Depression, Angsterkrankungen, PTBS, Anpassungsstörungen,
wahnhafte Störungen)

52
• Kognitives Training
• Einzelpsychotherapie
• Biografiearbeit
• Beratung der Kollegen hinsichtlich Diagnostik, Therapie und Umgang mit Demenzkranken
Normales Altern
• Einfache Reaktionszeit: Abnahme ab 25. Lebensjahr
• Aufmerksamkeit / Merkfähigkeit: Höhepunkt zwischen dem 20. und 30. Lebensjahr
• räumliche Leistungen, Urteilsvermögen Höhepunkt ab dem 40. Lebensjahr, dann lange
gleichbleibende Abnahme im 6., 7. oder 8. Lebensjahrzehnt
• Wortschatz und allgemeines Wissen kann bis ins hohe Alter ansteigen
Gutartige Altersvergesslichkeit
• subjektiv erlebte Gedächtnisminderung alter Menschen
• Erschwernis, sich an persönlich weniger wichtige Aspekte von Ereignissen zu erinnern
• normale Alterserscheinung
• hängt auch mit Altersbild zusammen: alt = vergesslich

Leichte kognitive Störung (Mild Cognitive Impairment, MCI)


• unscharf definierter Begriff
• erworbenes organisches kognitives Defizit, das nicht oder nur in geringem Maß zu einer
Alltagsbeeinträchtigung führt
• häufig, aber nicht immer Vorstadium einer Demenzerkrankung
• vor allem beim „amnestischen“ Subtyp mit ganz im Vordergrund stehender Gedächtnisstö-
rung, wodurch das Risiko einer mittelfristigen Entwicklung einer Alzheimer-Demenz stark
erhöht ist (Schmidtke u. Hermeneit 2008), 50-80 % Konversionsraten
• ICD-10: F06.7
• Verlaufsuntersuchung in 6 Monaten

Demenz - Definition
Demenz (ICD-10-Code F00-F03) ist ein Syndrom als Folge einer meist chronischen oder fortschrei-
tenden Krankheit des Gehirns mit Störung vieler höherer kortikaler Funktionen, einschließlich Ge-
dächtnis, Denken, Orientierung, Auffassung, Rechnen, Lernfähigkeit, Sprache, Sprechen und Urteils-
vermögen im Sinne der Fähigkeit zur Entscheidung. Das Bewusstsein ist nicht getrübt.
Für die Diagnose einer Demenz müssen die Symptome nach ICD über mindestens sechs Monate
bestanden haben. Die Sinne (Sinnesorgane, Wahrnehmung) funktionieren im für die Person üblichen
Rahmen. Gewöhnlich begleiten Veränderungen der emotionalen Kontrolle, der Affektlage, des So-
zialverhaltens oder der Motivation die kognitiven Beeinträchtigungen; gelegentlich treten diese Syn-
drome auch eher auf. Sie kommen bei Alzheimer-Krankheit, Gefäßerkrankungen des Gehirns und
anderen Zustandsbildern vor, die primär oder sekundär das Gehirn und die Neuronen betreffen. Es
ist davon auszugehen, dass bisher bei über der Hälfte aller Senioren (älter als 65 Jahre) im allge-
meinärztlichen PatientInnengut mit vorhandener Demenz nie eine solche Diagnose gestellt wurde.

Demenzformen
Bei der Demenz können verschiedene Formen unterschieden werden, wie z.B. die Alzheimer Demenz,
Levy-Body Demenz, Parkinsondemenz, frontotemporale Demenz (Verhaltensstörung), semantische
Demenz (Flüssige Aphasie und Verlust des semantischen Wissens), posteriore kortikale Atrophie (Seh-
störung), primär progressive Aphasie (Sprachstörung) und vaskuläre Demenz.

53
Die häufigsten Formen sind die Alzheimer Krankheit gefolgt von der vaskulären und der Levy-Body
Demenz. Im klinischen Alltag sind jedoch oft Mischformen zu beobachten.

Delir (delirium) - Definition


Veraltete Begriffe sind: akuter exogener Reaktionstyp, akuter Verwirrtheitszustand, Durchgangssyn-
drom (Klinikjargon: „Patient ist durchgängig“), „postoperative Psychose“, „acute brain syndrome“.

Symptomatik Nach DSM 5 und ICD 10


Leitsymptom ist eine Bewusstseinstrübung (reduzierte Klarheit der Umgebungswahrnehmung)
Der Beginn eines Delirs wird als akut wahrgenommen. In vielen Fällen können bereits tage- oder
wochenlange Vorboten wie Schlaflosigkeit, Zittern oder Angst festgestellt werden.

Bei der Dauer eines Delirs wird zwischen akut (ein paar Stunden bis Tage) und andauernd (Wochen
bis Monate) unterschieden.

Formen des Delirs:


Hypoaktives Delir: Ist die häufigste Form des Delirs, wird allerdings häufig nicht erkannt. Eine Ver-
wechslung mit Depression ist möglich. Häufige Symptome sind Bewegungsarmut, Antriebslosigkeit
und Lethargie
Hyperaktives Delir: Häufig Unruhe, auch Ablehnen von Behandlungen; Stimmungslabilität. Oft
Symptome wie „Nesteln“ und Weglauftendenzen.

Risikofaktoren für das Auftreten eines Delirs:


• Demenz oder leichte kognitive Störung sind der größte Risikofaktor
• hohes Alter
• schwere körperliche Erkrankungen
• Infektionen
• postoperativ
• bei Operationen am offenen Herzen und orthopädischen Eingriffen über 50%
• Medikamente: bei 40% aller Delirien Medikamente ursächliche Rolle, besonders häufig
Delirien bei Menschen, die vor stationärer Aufnahme mit Benzodiazepinen behandelt wur-
den, vor allem bei Polypharmazie im Alter (mehr als 6 Medikamenten hohes Risiko)
• metabolische Störungen wie Elektrolytentgleisungen, Dehydratation oder Hypoxie

Delir ist ein Risikofaktor für negativen Outcome, erhöhten Pflegeaufwand, verlängerte Verweil-
dauer, erhöhte Kosten und Mortalitätsrate.

Demenzdiagnostik in der Praxis


Nicht alle Angaben, die in Lehrbüchern zu finden sind, stimmen mit Erfahrungen aus der Praxis
überein. Folgende Fallstricke können zu falschen Diagnosen führen:
Nicht nur der Beginn einer Depression, sondern auch der Beginn einer Demenz wird oft als plötzlich
erlebt (Auslöser können kritische Lebensereignisse wie z.B. Tod der Ehefrau, Krankenhausaufenthalt
etc. sein). Gerade Angehörige schildern dies oft so. Das akute Ereignis hat allerdings lediglich dazu
geführt, dass die schon (oft seit Jahren) bestehende Demenz klinisch manifest wird. Auch bei Demenz
liegt oft Krankheitseinsicht vor („Ich bin vergesslich“, „Bei mir stimmt etwas nicht“). Viele PatientInnen
können ihre Einschränkungen sehr detailliert beschreiben, beziehen diese aber oft auf das „Älterwer-
den“. Sozialer Rückzug findet sich nicht nur bei Depressionen sondern oft auch bei Demenz wegen
Versagensängsten.

54
Der Verlauf ist trotz Progredienz auch bei Demenz schwankend (Tagesform, erstaunliche Einzelleis-
tungen), vor allem bei vaskulären Demenzen.
Anamnese
Besonders wichtig für die Diagnosestellung ist ein ausführliches Anamnesegespräch mit dem Patien-
tInnen in Form eines halbstrukturierten Interviews. Man gewinnt dabei wichtige Informationen über
das Bildungsniveau, mögliche Sprachstörungen, Altgedächtnisstörung („Langzeitgedächtnis“), mög-
liche psychische Belastungen (inkl. Traumatisierungen) sowie Nervosität und Angst in der Untersu-
chungssituation.
Fremdanamnese
Um die Alltagsrelevanz der Beschwerden beurteilen zu können, werden Fremdinterviews (third party
interview) durchgeführt und/oder die PatientInnen im Stationsalltag beobachtet. Neben Angehörigen
sollten – wenn möglich - auch Pflegekräfte und Therapeuten befragt werden.
Screeningverfahren
Häufig in der Klinik und zunehmend auch in ärztlichen Praxen eingesetzte Screeningverfahren sind:

Mini Mental Status Examination (MMSE)


Der MMSE von Folstein (1975) wird häufig eingesetzt, obwohl er für die Frühdiagnostik keine siche-
ren Ergebnisse liefert und deshalb dafür nicht geeignet ist. Zwischen normaler kognitiven Leistungs-
fähigkeit und leichter kognitiver Störung kann er nicht unterscheiden. Wegen der kurzen Durchfüh-
rungszeit ist er aber dennoch weit verbreitet und gehört zum „Geriatrischen Assessment“ (Praxis und
Klinik) dazu. Der MMSE kann einen Demenzverdacht stützen. Außerdem kann man Verläufe bei
bekannter Demenz dokumentieren. Der durchschnittliche Punkteverlust beträgt pro Jahr bei demen-
ten PatientInnen 4 MMSE-Punkte. Dieses Demenzscreening prüft verschiedene kognitive Leistungen,
darunter die zeitliche und örtliche Orientierung, das Gedächtnis (3 Wörter), die Konzentration und
Rechenfähigkeit, Auffassung und Verständnis, Visuokonstruktion und sprachliche Leistungen.

Uhrentest
Der Uhrentest (modifiziert nach Shulman et al., 1993) kann auch einen Demenzverdacht stützen. Es
werden visuokonstruktive Fähigkeiten und konzeptgebundenes Denken (im Sinne der Umsetzung ei-
nes im Gedächtnis repräsentierten Zeitbegriffs in eine visuoräumliche Beziehung) geprüft. Menschen
mit Depressionen haben hier keine Schwierigkeiten. Bisher liegt keine ausreichende Normierung vor.
Die Werteskala reicht von 1 (ohne Fehler) bis 6 Punkten (keine Uhr erkennbar). Ein Score von 3-6
Punkten wird als pathologisch angesehen.
Neuropsychologische Diagnostik
Auf der Basis der vorliegenden Screening-Informationen wird je nach Fragestellung bzw. Arbeitshy-
pothese (z.B. normales Altern, leichtgradige Demenz) eine Testbatterie zur Abklärung der kognitiven
Leistungsfähigkeit zusammengestellt und durchgeführt.
Als Beispiel für Testbatterien seine hier genannt:
• Consortium to Establish a Registry of Alzheimer´s Disease (CERAD)
• Montreal Cognitive Assessment (MOCA)
• Syndrom Kurztest (SKT)
• Rivermead Behavioral Memory Test (RBMT)
Da neben der Merkfähigkeit (im Volksmund Kurzzeitgedächtnis) eine weitere kognitive Funktion in
„alltagsrelevantem Ausmaß“ eingeschränkt sein muss, damit die ICD-10-Kriterien erfüllt sind, unter-
suchen die zuvor genannten Testverfahren weitere kognitive Funktionen wie sprachliche Parameter,
Planungs- und Organisationsfähigkeit, Urteilsfähigkeit oder der Orientierung. Die Progredienz einer
Demenzerkrankung wird durch regelmäßige psychodiagnostische Verlaufsuntersuchungen ermittelt.

55
Bildgebende Verfahren, Liquordiagnostik, Labor-Ausschlussdiagnostik
Ein MRT mit Hippocampusvolumetrie kann zur Diagnosefindung beitragen. Ein unauffälliger Befund
bedeutet jedoch nicht, dass keine Demenz vorliegt. Bei einigen PatientInnen erfolgt eine Liquorpunk-
tion. Die Indikation für diese Untersuchung sollte sorgfältig gestellt werden. Blutbild, CRP oder Blut-
senkung (Hinweise für entzündliche/vaskulitische Erkrankungen), TSH (Hypothyreose), GOT, CK,
LDH, Harnstoff, Glukose (schwere internistische Erkrankungen), Vitamin-B12- und Folatspiegel,
Lues-Suchtest (nach Ermessen).

Differentialdiagnostik Demenz / Delir / Depression


Differenzialdiagnostik ist wegen der therapeutischen Konsequenzen (Medikation, Psychotherapie),
Prognose und Zukunftsplanung wichtig. Häufig kommt es auch zu einem gemeinsamen Auftreten
von Demenz und Depression. Hypothesen dazu sind:
• Depression als „Vorbote“ einer Demenz (in der Anamnese Demenzkranker häufig depressive
Episoden)
• Depression als mögliche Reaktion auf eine Demenz
• gemeinsame Ursachen oder
• belastende / traumatische Lebensereignisse als Ursache einer Demenz
Fragestellungen die hier auftreten: „Lassen sich die kognitiven Störungen allein im Rahmen einer
Depression erklären?“ bzw. „Liegt bei den PatientInnen eine Depression vor, oder hat er/sie eine
Depression und eine Demenz?“

Depression vs. Demenz


Depression Demenz
Benennens- und Wortfindungsstörun-
unauffällig
Sprache gen,
evtl. verlangsamt
teilweise floskelhaft
Orientierung unauffällig gestört
gestört, flache Lernkurve
oft gestört, keine demenzatypi-
Merkfähigkeit Perseverationen
schen Auffälligkeiten
Konfabulationen
Aufmerksamkeit oft gestört oft gestört
räumliche Fähigkeiten unauffällig kann gestört sein
selten, dann meist Schuld-
Gelegentlich, eher nicht im Frühsta-
Wahn oder Versündigungs-,
dium
Verarmungswahn
Tests gleichen Schwierig- gleichmäßige
Leistungsschwankungen
keitsgrades Leistungsminderung
oft adäquat, teilweise Bagatellisieren
Subjektive Klagen stärker als objektiver Befund
von Defiziten
Schuldgefühle, Versagensangst,
Testsituation Selbstabwertung, oft hoch motiviert
wenig Antrieb
ADL Erhalten in mittlerem Stadium eingeschränkt

Delir-Demenz
Wichtigste Differenzialdiagnose des Delirs ist die Demenz. In der Regel kann aber bereits durch eine
gute Anamnese und evtl. ergänzend eine Verhaltensbeobachtung mit Austausch im Team die Unter-
scheidung sicher erfolgen, auch wenn die Symptome sehr ähnlich sind. Ein Delir kann eine Demenz
überlagern und deren Symptomatik verstärken. Umgekehrt kann sich ein Delirium zu einer Demenz
weiterentwickeln.

56
Demenz Delir
akuter Beginn ja
Bewusstseinstrübung ja
gestörter Tag-Nacht-Rhythmus nur in Spätstadien oft
„gute und schlechte Schneller Wechsel (innerhalb
fluktuierender Verlauf
Tage“ Stunden oder Minuten)

Klinisch-psychologische Behandlung

Unter psychologischer Behandlung wird die Behandlung von psychischen Beeinträchtigungen mittels
klinisch-psychologischen Interventionen, die im Erleben und Verhalten ansetzen und sich in der sozi-
alen Interaktion zwischen Helfern und Hilfesuchenden vollziehen, verstanden. Diese dienen der Ge-
sundheitsförderung und Prävention sowie der Behandlung und Therapie ebenso wie der Rehabilita-
tion. Es handelt sich um zielorientierte Prozesse zur Erreichung von Veränderungen, die theoretisch
fundiert sind und empirische Wirksamkeit aufweisen (Perez & Baumann, 1998). Dazu gehören unter
anderem das psychologische Gespräch und therapeutische Interventionen (z.B. Ressourcenanalyse,
Aktivitätsaufbau, Förderung emotionaler Bewältigung, kognitive Restrukturierung), Beziehungsarbeit
oder Entspannungstechniken. Übergreifende Therapieziele beziehen sich unter anderem auf die För-
derung der Selbstständigkeit, die Verbesserung sozialer Fähigkeiten, Klärung zwischen den Genera-
tionen, Lebensbilanzierung (Akzeptanz und Aussöhnung mit dem gelebten Leben), Bearbeiten von
Verlusterfahrungen und Auseinandersetzung mit dem Altern und Tod. Die häufigsten Störungsbilder,
die an der Universitätsklinik für Geriatrie neben der Demenz behandelt werden, sind:
• somatoforme Störungen (Prävalenz 24%; ELDERMEN-Studie, Schneider et al. 2003)
• Depressionen (Suizidalität) bzw. depressive Störung (Prävalenz 9% plus 18% subdiagnos-
tische Depressionen; Helmchen et al. 1996)
• Angststörungen (Prävalenz 5%; Helmchen et al. 1996)

Klinisch-psychologische Beratung der Angehörigen


Das Hauptanliegen der psychologischen Beratung liegt darin, die Angehörigen dabei zu unterstüt-
zen, eigene Potentiale zu entwickeln und wahrzunehmen sowie Bewältigungskompetenzen zu för-
dern. Das Angebot umfasst psychoedukative Interventionen, die neben Informations- und Wissens-
vermittlung auch über Veränderungen bei den verschiedenen Krankheitsbildern (z.B. Demenz, De-
pression) aufklärt. Weiters wird über den angemessenen Umgang mit Krankheitssymptomen (z.B.
Verhaltensaufälligkeiten bei Demenzkranken) informiert und beraten. Daneben werden die Angehö-
rigen über entlastende Angebote (Tageszentren, Selbsthilfegruppen,...) in Kenntnis gesetzt.

Literatur:
Förstl, H. (Hrsg.) (2003). Lehrbuch der Gerontopsychiatrie und –psychotherapie. Grundlagen-Klinik-Therapie (2. Aufl.).
Stuttgart: Thieme.
Häcker, H.-O. & Stapf, K.-H. (2004). Dorsch. Psychologisches Wörterbuch. Bern: Hans-Huber.
Maercker, A. (2002). Alterspsychotherapie und klinische Gerontopsychologie. Berlin Heidelberg: Springer.
Oswald, W.D., Gatterer, G. & Fleischmann, U.M. (2008). Gerontopsychologie. Grundlagen und klinische Aspekte der
Psychologie des Alterns. Wien: Springer.
Perez, M., & Baumann, U. (1998). Lehrbuch: Klinische Psychologie – Psychotherapie (2. Ausg.). Bern: Hans-Huber.

57
Psychoonkologie
Mag. Franz Wendtner

Was ist Psychoonkologie?


Die Psychoonkologie ist eine noch relativ junge, fachübergreifende Disziplin der Klinischen Psycho-
logie bzw. Psychotherapie, Psychiatrie und Sozialarbeit in der Onkologie. Sie befasst sich vorwiegend
mit den psychischen, familiären und sozialen Aspekten von Krebs und den Auswirkungen von Krebs-
erkrankungen auf das individuelle Leben von PatientInnen und Angehörigen. Ihr therapeutisches Ziel
ist die psychosoziale Unterstützung von Menschen in einer sehr belastenden Lebenssituation. Wis-
senschaftlich nachgewiesen ist, dass sich durch psychoonkologische Behandlung und Betreuung die
Krankheitsverarbeitung und die Lebensqualität der Betroffenen deutlich verbessert. Frühere Lehrmei-
nungen bezüglich einer Typ C (= Krebs-) Persönlichkeit haben sich nicht bestätigt. Forschungser-
gebnisse bezüglich einer Steigerung der Lebenserwartung von psychoonkologisch betreuten Patien-
tInnen bleiben widersprüchlich.

Was sind die Aufgabengebiete und Ziele der Psychoonkologie?


Die Diagnose Krebs trifft viele PatientInnen völlig unvorbereitet wie ein Blitz aus heiterem Himmel.
Ab sofort ist alles anders, sind alle Lebenspläne und –entwürfe in Frage gestellt und bedroht. Erste
Gedanken bei der Eröffnung der Diagnose Krebs kreisen häufig um Tod, Siechtum und Schmerz und
bald stellt sich die Frage, wie es nun weitergehen soll. V. a. Angst, Gefühle von Hilflosigkeit und
Ausgeliefertsein, sowie depressive Symptome begleiten viele PatientInnen temporär intermittierend
während der Erkrankung und darüber hinaus. In der psychoonkologischen Literatur zeigt sich, dass
rund ein Drittel der PatientInnen zumindest vorübergehend eine psychische Belastung in behand-
lungswürdigem Ausmaß aufweist. Auch Angehörige sind als Betroffene großen psychischen Belas-
tungen ausgesetzt.

Im psychoonkologischen Kontext gehören zu den meistdiagnostizierten Behandlungsindikationen


nach ICD 10:
F43.0 Akute Belastungsreaktion
F43.2 Anpassungsstörung in ihren verschiedenen Ausprägungen (F43.20 – 43.28)
F32 Depressive Episoden v. a. reaktiver Natur
F45.4 Anhaltende somatoforme Schmerzstörung
F52 Mangel oder Verlust von sexuellem Verlangen

Zu den am häufigsten angewandten Interventionen gehören:


Krisenintervention (bei der Diagnoseeröffnung, bei Verschlimmerung, bei Rezidiven)
Supportive Intervention (Unterstützung bei Angstbewältigung und Krankheitsverarbeitung)
Psychoonkologische und psychotherapeutische Diagnostik, Behandlung und Begleitung
Psychoedukation (Information zur Förderung der Copingkompetenzen)
Gruppentherapie (Entspannung, Qigong)
Angehörigenbetreuung (optimalerweise von Anfang an)
Sterbebegleitung
Hinterbliebenenbetreuung

Zur Verbesserung der Krankheitsbewältigung und der Lebensqualität kommt es durch:


Reduktion von Stress, Angst, depressiven Symptomen und Schmerzen
Verbesserung der psychischen und physischen Befindlichkeit sowie der Lebensqualität
Unterstützung bei der Entwicklung effektiven Copings
Unterstützung bei der Mobilisierung persönlicher Ressourcen
Optimierung der Kommunikation zwischen Betroffenen, Beteiligten und Behandlern

58
Stress
Stress ist ein sehr subjektives Geschehen und wird von verschiedenen Individuen unterschiedlich er-
lebt. Über vegetativ gesteuerte Vorgänge kommt es zu Reaktionen im körperlichen, kognitiven und
affektiven Bereich, sowie zu Verhaltensänderungen. Wesentlich für die Vermittlung der Stressreakti-
onen ist das auf verschiedenen hierarchischen Ebenen ablaufende, in komplexer Weise durch Rück-
kopplungsschleifen verzahnte Zusammenspiel von ZNS und Vegetativum sowie den entsprechenden
neuroendokrinen Prozessen. Transaktional betrachtet ist Stress eine Beziehung zwischen dem be-
troffenen Menschen und der Stress auslösenden Situation bzw. den Stressoren. Zur entsprechenden
Einschätzung kommt es im Rahmen der primären Bewertung. Sekundäre Bewertungen können über
Informationen und Rückmeldungen die Einschätzung modifizieren und zu Neubewertungen führen.
Als Stressoren werden sowohl internale wie externale situative Anforderungen aufgefasst, welche die
Grenzen der Reaktionskompetenzen und Bewältigungsfertigkeiten des Betroffenen erreichen oder
überfordern, wobei Kontrollüberzeugungen und Selbstwirksamkeitserwartungen sowohl für die sub-
jektiv wahrgenommene Belastung als auch für die Auswahl der Lösungsstrategien von entscheiden-
der Bedeutung sind.

Kontrollüberzeugung
Das Konstrukt des „locus of control of reinforcement“ bezieht sich auf die generalisierten Erwartungs-
haltungen einer Person hinsichtlich ihres subjektiv wahrgenommenes Ausmaßes an Kontrolle, das sie
über eine Situation entfalten kann. Man unterscheidet die internale und externale Kontrollüberzeu-
gung. Kontrollüberzeugungen gelten als relevante Moderator-variablen hinsichtlich Wahrnehmung,
Bewertung und Wirkung von Stressoren.

Internale Kontrollüberzeugung
Internale Kontrollüberzeugung liegt vor, wenn ein Individuum ein Ereignis als Konsequenz des eige-
nen Verhaltens wahrnimmt und bezieht sich daher auf die Überzeugung, selbst wesentlich Einfluss
auf den Verlauf eines Geschehens nehmen zu können. Menschen mit einer hohen internalen Kon-
trollüberzeugung gehen a priori davon aus, dass primär sie selbst es sind, die Kontrolle über die
Ereignisse haben. Sie neigen dazu, unter Stressbedingungen eher problembezogene Bewältigungs-
strategien einzusetzen.

Externale Kontrollüberzeugung
Externale Kontrollüberzeugung wird in soziale (powerful others control) und fatalistische (chance
control) externale Kontrollüberzeugung unterteilt. Sie liegt vor, wenn ein Ereignis als vom eigenen
Verhalten nicht oder nur wenig beinflussbar wahrgenommen wird. Menschen mit einer externalen
Kontrollüberzeugung gehen davon aus, keine oder eine nur unwesentliche Kontrolle über das Ge-
schehen entfalten zu können (verbunden mit Gefühlen von Hilflosigkeit). Stattdessen gehen sie davon
aus, dass andere Personen (soziale Externalität) oder zufällige Faktoren wie Glück oder Schicksal
(fatalistische Externalität) auf die Situationen Einfluss nehmen und greifen in Stresssituationen eher
auf emotionsbezogene Strategien zurück.

Selbstwirksamkeitserwartung
Das verwandte Konzept der „self-efficacy“ bezieht sich auf die subjektve Einschätzung des individu-
ellen Ausmaßes an (genereller) Bewältigungsfähigkeit/-fertigkeit. PatientInnen mit einer hohen Selbst-
wirksamkeitserwartung trauen sich mehr zu, sie agieren in der Regel effizienter und geben weniger
früh auf als PatientInnen mit einer geringen Selbstwirksamkeitserwartung, die auf belastende Situati-
onen eher ausweichend-vermeidend und resignierend reagieren.
Therapieansätze, welche die internale Kontrollüberzeugung und die Selbstwirksamkeit unterstützen,
fördern die Selbstkompetenz und damit die Krankheitsbewältigung. Zu ihnen gehört die kognitiv-
behaviorale Therapie (“Cognitive Behaviour Therapy“).

59
Was versteht man unter „Cognitive Behaviour Therapy“ (CBT)?
Die CBT enthält ein psychoonkologisches Kurztherapieprogramm für KrebspatientInnen und ihre An-
gehörigen (Greer et. al., 2006). Ein zentrales Element bildet neben der Behandlung der krankheits-
bezogenen Belastungen die persönliche Bedeutung der Erkrankung für die Betroffenen. In sechs
Sitzungen von je einer Stunde Dauer sollen effektive Copingstrategien unter Einbeziehung des Stress-
Modells nach Lazarus und Launier entwickelt, bzw. vermittelt werden. Wesentliche Elemente sind
dabei die Förderung von Kontrolle und aktiver Teilnahme an der Behandlung, weiters die Entspan-
nung, sowie die Vermittlung kognitiver und behavioraler Techniken, wie z.B. die Identifikation auto-
matischer negativer Gedanken und Entkatastrophisierungen. Weitere Schwerpunkte dieser Kurzzeit-
therapie bilden die Induktion von „Fighting spirit“, das Erreichen eines aktiven Bewältigungsstiles bei
realistisch–optimistischer Haltung, sowie die Optimierung der Kommunikation mit dem Partner.

Welche Psychotherapieformen werden noch angewandt?


In der Psychoonkologie kommen noch weitere Verfahren wie z. B. Psychodrama, Gestalttherapie,
Imagination und körperorientierte Verfahren, um nur einige zu nennen, zur Anwendung. Auch nicht
primär psychologische Herangehensweisen wie Qigong und Musiktherapie – aktiv und rezeptiv –
finden mehr und mehr Eingang in die Behandlung von KrebspatientInnen.

Welche Bedeutung hat die Entspannung in der Behandlung und Unterstützung von KrebspatientIn-
nen?
Entspannung dient der Reduktion körperlicher und psychischer Anspannung und der Stressreduktion.
Ein wesentlicher Baustein vieler Therapieansätze ist die Progressive Muskelrelaxation (PMR) nach
Jacobson.

Wie funktioniert die Progressive Muskelrelaxation nach Jacobson?

Eine Übungsanleitung zur Selbsterfahrung:


Wenn Sie mit der PMR bisher noch keine Bekanntschaft gemacht haben, gehen Sie am besten wie
folgt vor:
Sorgen Sie dafür, dass Sie in den nächsten 20-30 Minuten nicht gestört werden und machen Sie es
sich im Sitzen oder Liegen bequem. Während des Übens konzentrieren Sie sich auf die angegebenen
Muskelgruppen, und spannen diese spürbar für fünf bis sieben Sekunden an. Ist dies wegen Schmer-
zen nicht möglich, so spannen Sie die entsprechende Muskelgruppe nur in Ihrer Vorstellung an.

Zählen Sie still mit – z. B. 1 – 5, dann entspannen Sie diese Muskelgruppe, indem Sie los-lassen.
Nach jedem Anspannen nehmen Sie sich 20 - 30 Sekunden Zeit und spüren in dieses Losgelassen–
haben, Entspannt-Sein, Locker-Sein ... hinein, nehmen es intensiv wahr und fördern es durch Affir-
mationen wie: „ganz ruhig, ganz locker...“ oder eine ähnliche, einfache und individuell für Sie pas-
sende Formulierung.

Üben Sie mindestens einmal pro Tag diese 16 Muskelgruppen in der angegebenen Reihenfolge:

1. Dominante Hand und Unterarm,


2. Dominanter Oberarm,
3. Nicht dominante Hand und Unterarm,
4. Nicht dominanter Oberarm,
5. Stirn,
6. Obere Wangenpartie und Nase,
7. Untere Wangenpartie und Kiefer,
8. Nacken, Hals und Schultern,
9. Rücken,
10. Brust,
11. Bauchmuskulatur,
12. Dominanter Oberschenkel,
13. Dominanter Unterschenkel,

60
14. Dominanter Fuß,
15. Nicht dominanter Oberschenkel,
16. Nicht dominanter Unterschenkel,
17. Nicht dominanter Fuß.

Zumeist ist nach zwei bis vier Wochen die Entspannungsreaktion ausreichend konditioniert um die
Muskelgruppen in vier Bereiche zusammenzufassen. Dann können Sie nacheinander die Muskeln
der Arme, des Kopfes, des Rumpfes und der Beine anspannen und wieder entspannen. Noch später
können Sie die Bereiche in Zahlen überführen und dann, von 1 - 4 denkend, die betreffenden Be-
reiche entspannen. Die PMR ist ausgezeichnet mit entspannender und ruhiger Musik, Imaginationen,
Visualisierungen oder Phantasiereisen kombinierbar.

Literatur:

Bandura A (1977) Self-efficacy: Toward a unifying theory of behavior change. Psychological Review 84: 191–215
Chida, Y., Hamer, M., Wardle. J, & Steptoe, A. (2008). Do stress-related psychosocial factors contribute to cancer inci-
dence an survival? Nat Clin Pract oncol., 5(8), 466-75.
Faller, H. (2001). Krankheitsbewältigung und Überlebenszeit bei Krebskranken. Psychotherapeut, 46, 20-35.
Faller, H. (2004). Beeinflussen psychologische Faktoren den Verlauf einer Krebserkrankung? Ergebnisse, Methoden, Me-
chanismen. Z Med Psychol, 13, 99-108.
Faller, H. (2005). Wirksamkeit psychosozialer Interventionen. In H. Faller (Hrsg.) Psychotherapie bei somatischen Erkran-
kungen (S. 199-210). Stuttgart, New York: Georg Thieme Verlag.
Greer. S. (2006). Description and Effectiveness of Psychotherapy for Cancer Patients. S. 152-162. In: P. Herschbach, P.
Heußner & Sellschopp, A. (Hrsg.) Psycho-Onkologie. Pabst Science Publishers: Lengerich
Heußler, P. & Riedner, C. (2005). Psycho-sozialer Distress als Begleitsymptom der Krebserkrankung. Dtsch Med Woch-
enschr, 130, 2155-2157.
Jimmie, C. & Holland, M.D. (2002). History of Psycho-Oncology: Overcoming Attitudinal and Conceptual Barries. Psy-
chosomatic Medicine, 64, 206-211.
Lazarus, R. S., Folkman, S. (1984). Stress, Appraisal and Coping. New York: Springer
Ledesma, D & Kumano, H. (2009). Mindfulness-based stress reduction and cancer: a meta analysis. Psychooncology,
18(6), 571-9.
Moorey, S. & Greer, S. (2002). Cognitive Behaviour Therapy for People with Cancer. Oxford: Oxford University Press.
Rotter JB (1966) Generalized expectancies for internal versus external control reinforcement. Psychological Monographs
80: 1–28
Schwarz, R. (2005). Psychotherapie bei Krebs. In H. Faller (Hrsg.) Psychotherapie bei somatischen Erkrankungen (S. 168-
183). Stuttgart, New York: Georg Thieme Verlag.
Sellschopp, A., Herschbach, P., Keller, M. & Ravens-Sieberer, U. (2001). Psychoonkologie und postoperative Lebensqua-
lität. Manual Gastrointestinale Tumoren. Tumorzentrum München.
Spencer, R., Nilsson, M., Wright, A., Pirl, W. & Prigerson, H. (2010). Anxiety disorders in advanced cancer patients.
Cancer, 116(7), 1810-1819.
Tschuschke, V. (2003). Psychologisch-psychotherapeutische Interventionen bei onkologischen Erkrankungen. Psychothe-
rapeut, 48, 100-108.
Weis, I. & Wechsung, P. (2002). Psychoonkologische Interventionen. Onkologe, 8(1), 71-72.
Weiß, J. & Heckl, U. (2005). Psychotherapie chronisch körperlich Kranker aus Sicht der Verhaltenstherapie. In H. Faller
(Hrsg.) Psychotherapie bei somatischen Erkrankungen (S. 43-56). Stuttgart, New York: Georg Thieme Verlag.

61
Supervision als Präventions- bzw. Interventionsmaßnahme bei
Belastungen am Arbeitsplatz Krankenhaus
Dr in. Brigitte Matschi, Mag. Lars Larsen

Zahlreiche Untersuchungen bestätigen, dass Krankenhauspersonal aller Berufsgruppen an ihren Ar-


beitsplätzen großen physischen und psychischen Dauerbelastungen ausgesetzt ist.
Folgende Belastungsbereiche werden in diesem Zusammenhang durch verschiedene Studienergeb-
nisse bestätigt (Arbeitsklima-Index Gesundheitsberufe 2006, Beschoner & Braun,2007):
• erhöhter Zeitdruck
• seelisch belastende und aufreibende Arbeit
• zu kurze Erholungsphasen nach Nacht- und Wochenenddiensten
• große Unzufriedenheit mit der Arbeitszeitregelung, dem betrieblichen Klima und den Mitbe-
stimmungsmöglichkeiten
• geringe Vereinbarkeit von Beruf, privaten Pflichten und Interessen
• ungünstige Arbeitszeitbedingungen, mangelnde Berufserfahrung und medizinische Verant-
wortung für viele moribunde PatientInnen (Herschbach, 1991)
• Arbeitsbelastung bei gleichzeitig geringer Möglichkeit, selbst Einfluss auf das therapeutische
Geschehen zu nehmen (Bauer et al., 2003)
• mangelnde Kooperation und Transparenz, zu geringe Gestaltungsfreiheit von Aufgaben so-
wie das Arbeitsklima (Werner & Hennch, 1995).
Daraus ergibt sich u.a.
• hohe Suizidhäufigkeit bei Psychiatern und Psychotherapeuten (Reimer et al., Jahr?)
• risikobehafteter Umgang mit Alkohol (Braun, 2007)
• Suchtmittelgefährdung oder Suchtmittelabhängigkeit (Mäulen, 2005).
Supervision (und Coaching) wird als Präventions- bzw. Interventionsmaßnahme zur Verhinderung
von Burnout, affektiven Störungen und Suchtmittelmissbrauch - bedingt durch Belastungen am Ar-
beitsplatz – neben Veränderungen in der Organisation empfohlen.
Am Universitätsklinikum der Paracelsus Medizinischen Privatuniversität (früher St. Johanns-Spital)
wurde bereits im Jahr 1990 Supervision für alle besonders belasteten MitarbeiterInnen aller Berufs-
gruppen in Form eines Forschungsprojektes eingeführt. Auf Grund guter Erfahrungen damit wurde
im Jahr 2000 am Universitätsinstitut für Klinische Psychologie die Koordinationsstelle für Supervision
eingerichtet. Alle Ansuchen um Supervision von MitarbeiterInnen des Landeskrankenhauses und der
Christian-Doppler-Klinik werden vom Leiter der Koordinationsstelle für Supervision in Hinblick auf
Indikation für eine der angebotenen Supervisionsformen geprüft. Ausschließlich externe, d.h. institu-
tionsunabhängige SupervisorInnen übernehmen dann die Durchführung von Supervision, Coaching
oder arbeitsplatzbezogener Krisenintervention. So wurden z.B. seit 1990 am Landeskrankenhaus
4.340 Einheiten (á 45 Minuten) Teamsupervision, seit 1995 2.000 Einheiten Coaching und seit dem
Jahr 2000 1.120 Einheiten arbeitsplatzbezogene Krisenintervention durchgeführt.
In österreichischen Krankenanstalten wurde 1993 im § 11c des Krankenanstaltengesetzes, BGBl Nr.
801/1993 Supervision gesetzlich geregelt:
§ 11c. „Die Landesgesetzgebung hat die Träger der nach Anstaltszweck und Leistungsangebot in
Betracht kommenden Krankenanstalten zu verpflichten, durch geeignete Maßnahmen sicher zu stel-
len, dass den in den Krankenanstalten Beschäftigten und einer entsprechenden Belastung ausgesetz-
ten Personen im Rahmen ihrer Dienstzeit im erforderlichen Ausmaß Gelegenheit zur Teilnahme an
einer berufsbegleitenden Supervision geboten wird. Die Supervision ist durch fachlich qualifizierte
Personen auszuüben.“

62
Was ist Supervision?
Supervision ist eine feldspezifische Beratungsmethode zur Sicherung und Verbesserung der Qualität
der beruflichen Arbeit. Supervision hat sich als Beratungsform in Profit- als auch in Not-for-Profit-
Organisationen mit dem Ziel einer methodischen Reflexion des beruflichen Handelns und der Ar-
beitssituation sowie der Verbesserung der Kooperation und Kommunikation etabliert.
Im Verlauf einer Supervision wirken zahlreiche Faktoren auf individueller, interaktioneller und orga-
nisatorischer Ebene ein. Die bedeutendsten Faktoren sind:
Die Reflexivität: die SupervisandInnen betrachten gemeinsam unter Anleitung eines Supervisors ver-
gangene Abläufe im Arbeitsprozess, im Kontakt zu KlientInnen oder die PatientInnen, planen und
entwickeln gemeinsam und konstruktiv Änderungen von Arbeitsabläufen, Beziehungen usw.
Das Feedback: in einer vertraulichen und wertschätzenden Atmosphäre erhalten die TeilnehmerInnen
Rückmeldungen über ihr Verhalten, ihre Arbeitsweise, ihre Beziehungen zueinander, ihre Kommuni-
kationsmuster mit dem Ziel, die Arbeitsbeziehung und die Kooperation zu verbessern.
Spiegel- oder Resonanzphänomene: Darunter werden jene (spiegelgleichen) Wiederholungen von
Konflikten und Problemen in der Supervisionsgruppe verstanden, die den TeilnehmerInnen aus ihrer
praktischen Arbeit bekannt sind. Der Supervisor hält den TeilnehmerInnen gleichsam einen Spiegel
vor, in dem diesen ihr Verhalten, das am Arbeitsplatz oft zu Problemen in der Zusammenarbeit führt,
in der Supervisionsgruppe bewusst wird. Durch die Distanz zum Arbeitsplatz, die in der Supervisions-
gruppe gegeben ist und unter Einhaltung der Spielregeln (Supervisionskontrakt) unter der Führung
und Anleitung des Supervisors gelingt es den Supervisanden im „Idealfall“, destruktives Verhalten zu
erkennen und gemeinsam konstruktive Lösungen zu entwickeln und auch zu üben.

Supervisionsformen bzw. Supervisionsettings


Abhängig von
• der Organisation, in der Supervision stattfinden soll,
• der Arbeitsbeziehung der SupervisandInnen (so werden die TeilnehmerInnen an der Supervi-
sion genannt)
• und den Zielen des Auftraggebers für Supervision
wird aus verschiedenen Supervisionsformen nach ausführlichen Gesprächen mit dem Auftraggeber
und den SupervisandInnen das geeignete Setting ausgewählt.
Folgende Supervisionsformen werden praktiziert:
• Einzelsupervision: reflektiert und beeinflusst werden eigene Bewältigungsmuster (therapeuti-
scher) Arbeit, entstandenes diffuses Unbehagen und Unvermögen, Zusammenhänge ange-
messen zu erfassen und adäquat zu handeln. In dieser geschützten Atmosphäre soll die be-
rufliche Identität gestützt werden auch unter Einbeziehung der ganzheitlichen Entwicklung vor
dem Hintergrund der eigenen Lebensgeschichte.

• Gruppensupervision: Mitglieder einer oder mehrerer Berufsgruppen finden sich unter Anlei-
tung einer SupervisorIn zusammen mit dem Ziel der grundsätzlichen Erweiterung der berufli-
chen Kompetenzen und Fähigkeiten. Reflektiert wird die berufliche Praxis wie Entlastung bei
beruflichen Problemen, Psychohygiene, Hilfestellung bei schwierigen Situationen und Kon-
flikten, Aufzeigen von blinden Flecken und Transparentmachen von Zusammenhängen.

• Teamsupervision: Mitglieder eines Arbeitsteams richten gemeinsam den Fokus auf die inter-
nen Arbeitsbeziehungen und auf organisatorische Probleme im Team als Teil einer Organi-
sation.

63
• Coaching: Mit Coaching wird (auf Führungsebene) eine Kombination aus individueller Be-
ratung, persönlichem Feedback und praxisorientiertem Training bezeichnet (Fischer-Epe,
2011). Dabei können Fragestellungen behandelt werden, die sowohl die beruflichen Aufga-
ben und Rollen wie auch die Persönlichkeit des Klienten betreffen. Die Themen reichen von
Führungsfragen über Rollenklärungen, Motivationsthemen, Konfliktlösungsstrategien, eigene
Gesunderhaltung bis hin zur Karriereplanung (Fischer-Epe, 2011). Grundsätzlich handelt es
sich bei Coaching um eine Form der Prozessberatung und –begleitung, es geht nicht darum,
seitens des Coachs Lösungen vorzugeben, vielmehr bietet Coaching professionelle Reflexi-
onshilfe mit dem Ziel, eigenständig Handlungsalternativen und Lösungen zu entwickeln.

• Intervision: Intervision ist eine beratende Tätigkeit (u.a. Fallsupervision) unter hierarchisch
Gleichrangigen oder über ein „hierarchisches Gefälle“ hinweg.

• Balintgruppe: Dem ungarischen Arzt und Psychoanalytiker Michael Balint (1896 bis 1970)
war es ein ganz besonderes Anliegen, ÄrztInnen für eine ganzheitliche, über die Organme-
dizin hinausgehende Sichtweise zu gewinnen, indem die seelische, die körperliche, aber auch
die kognitive und handlungsbezogene Dimension der Krankheit betrachtet wird. In einer Bal-
intgruppensitzung stellen TeilnehmerInnen in einem „freien“ Fallbericht PatientInnen vor. An-
schließend teilen die anderen Gruppenmitglieder ihre Einfälle und ihre gefühlsmäßigen Re-
aktionen auf diese Fallschilderung mit. Gemeinsam mit der GruppenleiterIn, welche ausrei-
chend tiefenpsychologische, psychotherapeutische Kenntnisse und Selbst- und Gruppener-
fahrung haben soll, entsteht so auf Grund der freien Assoziationen und der Reaktionen der
Gruppe ein neues Bild der vorgestellten PatientInnenInnen und eine neue Problemdefinition.
Es ist Aufgabe der BalintgruppenleiterInnen, die unbewussten Beziehungen im Sinne von
Übertragung und Gegenübertragung zwischen Arzt und Patient und ihren Einfluss auf die
Behandlung den TeilnehmerInnen bewusster zu machen. Die BalintgruppenleiterIn ermuntert
die TeilnehmerInnen, mit dem „dritten Ohr“ (Reik) nicht Gesprochenes hören zu können,
denn erst mit dieser Haltung können somatische Symptome im Zusammenhang mit seeli-
schen Problemen der PatientInnen voll und ganz begriffen werden. Balintgruppen werden
heute außer für ÄrztInnen auch für PsychotherapeutInnen, PsychologInnen, Krankenpflege-
personal, LehrerInnen, SozialarbeiterInnen, SeelsorgerInnen und für RichterInnen angeboten.

Voraussetzungen für eine erfolgreich verlaufende Supervision


Von der SupervisorIn wird neben einer fundierten Ausbildung auch Subjekt-, Gruppen-, Beratungs-,
Prozess-, Organisations- und Feldkompetenz erwartet und dass sie/er:
- in der Supervisionsgruppe eine Atmosphäre herstellt, die einer konstruktiven gemeinsamen
Arbeit dienlich ist
- aktiv leitet und den Verlauf der Gruppensitzung strukturiert
- TeilnehmerInnen dann beschützt, wenn dies notwendig ist, die SupervisandInnen ermutigt,
neues Verhalten zu erproben, Grenzen setzt und diese auch aufzeigt
- auftretende Emotionen, Gedanken und Handlungsimpulse ordnet
- allparteilich ist und die Einhaltung der zu Beginn mit allen TeilnehmerInnen vereinbarten
Spielregeln gewährleistet (Supervisionskontrakt).

Von den SupervisandInnen (den TeilnehmerInnen an der Supervisionsgruppe) wird erwartet, dass sie:
- an der Supervision freiwillig teilnehmen und
- dass sie die Bereitschaft zur Veränderung mitbringen.

64
Literatur:
Beschoner, P. & Braun, M. (2007). Das Burnout-Syndrom bei ÄrztInnen in Deutschland. Nervenheilkunde 3/2007.
Schattauer GmbH.
Braun, M., Freudenmann, R. W., Schönfeldt-Lecuona, C. & Beschoner, P. (2007). Burnout, Depression und Substanzge-
brauch bei ÄrztInnen. Ein Überblick zur derzeitigen Datenlage. Psychoneuro, 33(1+2), 19–22.
Werner, A. & Hennch, C. (1995) Stationsteam-Supervision in der medizinischen Klinik. Abschlussbericht des Pilotprojektes.
Klinikum der Universität Heidelberg, Medizinische Klinik. Heidelberg.
Widauer, H. (1997) Kann Supervision im Krankenhaus als gesundheitsfördernde Maßnahme einen Beitrag leisten? In A.
Grundböck (Hrsg.), Gesundheitsförderung – eine Strategie für Krankenhäuser im Umbruch. Projekte aus Österreich und
Deutschland. Wien: Facultas-Universitäts-Verlag.

65
Sexuelle Gewalt bei Kindern und Jugendlichen:
Erkennen, anzeigen und vorbeugen
Maga. Sigrun Eder und Maga. Dr in. Corinna Fritz

Strafbare Handlungen gegen die sexuelle Integrität und Selbstbestimmung in Österreich werden in
der Kriminalstatistik erfasst. Im ersten Halbjahr 2018 wurde „Schwerer sexueller Missbrauch von
Unmündigen“ 2 (§ 206 StGB) in 195 Fällen zur Anzeige gebracht. Im Vergleich dazu wurde „Sexueller
Missbrauch von Unmündigen“3 (§ 207 StGB) in 169 Fällen angezeigt. Es wurden 52 Kinder unter 6
Jahre, 40 Kinder von 6 bis unter 9 Jahren und 101 Kinder zwischen 10 und 13 Jahren sowie 2
zwischen 14 und 17 Jahren im ersten Halbjahr 2018 Opfer eines schweren sexuellen Missbrauchs
(§ 206 StGB). Hinsichtlich § 207 (Sexueller Missbrauch von Unmündigen) zählte die Altersgruppe
der 10 bis unter 13-jährigen zu den Hauptbetroffenen (n= 68).

Sexuelle Gewalt findet im Verborgenen statt und wird deutlich weniger oft zur Anzeige gebracht als
sie von Kindern und Jugendlichen erlebt wird. Auffallend ist, dass vorwiegend bei sexuellen Grenz-
überschreitungen außerhalb der Familie die Erstattung einer Anzeige erfolgt. Schätzungen der WHO
zufolge erleben weltweit jährlich 150 Millionen Mädchen und 73 Millionen Buben sexuelle Gewalt.
Statistisch wird angenommen, dass in Österreich jedes vierte Mädchen und jeder siebte Bub unter
16 Jahren sexuell missbraucht wird. Die meisten Kinder sind zu Beginn des Missbrauchs zwischen 6
und 12 Jahre alt. An zweiter Stelle folgt die Altersgruppe der 0 bis 6-jährigen, an dritter Stelle die
der über 14-jährigen.

Sexueller Missbrauch ist interpersonelle Gewalt. Sie erfolgt, wenn die zwischenmenschliche Begeg-
nung eine sexuelle Handlung beinhaltet, von einer Abhängigkeitsbeziehung sowie von ungleichen
Machtverhältnissen zwischen Opfer und gewaltausübender Person gekennzeichnet ist. Zusätzlich ist
das Opfer einem großen Geheimhaltungsdruck ausgesetzt, der durch wiederkehrende Drohungen
aufrechterhalten wird. Die Drohungen zielen meist auf das Infrage stellen der Unversehrtheit geliebter
Menschen des Kindes ab. Auch wird dem Opfer vermittelt, dass es bei Aufdeckung der sexuellen
Gewalt selbst von geliebten Dingen getrennt wird. Sexuelle Gewalt beginnt dort, wo das kindliche

2 (1) Wer mit einer unmündigen Person den Beischlaf oder eine dem Beischlaf gleichzusetzende geschlechtliche

Handlung unternimmt, ist mit Freiheitsstrafe von einem bis zu zehn Jahren zu bestrafen. (2) Ebenso ist zu bestrafen,
wer eine unmündige Person zur Vornahme oder Duldung des Beischlafs oder einer dem Beischlaf gleichzusetzen-
den geschlechtlichen Handlung mit einer anderen Person oder, um sich oder einen Dritten geschlechtlich zu erre-
gen oder zu befriedigen, dazu verleitet, eine dem Beischlaf gleichzusetzende geschlechtliche Handlung an sich
selbst vorzunehmen. (3) Hat die Tat eine schwere Körperverletzung (§ 84 Abs. 1) oder eine Schwangerschaft der
unmündigen Person zur Folge, so ist der Täter mit Freiheitsstrafe von fünf bis zu fünfzehn Jahren, hat sie aber den
Tod der unmündigen Person zur Folge, mit Freiheitsstrafe von zehn bis zu zwanzig Jahren oder mit lebenslanger
Freiheitsstrafe zu bestrafen. (4) Übersteigt das Alter des Täters das Alter der unmündigen Person nicht um mehr
als drei Jahre, besteht die geschlechtliche Handlung nicht in der Penetration mit einem Gegenstand und hat die Tat
weder eine schwere Körperverletzung (§ 84 Abs. 1) noch den Tod der unmündigen Person zur Folge, so ist der
Täter nach Abs. 1 und 2 nicht zu bestrafen, es sei denn, die unmündige Person hätte das 13. Lebensjahr noch nicht
vollendet.

3 (1) Wer außer dem Fall des § 206 eine geschlechtliche Handlung an einer unmündigen Person vornimmt oder
von einer unmündigen Person an sich vornehmen lässt, ist mit Freiheitsstrafe von sechs Monaten bis zu fünf Jahren
zu bestrafen. (2) Ebenso ist zu bestrafen, wer eine unmündige Person zu einer geschlechtlichen Handlung (Abs.
1) mit einer anderen Person oder, um sich oder einen Dritten geschlechtlich zu erregen oder zu befriedigen, dazu
verleitet, eine geschlechtliche Handlung an sich selbst vorzunehmen.(3) Hat die Tat eine schwere Körperverletzung
(§ 84 Abs. 1) zur Folge, so ist der Täter mit Freiheitsstrafe von einem bis zu zehn Jahren, hat sie aber den Tod der
unmündigen Person zur Folge, mit Freiheitsstrafe von fünf bis zu fünfzehn Jahren zu bestrafen.(4) Übersteigt das
Alter des Täters das Alter der unmündigen Person nicht um mehr als vier Jahre und ist keine der Folgen des Abs.
3 eingetreten, so ist der Täter nach Abs. 1 und 2 nicht zu bestrafen, es sei denn, die unmündige Person hätte das
zwölfte Lebensjahr noch nicht vollendet.

66
Vertrauen missbraucht wird, um inadäquate sexuelle Bedürfnisse der gewaltausübenden Person zu
stillen. Sexueller Missbrauch wird somit als die Einbeziehung und Nötigung von Kindern oder Ju-
gendlichen zu sexuellen Aktivitäten verstanden, die sie aufgrund entwicklungsbedingter Unreife nicht
vollständig erfassen können, bei denen sie außerstande sind bewusst einzuwilligen und wobei soziale
Grenzen der Familie bzw. der Gesellschaft erheblich verletzt werden.

Sexuelle Gewalt umfasst eine Vielzahl von grenzverletzendem Verhalten: Kitzelspiele mit sexueller
Komponente; Geschlechtsteile zeigen, sehen und berühren wollen; Masturbation in Anwesenheit des
Kindes; Zeigen und aufnehmen pornographischer Filme und Fotos; Beobachten des Kindes beim
Ausziehen und Waschen; Vollzug des Geschlechtsverkehrs in Anwesenheit des Kindes; Sich reiben
am Körper des Kindes; Zwang des Kindes zur oralen Befriedigung; orale, anale, vaginale Vergewal-
tigung mit Finger, Penis oder einem Gegenstand. Bei 85 bis 95 Prozent der Opfer sexueller Gewalt
finden sich körperliche Normalbefunde. Denn die Häufigkeit pathologischer Befunde hängt entschei-
dend von der Art und dem Ablauf des Übergriffs ab. Das Fehlen medizinischer Befunde schließt
jedoch einen zurückliegenden sexuellen Übergriff in keinster Weise aus.

Missbrauch kann innerhalb der Familie, außerhalb der Familie durch eine dem Kind vertraute Person
sowie durch eine dem Kind fremde Person geschehen. Innerhalb der Familie hat sexuelle Gewalt
meist eine lange Geschichte und weist auf gestörte familiäre Beziehungen, starke Abhängigkeiten,
Isolation und Sprachlosigkeit zwischen Eltern und Kindern hin. Sexuelle Ausbeutung passiert über-
wiegend im sozialen Nahbereich. Mädchen werden zu einem Drittel von Personen aus der Familie
missbraucht (z.B. Väter, Stiefväter, Brüder, Mütter, im Haushalt lebende Großväter). Der größte Teil
jedoch kommt aus dem außerfamiliären Bereich (Verwandte, männliche Jugendliche). Buben erfah-
ren meist von Bezugspersonen im außerfamiliären Nahraum (u.a. Bekannte, Trainer, Pädagogen)
sexuelle Gewalt.

Sexuelle Gewalt wird bekannt, wenn Kinder und Jugendliche darüber sprechen, eine Drittperson
Übergriffe beobachtet oder Verhaltensveränderungen beim Kind sichtbar werden. Vertrauensperso-
nen, die von missbräuchlichen Handlungen in Kenntnis gesetzt werden, sollen die Ruhe bewahren,
keine das Vertrauen einschränkende falschen Versprechen leisten („Ich behalte dein schlechtes Ge-
heimnis für mich“, „Ich passe auf dich auf“, „Du musst nie mehr in die Familie zurück“) und das Kind
nicht zu den Vorfällen im Detail befragen, weil das im Falle einer Anzeige die Qualität der Aussage
erheblich mindert. Für das Setzen weiterer Schritte sind immer Experten aus dem Kinderschutz (z.B.
Kinderschutzzentrum) hinzuzuziehen. Im Krankenhaus sollen Verdachtsfälle zur Aufdeckung von Ge-
walt an Kindern und zur Einleitung von adäquaten Schutzmaßnahmen unbedingt in die interne Kin-
derschutzgruppe 4 (KSG) eingebracht werden. Die Kinderschutzgruppe ist ein beratendes, multipro-
fessionelles Gremium (Ärzte, Sozialarbeiter, Psychologen), welches im Auftrag des zuweisenden sta-
tionsführenden Arztes Anamnese und Befunde erhebt, eine Diagnose stellt, eine Einschätzung der
Gesamtsituation erarbeitet und einen abschließenden Befundbericht abgibt.

Bei der Wahrnehmung und Bewertung von Veränderungen im Zusammenhang mit Verdacht auf
sexuelle Gewalt ist die Berücksichtigung des individuellen Entwicklungsstandes sowie die Persönlich-
keit des betroffenen Kindes und Jugendlichen erforderlich. Darüber hinaus ist es von zentraler Be-
deutung zu wissen, dass es kein eindeutiges „Missbrauchs-Syndrom“ gibt. Denn viele der Verände-
rungen, die Kinder im Erleben und Verhalten zeigen, können auch in Zusammenhang mit anderen
Belastungen, wie z.B. Trennung/Scheidung der Eltern, Konflikte in der Familie und/oder Gleichaltri-
gen-Gruppe, schulischer Leistungsdruck stehen. Rötungen, Wundsein, Entzündungen, Biss-Spuren,

4 Grundsatzbestimmung im Krankenanstalten- und Kuranstaltengesetz (KAKuG 2004)

67
blaue Flecken im Genital-, Anal-, Mund- oder Halsbereich, Geschlechtskrankheiten, Bauchschmer-
zen oder Schwangerschaft können als körperliche Symptome bei Opfern sexueller Gewalt auftreten.
Ebenso vermitteln Kinder und Jugendliche durch Einnässen, Einkoten, Ess-Störungen, Suchtverhal-
ten, Schlafstörungen einen hohen Problemdruck. Psychische Symptome sind feststellbar, wenn Kinder
und Jugendliche stark an Bezugspersonen klammern, Ängste, Zwänge, Depressionen, Sprachstörun-
gen, Rückzugsverhalten entwickeln oder in der Schule in ihrer Leistungs- und Konzentrationsfähigkeit
reduziert sind. Oftmals wird bei Betroffenen aggressives Verhalten, eine sexualisierte Sprache, sexu-
alisiertes Verhalten (Imitation von Geschlechtsverkehr oder intimen Berührungen) Selbstverletzung
oder Selbsttötungsabsichten beobachtet. Es empfiehlt sich, im Einzelfall über wahrgenommene Auf-
fälligkeiten mit PsychologInnen aus Opferschutzeinrichtungen zu sprechen, da jene Institutionen bei
Verdacht auf sexuelle Gewalt spezifische Anlaufstellen sind und/oder die Kinderschutzgruppe im
Krankenhaus zu involvieren.

Die Notwendigkeit einer forensischen Befund- und Spurensicherung ist im Einzelfall zu entscheiden.
Diese soll nur vorbereitet und im Einverständnis des Kindes durchgeführt werden. Dabei ist jede Form
von Druck, gewaltsames Festhalten oder massives Überreden streng kontraindiziert. Die ärztliche
Untersuchung ist qualifiziert und einfühlsam durchzuführen, ansonsten kann sie zu einem traumati-
schen Erlebnis (potenzielle Reviktimisierung) werden. Sie dient hauptsächlich der Beurteilung des
Anogenitalbefundes, obwohl sich die somatische Untersuchung aus der Erhebung des Allgemeinsta-
tus und des Genitalstatus zusammensetzt. Da es typischerweise zu einer verzögerten ärztlichen Vor-
stellung kommt, ist ein Großteil der Verletzungen bereits abgeheilt. Um biologische Spuren allerdings
erfolgreich nachweisen zu können, soll bei einem kurz zurückliegenden Ereignis (bei präpubertären
Kindern etwa innerhalb von 24, bei Pubertierenden etwa 72 Stunden) und bei blutenden Verletzun-
gen eine sofortige ärztliche Vorstellung stattfinden. Zur Beurteilung medizinischer Befunde bei Ver-
dacht auf sexuelle Gewalt ist es essentiell, den Normalzustand in den verschiedenen Altersstufen zu
kennen, um Fehlschlüsse zu vermeiden. Trotz medizinischer Befunderhebung basiert die Diagnose
primär auf einer qualifiziert erhobenen Aussage des Kindes.

In Österreich sind Buben und Mädchen ab 14 Jahren sexualmündig. Das bedeutet, dass sexuelle
Handlungen unter dieser Altersgrenze zwischen Erwachsenen und Kindern als sexuelle Gewalt be-
handelt werden. Bei Jugendlichen über 14 Jahren gestaltet sich die Bewertung, ob das, was stattge-
funden hat tatsächlich sexuelle Gewalt ist, diffiziler. Diese Altersgruppe hat die schwierige Aufgabe,
mit aller Deutlichkeit aufzuzeigen, dass die erlebten sexuellen Handlungen mit älteren Personen nicht
im Einvernehmen stattgefunden haben. Bei Jugendlichen, die zwischen 14 und 16 Jahre alt sind und
eine Beziehung mit einem erheblich älteren Partner/einer Partnerin eingehen, könnten sexuelle
Handlungen dann als sexuelle Gewalt darstellen, wenn die ältere Person die Unreife der jugendli-
chen Person ausnützt.

Sexuelle Gewalt ist eine Straftat, die zur Anzeige gebracht werden soll. Doch es ist von größter Not-
wendigkeit im Einzelfall abzuwägen, inwiefern die Erstattung der Anzeige – gerade bei fehlenden
oder unzureichenden Sachbeweisen – sinnvoll ist. Schließlich sind damit für das betroffene Kind oder
den Jugendlichen vielseitige Belastungen (Exekutivbeamte über sexuelle Gewalt berichten, Einver-
nahme bei Gericht, lange Verfahrensdauer) und die Gefahr der Re-Traumatisierung verbunden. Da-
her soll unbedingt bei Erwägung einer Anzeige mit einer Opferschutzstelle (z.B. Kinderschutzzentrum,
Weißer Ring, Gewaltschutzzentrum) Kontakt aufgenommen werden.

68
Ergibt sich für einen Arzt in Ausübung seines Berufes der Verdacht, dass ein Minderjähriger 5 miss-
handelt, gequält, vernachlässigt oder sexuell missbraucht worden ist, so hat 6 der Arzt Anzeige an die
Sicherheitsbehörde zu erstatten. Richtet sich der Verdacht gegen einen nahen Angehörigen (§ 166
StGB), so kann die Anzeige solange unterbleiben, als dies das Wohl des Minderjährigen erfordert
und eine Zusammenarbeit mit dem Jugendamt oder gegebenenfalls eine Einbeziehung einer Kin-
derschutzeinrichtung (Kinderschutzgruppe, Kinderschutzzentrum) erfolgt. Bei Verdacht auf Misshand-
lung, Vernachlässigung oder sexueller Gewalt haben Angehörige eines medizinischen Gesundheits-
berufes, die aufgrund berufsrechtlicher Vorschriften zur Verschwiegenheit verpflichtet sind, dem Ju-
gendwohlfahrtsträger Meldung 7 (Mitteilung „Gefährdung des Kindeswohls“) zu erstatten, sofern dies
zur Verhinderung einer weiteren erheblichen Gefährdung des Kindeswohls erforderlich ist. Für eine
Anzeige spricht, dass die gewaltausübende Person weggewiesen oder verhaftet und dadurch die
sexuelle Gewalt gestoppt wird. Auch können durch die Aufdeckung und Anzeigeerstattung potenzi-
elle Opfer besser geschützt werden. Für Betroffene stellt der Weg zu Gericht eine Möglichkeit dar,
sich zu wehren und die Gesellschaft für das Auftreten sexueller Gewalt zu sensibilisieren. Gegen eine
Anzeige spricht die geringe Verurteilungsquote, die lange Verfahrensdauer, mehrere Einvernahmen
durch fremde Personen in einer fremden Umgebung und der Umstand, dass immer die betroffene
Person selbst (ab dem 4. Lebensjahr) – unabhängig davon, wer Anzeige erstattet hat – vor Gericht
erscheinen und aussagen muss. Für Betroffene ist es wünschenswert, sich selbst zum Zeitpunkt ihrer
Wahl für die Erstattung der Anzeige entscheiden zu können. Das erhöht die Qualität der Aussage,
auch kommen sie besser mit den Belastungen im Strafverfahren zurecht. Sexuelle Gewalt soll aus
psychologischer Sicht dann zur Anzeige gebracht werden, wenn sich das Opfer mit der Vorstellung
vor Gericht als Zeuge auszusagen identifizieren kann. Schließlich kann eine einmal erstattete Anzeige
nicht mehr zurückgezogen werden.

Keinesfalls sollen betroffene Kinder, Jugendliche und ihre Bezugspersonen alleine die schwere Ent-
scheidung zur Durchführung der Anzeigeerstattung treffen oder den Weg zur Exekutive und zu Ge-
richt antreten. Deshalb bieten Opferschutz-Einrichtungen eine sogenannte Prozessbegleitung an.
Diese ist ein kostenloses juristisches und psychosoziales Angebot für Opfer sexueller und/oder kör-
perlicher Gewalt, die eine Anzeige erwägen oder bereits erstattet haben.

Im Rahmen der Prozessbegleitung werden die Aufgaben zwischen der psychosozialen und der juris-
tischen Prozessbegleitung verteilt, die in ständiger Kooperation miteinander stehen.

Die Prozessbegleitung beinhaltet u.a.:


• Information zu Abläufen bei Exekutive und Gericht,
• Akteneinsicht,
• Begleitung zu Exekutive und Gericht,
• juristische Vertretung vor Gericht,
• Privatbeteiligtenanschluss,
• Koordination der notwendigen Schritte und Aufgaben mit den befassten Stellen sowie Nach-
betreuung bei Gerichtsverfahren.

Die Inanspruchnahme einer Prozessbegleitung ist unbedingt zu empfehlen, da Betroffene im Ge-


richtsverfahren enormen Belastungen ausgesetzt sind. Es handelt sich um: das lange Warten zwi-
schen der Anzeigeerstattung und der Ladung zur Befragung bei Gericht, die Sorge des Kindes die
beschuldigte Person könnte die Drohungen wahrmachen, es selbst würde vor Gericht als Lügner

5 Ein Minderjähriger ist eine Person, die das 18. Lebensjahr noch nicht vollendet hat. Hat eine Person das 14.
Lebensjahr noch nicht erreicht, so ist sie unmündig. Ein Minderjähriger, der das 14., aber noch nicht das 18. Le-
bensjahr erreicht hat, ist ein Jugendlicher.
6 Ärztegesetz
7 Jugendwohlfahrtsgesetz

69
dastehen oder könne eine Frage nicht verstehen, die Aussicht möglicherweise zusätzlich durch eine
sachverständige Person zur Sammlung weiterer Beweise begutachtet zu werden sowie die Unsicher-
heit über die Dauer und den Ausgang des Verfahrens.

Damit eine Person Prozessbegleitung in Anspruch nehmen kann, muss sie „Opfer“ im Sinne der
Strafprozessordnung sein. Aber nicht alle Opfer haben Anspruch auf Prozessbegleitung, sondern nur
diejenigen, denen ein „Schaden am eigenen Körper oder aber an der Seele“ entstanden ist.

Dies ist dann der Fall, wenn …

• … eine Person durch eine vorsätzlich begangene Straftat Gewalt oder gefährlicher Drohung
ausgesetzt oder in ihrer sexuellen Integrität beeinträchtigt worden sein könnte (Schaden am
eigenen Körper, § 65, Z. 1 a, StPO) oder
• es sich um Ehegatten, Lebensgefährten, Verwandten in gerader Linie, Bruder oder Schwester
einer Person handelt, deren Tod durch eine Straftat herbeigeführt worden ist oder um andere
Angehörige, die Zeugen dieser Tat waren und daher besonders betroffen sind (Seelischer
Schaden, § 65, Z. 1 b, StPO).

Opfer können auch solche Personen sein, denen "nur" ein Schaden am Vermögen entstanden ist (§
65, Z. 1 c, StPO). Diese Personen haben keinen Anspruch auf Prozessbegleitung. Aber auch sie
können sich dem Verfahren als Privatbeteiligte (Verlinkung) anschließen. Im Einzelfall können sie
einen Antrag auf Verfahrenshilfe stellen und hätten so Anspruch auf Vertretung durch einen Anwalt/
eine Anwältin.

Wird sexuelle Gewalt zur Anzeige gebracht, haben Betroffene vor der Exekutive auszusagen und sich
vor Gericht als Zeuge (kontradiktorische oder schonende Befragung) befragen zu lassen. Bei Gericht
findet die Befragung zum Sachverhalt in einem Nebenraum des Verhandlungssaales statt. Zeugen
werden dort in Anwesenheit einer Vertrauensperson (Prozessbegleiter) von gerichtlich beeideten
Sachverständigen (Kinder <14 Jahre) oder vom zuständigen Richter/zuständigen Richterin (Kinder
>14 Jahre) befragt. Dieser Vorgang wird videoaufgezeichnet, um dies im Falle einer Hauptverhand-
lung verwenden zu können und dem Kind/Jugendlichen eine neuerliche Befragung zu ersparen.
Dazu muss jedoch das Kind seine Einwilligung geben. Kinder und Jugendliche erhalten auch eine
kindgerechte Wahrheitsbelehrung. Sollen sie gegen einen Angehörigen aussagen, dann können sie
von ihrem Entschlagungsrecht Gebrauch machen und sich einer Befragung entziehen.

Welche Handlungen sexueller Gewalt entsprechen ist klar definiert. Ebenso besteht die Möglichkeit
nach erlebter sexueller Gewalt eine Prozessbegleitung und/oder psychologische/therapeutische Un-
terstützungsangebote in Anspruch zu nehmen. Doch vor der Erinnerung an das Erlebte können Kin-
der und Jugendliche nicht bewahrt werden. Daher ist es von größter Bedeutung, dass Erwachsene
rechtzeitig die Scheu vor einer umfassenden und altersgerechten Sexualerziehung ablegen und Kin-
dern und Jugendlichen klare Präventivbotschaften vermitteln.
Die Präventivbotschaften lauten:
 Über deinen Körper bestimmst du allein.
 Gefühle sind wichtig, du kannst ihnen vertrauen.
 Es gibt angenehme und unangenehme Berührungen.
 Du hast das Recht, „nein“ zu sagen.
 Es gibt gute und schlechte Geheimnisse.
 Sprich darüber und suche Hilfe.
 Du bist nicht schuld.

70
Neben der Wissensvermittlung sollen Erwachsene und Bezugspersonen im Kontakt mit Kindern, die-
sen Aufmerksamkeit und Zuwendung geben, Vertrauen aufbauen und schenken, das Selbstvertrauen
stärken, die kindliche Unabhängigkeit fördern sowie das Selbstbewusstsein und die Durchsetzungs-
fähigkeit aktivieren. Zusätzlich soll verdeutlicht werden, dass unbedingter Gehorsam und Geheim-
haltungsverbote immer in Frage zu stellen sind.
Die genannten Bausteine sollen in der Erziehung eines jeden Kindes berücksichtigt werden, da sie es
robuster im Umgang mit schwierigen Situationen macht und zugleich die Gefahr reduziert, Opfer
sexueller Gewalt zu werden bzw. dass es übermäßig lange das Erleben von sexueller Gewalt ver-
schweigt.

Fallbeispiel I

Ein 17-jähriges Mädchen wird akut auf der Kinder- und Jugendpsychiatrie wegen selbstverletzendem
Verhalten und einer depressiven Episode aufgenommen. Sehr rasch berichtet das Mädchen von
mehrjährigen sexuellen Übergriffen (Berührungen über der Kleidung) durch den Pflegevater und ist
sich unsicher, ob sie Strafanzeige erstatten soll. Die Pflegemutter zweifelt an den Vorwürfen ihrer
Pflegetochter und erachtet sie als psychisch krank.

Welchen ersten Schritt setzen Sie?


a) Sie machen eine „Gefahr in Verzug“ Meldung beim Jugendamt.
b) Sie bringen den Fall in die Kinderschutzgruppe ein.
c) Sie erstatten Strafanzeige.
d) Sie stellen den Kontakt zum Kinderschutzzentrum her, um fallbezogene Informationen zur
Prozessbegleitung einzuholen.

Lösung b) Sie bringen den Fall in die Kinderschutzgruppe des Krankenhauses ein, um in diesem
beratenden Gremium die weitere Vorgangsweise zu erarbeiten.
Ergänzende Information: Schließlich handelt es sich bei der beschuldigten Person um einen nahen
Angehörigen des Mädchens, weshalb eine Strafanzeige solange unterbleiben kann, wie es dem Kin-
deswohl bei gleichzeitiger Einbeziehung Opferschutzeinrichtungen oder der Kinderschutzgruppe
dienlich ist.

Fallbeispiel II

Ein 13-jähriges Mädchen fällt montags in der Schule durch blaue Flecken an den Unterarmen,
Knutschflecken am Hals sowie durch unkonzentriertes und weinerliches Verhalten auf. Auf Drängen
der Freundin erzählt ihr das Mädchen, dass sie auf einer Party in stark alkoholisiertem Zustand Petting
zugestimmt, einen Geschlechtsverkehr mit ihrem 23-jährigen Schwarm wiederholt abgelehnt hat.
Jedoch habe der 23-jährige sie gegen ihren Willen festgehalten und vergewaltigt. Die Freundin
überredet das 13-jährige Mädchen Rat bei der Schulärztin einzuholen.

Zu welchen Maßnahmen wären Sie als Schularzt/Schulärztin verpflichtet?


a) Zur Erstattung einer Strafanzeige gegen den 23-jährigen.
b) Zu einer detaillierten Befragung des 13-jährigen Mädchens.
c) Zu einer Meldung an das Jugendamt.
d) Zur Weiterverweisung an das KISZ bzgl. Prozessbegleitung.

71
Lösung a) Gemäß dem Ärztegesetz haben Sie Strafanzeige gegen den 23-jährigen zu erstatten.
Ergänzende Information: Aus psychologischer Sicht ist für das Kindeswohl jedoch die Weiterverwei-
sung an das Kinderschutzzentrum wegen Prozessbegleitung günstiger, damit sich das Mädchen aus
freien Stücken für die Erstattung einer Anzeige entscheiden kann, wodurch die Belastungen im Ge-
richtsverfahren besser bewältigbar und die Qualität der Aussage erhöht werden würde.

Empfohlene Literatur

Fachbücher/Broschüren
BM für Wirtschaft, Familie und Jugend (2011). Gewalt gegen Kinder und Jugendliche. Leitfaden für die Kinderschutzarbeit
in Gesundheitsberufen. Wien: BM für Wirtschaft, Familie und Jugend.
Deegener, G. (2005). Kindesmissbrauch – erkennen, helfen, vorbeugen. Weinheim: Beltz.
Dettmeyer, H. & Banaschak, T. (2008). Kindesmisshandlung. Medizinische Diagnostik, Intervention und rechtliche Grund-
lagen. Heidelberger: Springer.
Eder, S. & Kettl, S. (2012). Lorenz wehrt sich. Hilfe für Kinder, die sexuelle Gewalt erlebt haben. Salzburg. edition rieden-
burg.
Enders, U. (2003). Zart war ich – bitter war´s. Handbuch gegen sexuellen Missbrauch. Köln: Kiepenheuer & Witsch.
Gründer, M.; Kleiner, R. & Nagel, H. (2010). Wie man mit Kindern darüber reden kann. Ein Leitfaden zur Aufdeckung
sexueller Misshandlung. Weinheim: Juventa.
Hermann, B.; Banaschak. S; Csorba, R.; Navratil, F. & Dettmeyer, R. (2014). Medizinische Diagnostik bei sexuellem Kin-
desmissbrauch. Konzepte, aktuelle Datenlage und Evidenz. Deutsches Ärzteblatt, 41, 692-703.

Bilderbücher
Janouch, K. & Lindman, M. (2005). Bevor ich auf die Welt kam. Wie Babys entstehen. Hamburg: Oetinger.
Lauer, K. & Bley, A. (2006). Das kummervolle Kuscheltier. München: arsEdition.
Mönter, P. & Wiemers, S. (1999). Küssen nicht erlaubt. Freiburg: KeRLE.
Wabbes, M. (2005). Ich dachte, du bist mein Freund. Kinder vor sexuellem Missbrauch schützen. Gießen: Brunnen.
Cole, B. (2005). Prinzessin Pfiffigunde. Hamburg: Carlsen.

Ansprechpartner

Meldestelle Kinderpornografie Weißer Ring


Währinger Straße 3/18, 1090 Wien Giselakai 43, 5020 Salzburg
meldung@stopline.at 0699/13434005

Kinderschutzzentrum Salzburg Gewaltschutzzentrum


Leonhard-von-Keutschach-Straße 4, 5020 Salzburg Paris-Lodron-Straße 3a, 5020 Salzburg
beratung@kinderschutzzentrum.at office@gewaltschutzsalzburg.at
0662/44911 0662/870100

Neustart Salzburg
Schallmooser Hauptstraße 38, 5020 Salzburg
office.salzburg@neustart.at
0662/650436

72
Suizidrisikoeinschätzung und Risikomanagement
Priv.-Doz. Mag. Dr. Martin Plöderl und Mag. Dr. Clemens Fartacek

Jährlich sterben weltweit ca. eine Million Menschen durch Suizid (WHO, 2014), das sind 1,4% aller
Todesfälle. Suizid ist altersübergreifend die 15-häufigste Todesursache und die zweithäufigste im
Alter von 15 bis 29 Jahren. In Österreich nahmen sich im Jahr 2017 insgesamt 1224 Personen das
Leben, in Deutschland starben 2016 insgesamt 9838 Personen durch Suizid. In Österreich sind das
13,7 von 100.000 Personen (Deutschland 11,9) und etwa drei Mal mehr Suizid- als Verkehrstote
(Statistik Austria, 2018). Suizidversuche werden auf das Zwanzigfache von Suiziden geschätzt. Vor
diesem Hintergrund wird suizidales Verhalten als globales und nationales Public-Health Problem
deutlich und der Aufruf zur Reduktion suizidalen Verhaltens durch spezifische Behandlungskonzepte
und Präventionsprogramme nachvollziehbar (WHO, 2014).
Der Suizidprävention stehen mittlerweile eine Reihe vielversprechender Behandlungskonzepte und
Präventionsstrategien zur Verfügung, die nachweislich suizidales Verhalten reduziert haben (für einen
Überblick siehe letzten Abschnitt). Eine große Herausforderung für die Suizidprävention stellt jedoch
die Suizidrisikoeinschätzung dar. Die statistische Seltenheit und die Komplexität der Entstehung eines
Suizides erschwert dessen Vorhersage. Dies könnte erklären, weshalb Suizidrisikofaktoren eine ge-
ringe Spezifität aufweisen: die meisten Personen suizidieren sich nicht, selbst wenn sie alle Risikofak-
toren tragen (Large, Ryan, & Nielssen, 2011). Für die Suizidrisikoeinschätzung bestehende Heuristi-
ken basieren auf Suizidmodellen, Suizidrisikofaktoren und Warnzeichen sowie Strategien in der Ge-
sprächsführung und im Beziehungsaufbau, worauf im Folgenden eingegangen wird.

Suizidmodelle
Zur Entstehung von Suiziden gibt es mittlerweile eine Vielzahl an Erklärungsmodellen, von denen hier
drei exemplarisch angeführt werden. Nach Mann, Waternaux, Haas, and Malone (1999) beginnen
suizidale Entwicklungen mit dem Einsetzen psychischer Erkrankungen oder anderen kritischen Leben-
sereignissen. In weiterer Folge resultiert suizidales Verhalten aus der Interaktion spezifischer dynami-
scher (Depressivität, Hoffnungslosigkeit, Suizidgedanken und Suizidpläne) und statischer Faktoren
(Impulsivität, Aggressivität und damit verbundene niedrige serotonerge Aktivität). Letztere setzen die
Hemmschwelle für suizidales Verhalten herab, sodass die Suizidpläne durchgeführt werden können.
Nach der Theorie des „Präsuizidalen Syndroms“ (Ringel 1953, 1984) ist der Zustand vor dem Suizid
durch Einengungen bezüglich der persönlichen Möglichkeiten (auch als situative Einengung bezeich-
net; z.B. Ohnmachtsempfinden, Verlust der Gestaltungs- und Entfaltungsmöglichkeiten, des Selbst-
wirksamkeitserlebens), der Gefühlswelt (auch als dynamische Einengung bezeichnet; die Emotionen
und Kognitionen sind von Depressivität, Angst und Panik geprägt), der zwischenmenschlichen Bezie-
hungen (die Betroffenen fühlen sich z.B. isoliert, einsam, unverstanden, verlassen) und der Wertewelt
(z.B. Reduktion der Beziehung zu werten, ihrer Realisierung, was sich in Interessenlosigkeit, Gleich-
gültigkeit und in Außenseiterpositionen ausdrücken kann).
Nach dem Interpersonell psychologischen Modell von Joiner (2005) nehmen sich Menschen aus
zwei Gründen das Leben: erstens, wenn gelernt wurde, über Grenzen zu gehen (z.B. durch Gewalter-
lebnisse, frühere Suizidversuche) und somit die Fähigkeit zum Suizid erworben wurde. Zweitens, wenn
der Wunsch zu sterben vorliegt, ausgelöst durch das subjektive Gefühl, eine Belastung für andere
und nicht mehr mit anderen verbunden zu sein. Problematisch ist, dass die Vorhersagekraft dieser
Modelle (welche PatientInnen suizidieren sich tatsächlich?), trotz gutem empirischem Fundament,
begrenzt ist. Ein Grund dafür könnte das komplexe Zusammenspiel von Risiko- und Schutzfaktoren
über die Zeit sein, weshalb neuere theoretische Ansätze auf komplexe nichtlineare dynamische Sys-
teme zur Erklärung suizidaler Prozesse zurückgreifen (Fartacek, Schiepek, Kunrath, Fartacek, &
Plöderl, 2016).

73
Risikofaktoren und Warnzeichen
Um das Suizidrisiko besser quantifizieren zu können, sollte das Vorhandensein und die Ausprägung
von Risikofaktoren erhoben werden. Zu den wichtigsten Risikofaktoren zählen: psychische Störungen
(v.a. affektive Störungen, Psychosen, Alkoholabhängigkeit, Borderline Persönlichkeitsstörung), Hoff-
nungslosigkeit, Aggressivität, Suizidgedanken, Suizidmethoden (Planung, Letalität und Verfügbar-
keit), Suizide in der Familie und vor allem Suizidversuche in der Vergangenheit. Solche Risikofaktoren
sind statische bzw. Langzeitrisikofaktoren. Für die klinische Routine sind zudem dynamische Risiko-
faktoren bzw. Warnzeichen wichtig, die kurzfristig auf die Emergenz akut suizidaler Krisen hinweisen.
Hierzu zählen mitunter Suizidgedanken mit zunehmender Penetranz, Suizidpläne und der Verlust der
Distanzierungsfähigkeit, die allesamt direkt auf eine akut suizidale Krise hinweisen. Zudem gibt es
eine Reihe indirekter Warnzeichen wie Schlafstörungen, Agitation, starkes Gedankenreisen, uner-
träglicher psychischer Schmerz, Ausweglosigkeit, Hoffnungslosigkeit usw. (Rudd et al., 2006), die
auf eine suizidale Krise hindeuten könnten und eine weitere professionelle Abklärung nötig machen.
Für die Suizidrisikoeinschätzung kann auch die Kenntnis von Hochrisikophasen wichtig sein, wie etwa
die Entlassung aus der stationären psychiatrischen Versorgung. Dabei ist die Suizidrate am ersten
Tag am höchsten, bevor sie kontinuierlich zu sinken beginnt (Qin & Nordentoft, 2005). Anders aus-
gedrückt passierten 16-25% aller Suizide (in Dänemark) im ersten halben Jahr nach der Entlassung,
2-4% innerhalb der ersten Woche. Zudem besteht ein erhöhtes Suizidrisiko auch in der ersten Woche
des psychiatrischen Aufenthaltes.
In der Behandlung von Suizidgefährdeten müssen Behandelnde mit zum Teil heftigen Gegenüber-
tragungsreaktionen wie Ohnmacht, Wut/Hass, Angst, Ärger, aber auch mit subtileren Formen wie
Müdigkeit bzw. Schläfrigkeit rechnen. Neuerdings wird auch diskutiert, inwieweit sich solche Gegen-
übertragungen nicht auch als Warnzeichen für akute suizidale Krisen eignen (Yaseen et al., 2013).
Unabhängig davon ist es nötig, dass Helfende diese Gefühle bei sich wahrnehmen und nicht impulsiv
darauf reagieren (Maltsberger, 2001). Zur Förderung der Selbstwahrnehmung ist Supervision und
Intervision hilfreich.

Gesprächsführung und therapeutische Beziehung


Ein Hindernis für die Suizidrisikoeinschätzung ist, dass viele Menschen Hemmungen haben, die Sui-
zidalität offen anzufragen. Der Grund dafür ist vermutlich der Mythos „schlafende Hunde soll man
nicht wecken“. In Wirklichkeit führt das offene Gespräch über Suizidalität meist zur Entlastung.
Für Helfende ist es wichtig, sich immer wieder in Erinnerung zu rufen, Suizidalität so genau wie
möglich zu eruieren. Ein Vertrauensverhältnis zwischen KlinikerInnen und PatientInnen und das offene
Thematisieren der Suizidalität sind deshalb die wichtigsten Faktoren für eine realistische Einschätzung
des Suizidrisikos und zudem eine erste, wichtige weil beziehungsfördernde und entlastende Interven-
tion. Vor diesem Hintergrund kann auch der Aufbau einer therapeutischen Beziehung als basale
Voraussetzung für die Suizidrisikoeinschätzung erachtet werden. Dies erscheint so selbstverständlich,
aber wie sieht es in der Wirklichkeit aus? Sind die Bedingungen gegeben, um eine therapeutische
Beziehung zu ermöglichen? Ist genügend Zeit vorhanden? Ist man ungestört, oder läutet das Telefon,
oder kommt ständig jemand bei der Tür herein? Wie lange lasse ich PatientInnen reden, ohne sie zu
unterbrechen? Wie sehr verstehe ich PatientInnen aus ihrer Perspektive heraus? Treffe ich mit meinen
Äußerungen das, was in meinem Gegenüber vorgeht? Halte ich das Leiden aus oder reagiere ich
sofort mich mit Deutungen, Erklärungen und Ratschlägen? Traue ich mich, den PatientInnen auf
Augenhöhe gegenüber zu treten? Wie authentisch bin ich? Können PatientInnen offen über ihre
Gründe zum Leben und Sterben reden? Dies alles sind Faktoren, die essentiell für das Gelingen einer
therapeutischen Beziehung mit Suizidgefährdeten sind (Michel & Jobes, 2011).

74
Zwei Zugänge, die für eine therapeutische Beziehung mit suizidalen Menschen förderlich sind, wur-
den in den letzten Jahren in der Suizidforschung ausführlich diskutiert. Jobes (2006) propagiert ein
partnerschaftliches Modell der Suizidprävention. Dabei begegnen sich Behandelnde und PatientIn-
nen in Augenhöhe, die „klassisch“ medizinische Rollenverteilung zwischen allwissenden ÄrztInnen
und bedürftigen PatientInnen ist überwunden, beide tragen gleichwertig mit ihren zur Verfügung ste-
henden Wissen und Fähigkeiten zum Behandlungserfolg bei. Nach Michel and Valach (2004) gelingt
die therapeutische Beziehung dann am besten, wenn PatientInnen ungestört und frei die Entwicklung
im Leben bis zum Suizidversuch schildern können, um den Lebensentwurf und Brüche darin kennen
zu lernen, die mit Sinnverlust und Leiden einhergegangen sind und welche die eigentlichen Ursachen
der Suizidalität sind (narrativer Zugang).
Anzumerken bleibt, dass Studien auf Diskrepanzen zwischen der Risikoeinschätzung von KlinikerIn-
nen und von PatientInnen selbst verweisen. Eher selten ist, dass KlinikerInnen kein Risiko sehen,
betroffene PatientInnen jedoch schon. Häufiger sehen KlinikerInnen ein Risiko, aber die PatientInnen
nicht (Bewick, McBride, & Barkham, 2006). Zur Validierung der Risikoeinschätzung können valide,
reliable und praktikable Fragebögen hilfreich sein. Darüber hinaus ist die Anwendung von Fragebö-
gen empfehlenswert, weil es Hinweise gibt, dass einige suizidale PatientInnen ihr Befinden per Fra-
gebögen ehrlicher schildern als im persönlichen Gespräch, welches aber durch Fragebögen keines-
falls ersetzt werden kann.

Überblick Behandlung und suizidpräventive Maßnahmen


Wirksamkeitsstudien zu suizidpräventiven Maßnahmen sind aufwendig, da Suizide statistisch selten
sind und große Stichproben benötigt werden. Daher gibt es wenige nachweislich wirksame Metho-
den, die Suizide reduzieren. Mittlerweile gibt es jedoch etliche Behandlungskonzepte, die nachweis-
lich Suizidversuche reduzieren. Folgende Maßnahmen, die auch für PsychiatriepatientInnen in Frage
kommen, zeigten sich in randomisierten Kontrollgruppen-Studien bisher als erfolgversprechend.
Reduktion der Suizidversuche: Problemlöseorientierte Psychotherapie, dialektische Verhaltensthera-
pie oder mentalisierungsbasierte teilstationäre Therapie bei Borderline PatientInnen, kognitive Ver-
haltenstherapie, Kurztherapie ASSIP, Notfallkarte (um mit der Einrichtung in Kontakt treten zu kön-
nen), nahtlose Nachbetreuung, Flupenthixol als Depot, Lithium bei schweren affektiven Störungen,
Clozapine bei Schizophrenie. Die Suizidpräventive Wirkungsweise von Antidepressiva ist nach wie
vor umstritten (Gysin-Maillart, Schwab, Soravia, Megert, & Michel, 2016; Mann et al., 2005; M. D.
Rudd et al., 2015).
Reduktion von Suiziden: Nationale Suizidraten sanken mit dem häufigeren Verschreiben von Antide-
pressiva, was jedoch nicht von generellen Trends in den Suizidraten unterschieden werden kann
(Mann et al., 2005) oder nicht auch auf andere Faktoren zurückführbar wäre (Kapusta, 2009).
Deutliche Ergebnisse gibt es für Lithiumtherapie (Cipriani, Hawton, Stockton, & Geddes, 2013) und
für Maßnahmen, die PatientInnen ein Gefühl der Anbindung („connectedness“) geben können: Re-
gelmäßiger Briefkontakt oder einfache Nachsorgetermine nach dem stationären Aufenthalt aufgrund
von Suizidalität oder Depressivität konnten die Suizidrate senken (Fleischmann et al., 2008; Motto
& Bostrom, 2001). Die möglichst nahtlose Anbindung an ambulante Angebote nach der Entlassung
aus dem Krankenhaus stellt sich überhaupt als essentiell heraus (Knesper, 2010).
Die klinische Versorgung akut suizidaler Krisen beruht lege artis auf den Grundzügen der Krisenin-
tervention (Granello, 2010), die als zeitlich limitierte, interdisziplinäre Intervention mit einem spezifi-
schen, auf die unmittelbare Stabilisierung des Patienten ausgerichteten therapeutischen Vorgehen
beschrieben werden kann (Berger & Riecher Rössler 2004). Der Fokus liegt vor allem auf der emo-
tionalen Entlastung, dem Durcharbeiten und gemeinsames Verstehen der Hintergründe der Krise,
der Stabilisierung des Patienten in seiner realen Situation sowie der Neuorientierung und dem Aufbau
neuer Perspektiven.
Anzumerken bleibt, dass übliche Maßnahmen (z.B. die stationäre Aufnahme) bei PatientInnen mit
chronischer Suizidalität, die oft mit einer Persönlichkeitsstörung einhergeht, kontraproduktiv sein kön-
nen (Paris, 2007).

75
Literatur (* empfehlenswerte Basisliteratur)
Bewick, B. M., McBride, J., & Barkham, M. (2006). When clients and practitioners have differing views of risk: Bench-
marks for improving assessment and practice. Counselling and Psychotherapy Research, 6(1), 50-59.
Cipriani, A., Hawton, K., Stockton, S., & Geddes, J. R. (2013). Lithium in the prevention of suicide in mood disorders:
updated systematic review and meta-analysis. BMJ, 346, f3646.
Fartacek, C., Schiepek, G. K., Kunrath, S., Fartacek, R., & Plöderl, M. (2016). Real-time monitoring of nonlinear suicidal
dynamics: methodology and a demonstrative case report. Frontiers in Psychology, 7.
Fleischmann, A., Bertolote, J. M., Wasserman, D., De Leo, D., Bolhari, J., Botega, N. J., Thanh, H. T. (2008). Effective-
ness of brief intervention and contact for suicide attempters: a randomized controlled trial in five countries. Bulletin of the
World Health Organization, 86(9), 703-709.
Granello, D. H. (2010). A suicide crisis intervention model with 25 practical strategies for implementation. Journal of
Mental Health Counseling, 32, 218–235.
Gysin-Maillart, A., Schwab, S., Soravia, L., Megert, M., & Michel, K. (2016). A novel brief therapy for patients who at-
tempt suicide: a 24-months follow-up randomized controlled study of the Attempted Suicide Short Intervention Program
(ASSIP). PLoS medicine, 13(3), e1001968.
Jobes, D. A. (2006). Managing suicidal risk: A collaborative approach. New York: Guilford Press.
*Joiner, T. E. (2005). Why people die by suicide. Cambridge: Harvard University Press.
Knesper, D. (2010). American Association of Suicidology, & Suicide Prevention Resource Center Continuity of care for
suicide prevention and research: Suicide attempts and suicide deaths subsequent to discharge from the emergency de-
partment or psychiatry inpatient unit. Newton, MA: Education Development Center, Inc.
Large, M., Ryan, C., & Nielssen, O. (2011). The validity and utility of risk assessment for inpatient suicide. Australasian
psychiatry :bulletin of Royal Australian and New Zealand College of Psychiatrists, 19(6), 507-512.
Maltsberger, J. T. (2001). Treating the suicidal patient. Basic principles. Ann N Y Acad Sci, 932, 158-165; discussion
165-158.
Mann, J. J., Apter, A., Bertolote, J., Beautrais, A., Currier, D., Haas, A., Hendin, H. (2005). Suicide prevention strate-
gies: a systematic review. JAMA, 294(16), 2064-2074.
Mann, J. J., Waternaux, C., Haas, G. L., & Malone, K. M. (1999). Toward a clinical model of suicidal behavior in psy-
chiatric patients. The American journal of psychiatry, 156(2), 181-189.
Michel, K., & Jobes, D. A. (2011). Building a therapeutic alliance with the suicidal patient. Washington, DC: American
Psychological Association.
Michel, K., & Valach, L. (2004). The working alliance with suicide attempters. European Psychiatry, 19, 130s-130s.
Motto, J. A., & Bostrom, A. G. (2001). A randomized controlled trial of postcrisis suicide prevention. Psychiatric services,
52(6), 828-833.
Paris, J. (2007). Half in love with death: Managing the chronically suicidal patient: Routledge.
Qin, P., & Nordentoft, M. (2005). Suicide risk in relation to psychiatric hospitalization: evidence based on longitudinal
registers. Archives of general psychiatry, 62(4), 427-432.
Ringel, E. (1953/2005). Der Selbstmord. Abschluss einer krankhaften psychischen Entwicklung. (9 ed.). Eschborn: Klotz.
Ringel, E. (1984). Selbstmord: Appell an die anderen. München: Kaiser.
Rudd, M. D., Berman, A. L., Joiner, T. E., Nock, M. K., Silverman, M. M., Mandrusiak, M., Witte, T. (2006). Warning
signs for suicide: Theory, research, and clinical applications. Suicide and Life-Threatening Behavior, 36(3), 255-262.
*Sonneck, G., Kapusta, N., & Tomandl, G. (Eds.). (2012). Krisenintervention und Suizidverhütung. Wien: UTB.
Rudd, M. D., Bryan, C. J., Wertenberger, E. G., Peterson, A. L., Young-McCaughan, S., Mintz, J., Bruce, T. O. (2015).
Brief cognitive-behavioral therapy effects on post-treatment suicide attempts in a military sample: results of a randomized
clinical trial with 2-year follow-up. The American journal of psychiatry, 172(5), 441-449.
WHO. (2014). Preventing suicide: A global imperative. Retrieved September, 2014, from http://www.who.int/men-
tal_health/suicide-prevention/world_report_2014/en/
*Wolfersdorf, M. (2014). Suizid und Suizidprävention Ein klinischer, psychosozialer Auftrag. Neurologie und Psychiatrie,
16 (1), 36-44.
Yaseen, Z. S., Briggs, J., Kopeykina, I., Orchard, K. M., Silberlicht, J., Bhingradia, H., & Galynker, I. I. (2013).
Distinctive emotional responses of clinicians to suicide-attempting patients-a comparative study. BMC psychiatry, 13(1),
1.

76
Universitätsklinik für Psychiatrie, Psychotherapie und Psychosomatik der PMU
Vorstand: Prim. Univ. Prof. Dr. Wolfgang Aichhorn, MBA

Institut für Klinische Psychologie


LEITUNG: MAG. DR. ANDREAS K. KAISER, MSc

Dear students,

in the USMLE, “Medical Psychology” is not a separate category. Relevant topics are located in the
First aid USMLE Step 1 Handbook 2015 in Section III “Psychiatry”, subsection “Psychology.” In our
lecture we are covering the following USMLE topics: learning mechanisms (classical conditioning,
operant conditioning), transference and countertransference, and ego defenses. For these topics,
questions for our exams are formulated in English and are based on the USMLE textbook.

As general preparation for the USMLE, we have compiled various examples similar to those in the
actual exam for your information. These examples have been taken from experience and have a
direct reference to the lecture on Medical Psychology in the available script. The examples Nr. 10
(USMLE Step 1 Q Book page 431, Nr. 36), Nr. 15 (page 409, Nr. 36), Nr. 16 (page 427, Nr. 15),
Nr. 17 (page 431, Nr. 39), Nr. 18 (page 432, Nr. 42) and Nr. 19 (page 433, Nr. 50) are taken
from Daugherty, S. R., Ph.D., (2006). USMLE Step 1 QBook (third edition). Kaplan Publishing, New
York, Chicago.

We wish you every success in solving the test examples and even more so on the USMLE exam. If
you should have any questions concerning test preparation, please contact the psychologist who is
specialized on the topic. Name, telephone number and Email addresses are on the last page of the
script.

Best regards,

Mag. Dr. Andreas K. Kaiser, MSc


Mag. Dr. Clemens Fartacek
Priv.-Doz. Mag. Dr. Martin Plöderl
Coordinators

77
Medical psychology examples as preparation for the USMLE exam

1. A 17-year-old girl and her mother come to the emergency department of the university clinic for
child and youth psychiatry. The girl complains that she has lost 15 kilos since her parents have
separated and lived apart. She weighs only 36 kilos at a height of 161cm. She has met with a
psychotherapist four times, however has continued to lose another 2.5 kilos. The mother wants the
daughter hospitalized. The girl realizes that her calorie counting has affected her concentration how-
ever she doesn’t want to miss too much school by being hospitalized for a longer time. Which ap-
proach would you recommend?

(A) Referral to a nutritional counsellor


(B) Scheduled admission as orientation phase
(C) Scheduled admission as project phase
(D) Continuation of outpatient psychotherapy
(E) In-patient supervision (Unterbringungsbereich)

2. A 17-year-old girl is admitted to the child and youth psychiatry department for self-destructive
behavior and depression. The girl immediately reports several years of sexual abuse (contact over
clothes) from her foster father. She is unsure if she should press charges against him. The mother
doubts the girl’s accusations and considers her mentally ill. What is the first step you should take?

(A) You file an „imminent danger“ report at the youth welfare office
(B) You give the case to the child protection group
(C) You press charges
(D) You establish contact with the child protection center to obtain case related
information for a lawsuit (Prozessbegleitung)

3. A 13-year-old girl comes to school on Monday with bruises on her arms and hickeys (Knutsch-
flecken) on her neck. She is unconcentrated and weepy. A girlfriend convinces her to tell what hap-
pened. She explains that she was at a party and heavily under the influence of alcohol. She allowed
petting from a 23-year-old that she liked. Although she had not agreed to sex, the 23-year-old held
her down and raped her. The girlfriend convinces the 13-year-old girl to go to the school doctor for
help. What measures is the school doctor obligated to take?

(A) To bring charges against the 23-year-old


(B) To interrogate the 13-year-old in detail about the sexual assault
(C) To report to the youth welfare office
(D) To refer the case to the child protection agency (with reference to litigation)

4. A badly injured 17-year-old student is admitted to the hospital after a car accident. At first he is
in a deep coma (GCS3-4). Tests show that he has a skull fracture left temporal, haemorrhaging in
the III and IV ventricle, in the right side of the Pons and a generalized cerebral edema in the cisterna
ambiens. Physical examination and additional testing indicated a severe brain injury with major in-
volvement of the brain stem.
Two months after the trauma the patient begins early rehabilitation and intensive medical care and
is in a severly decelerated condition. At this point the patient recognizes his friends and his surround-
ings, he can take a few steps on his own and can communicate through mimic and gestures.

78
After another 6 months the patient returns home with slight neurological and cognitive deficiencies
and plans to pursue an education as an electrician. In which order should the neuropsychologist
make the following treatments?

1. Orientation training (A) 1-2-3-4-5


2. Family counselling (B) 3-4-5-1-2
3. Neuropsychological diagnostics (C) 2-5-1-3-4
4. Cognitive training, occupational reintegration (D) 5-4-3-1-2
5. Sensory stimulation (E) 5-2-1-3-4

5. An 80-year-old woman is brought to the emergency department with the ambulance after a neigh-
bour found her lying on the floor. The third-party anamnesis with her daughter proves that she has
been dealing with increasing forgetfulness and mood swings. The mother has difficulties in unfamiliar
surroundings. The patient denies the problem and doesn’t think she needs help. Which of the fol-
lowing is the most likely diagnosis?

(A) Cerebrovascular accident (Insult)


(B) Dementia
(C) Delirium
(D) Depression

6. A 59-year old patient was referred to your clinic by his general practitioner because of suicidal
thoughts. The patient seemingly has a lack of drive, is depressed and motorically slowed down.
Currently, there are relationship problems and his wife wants to get a divorce. In addition, there are
financial problems. During the interview the patient also seems to be slightly drunk. His reply to your
question “How do you feel?” is: “I think I can’t stand this anymore”. Which of the following statements
is the most helpful for the patient:

(A) Why does your wife want a divorce?


(B) We do have a lot of patients with similar problems. You will see: we sure can help you!
(C) What is hardest for you to bare at the moment?
(D) You seem to be really depressed. Therefore, I suggest some antidepressive medication.
(E) I think you are drunk. I will talk to you when you are sober again.

7. A 34-year-old patient is hospitalized in the psychiatric department after a suicide attempt with
medication. She reported that on the one hand, she had really wished to die, but on the other hand,
the overdose had been a cry for help. Currently, the patient seems to be depressed but motorically
agitated. She is known to have a bipolar disorder. The patient is single and has no children. In a
medical report from a previous treatment in an inpatient psychotherapeutic clinic, the patient is sus-
pected to have a Borderline personality disorder in addition to Bipolar disorder. During the treatment,
the patient talks for the first time about being repeatedly sexually abused by her father. Based on
scientific studies, which risk factor is known to contribute most to an increased suicide risk of the
patient?

(A) Bipolar Disorder


(B) Borderline Personality Disorder
(C) Sexual abuse in childhood
(D) Current mixed state (depressed with motoric agitation)
(E) Suicide attempt

79
8. Mrs. Meier, a long-term patient, comes to the practice with her spouse. She is anxious and nervous
and reports that she has been worried about her daughter for some time. She tells that her daughter
is studying in a far away city and hardly has any friends. The last time the parents had talked to their
daughter on the phone, she had said that she had to postpone her planned trip home by a few weeks
because she had to prepare for a big exam. Mrs. Meier explains further that her daughter had always
had such high expectations on herself and is well-known to be highly motivated in her studies. She’s
afraid that „the child“ is very lonely and could—if she can’t meet the enormous demands—„do
something“. Ultimately, the plan was to buy a house in a few years where the whole family could
live. They are looking forward to eventually playing with their grandchildren in the garden. Mr. Meier
doesn’t seem to have any worries about his daughter. He doesn’t appear to be anxious or in any
way emotional. He has nothing to say about future plans or any topic mentioned. With the current
evaluation a credible suicide prediction cannot be made. Which of the three people is most likely to
commit suicide?

(A) the daughter


(B) Mrs. Meier
(C) Mr. Meier

9. A 35-year-old patient is referred to the medical department for chronic stomach pain without
any medical correlate. She has suffered from pain and bloating for 5 years and has been exam-
ined multiple times. The patient comes from Bosnia, has 3 kids and moved to Austria in 2001. She
lost her father and brother in the war. The patient is exhausted and suffers from the fear of having
a serious illness. She wants to have a laparoscopy again.

Which therapies would be most effective?

(A) Another laparoscopy


(B) Further medical examinations
(C) A psychotherapeutic pain therapy

What influences the patient’s feelings of pain?

(D) A hormonal mood swing


(E) The loss of family members causing a depressive reaction
(F) A physical illness

10. A 10-year-old girl who is a suspected victim of child abuse is referred to a psychologist for
evaluation. During the evaluation, the patient is asked to construct a story using pictures. Which of
the following psychometric measures was used?

(A) Minnesota Multiphasic Personality Inventory (MMPI)


(B) Myers-Briggs Personality Inventory
(C) Rorschach Test
(D) Thematic Apperception Test
(E) Type A und B Behaviour Patterns Test

80
11. A 43-year-old woman, mother of two children aged 9 and 12, has been divorced for almost
one year. Since the divorce she has had difficulty sleeping, a loss of appetite and feelings of guilt.
In the shower she notices a lump in her breast. Concerned, she goes to her doctor who makes the
appropriate examinations. After receiving the results the doctor gently breaks the news to the
woman that she has breast cancer. In the course of the discussion the patient experiences a numb-
ing break in communication, tachycardia, heavy sweating, nausea and motor agitation. She wants
to flee the room. Which of the following is the most likely diagnosis?

(A) Acute stress disorder


(B) Post-traumatic stress disorder
(C) Adjustment disorder
(D) Reactive depression
(E) Panic attack

12. A 38-year-old breast cancer patient who was diagnosed one week before reports: “Since my
diagnosis with cancer, I’m scared all the time. I can’t eat or sleep and I cry all the time. It’s as if
someone has pulled the rug out from under my feet. I’m desperate and think about cancer day
and night. I’m just not myself.“ With which psychological disorder (DSM IV) would you diagnose
this patient?

(A) Panic Disorder (300.21, 300.01)


(B) Major Depressive Disorder/ Unipolar Depression (296.21)
(C) Adjustment Disorder with Depressed Mood (309.0)
(D) Somatization Disorder (300.81)

13. A 21-year old medical student goes to the student counselling center with symptoms of extreme
exhaustion. Lately she has had poor test results and has been absent from compulsory classes. She
neglects her school papers for weeks or completes work far below her usual standards. She is abrupt
with her fellow students. She’s afraid she will fail medical school and drop out. The relationship with
her boyfriend is also at risk as he thinks she is letting herself go. She isolates herself more and more
and drinks too much alcohol – often in the morning and before driving her car. Furthermore, she
has recently lost 10 kg (BMI 19,7). She sees no way out and sometimes thinks about taking her own
life. She has already thought out how she could obtain the right combination of drugs to proceed.
Sometimes in the evening she sits silently dazed, as if paralyzed, in front of her books. Her boyfriend
who is not allowed to speak with her in this state finds it all very eerie.

Which of the following is the most likely diagnosis?


(A) State of exhaustion (Burnout)
(B) Eating Disorder
(C) Depressive episode
(D) Risky use of alcohol
(E) Adjustment Disorder

Which therapeutic measures should be taken?

81
(F) Institutionalize in a psychiatric institution
(G) Psychiatriatic evaluation
(H) Learn autogenic training and go to a fitness studio
(I) Ambulatory long-term therapy with a problem-solving-oriented character

14. A 35-year-old general practitioner complains of fatigue. He thinks about his job day and night
and works sometimes up to 16 hours a day. On the weekend he makes numerous house calls in a
large rural district. His wife and five children rarely see him. His wife told him that she is considering
divorce if he doesn’t radically reduce his work load. She sees no future in the marriage as it now
stands. He is desperate, suffers from various psychosomatic symptoms, he has difficulty sleeping,
worries about his income and feels misunderstood and incapable of taking action.

Which of the following is the most likely diagnosis?


(A) Somatization Disorder
(B) Adjustment Disorder
(C) Obsessive-compulsive Disorder
(D) State of Exhaustion (Burnout)
(E) Partnership problems

Which therapeutic methods are recommended?


(F) Marriage counselling
(G) Recovery in a clinical center
(H) Medical treatment through a specialist
(I) Counselling session with psychologist/psychotherapist

15. Eight research scientists are brought into the hospital by paramedics. The scientists are experi-
encing diaphoresis, blurred vision, palpitations and hallucinations with brilliant colours. Police sus-
pect that the coffee at their lab meeting was laced with a psychoactive substance. Which of the
following substances is most likely to be found in the coffee pot?

(A) Lysergic acid diethylamide (LSD)


(B) Methadone
(C) Phencyclidine (PCP)
(D) Phenobarbital
(E) Tetrahydrocannabinol (THC)

16. A 19-year-old man describes episodes in which he feels as if he’s “floating out of his body“
and observing himself from a place about 2 feet over his head. He is aware of everything going on
around him and can respond to others. Which of the following terms most accurately describes
what this patient is experiencing?
(A) Delirium tremens
(B) Delusion
(C) Dementia
(D) Depersonalization
(E) De-realization
(F) Hallucination
(G) Illusion

82
17. A 47-year-old man is hospitalized with acute pancreatitis. His history reveals a history of epi-
sodic binge alcohol consumption. When the physician meets with the patient to discuss the condi-
tion, the patient asks: „Doctor, what caused this?“ Which of the following is the most appropriate
response to this question?

(A) „I’m not sure.“


(B) „Probably a consequence of alcohol consumption.“
(C) „Probably your alcoholism.“
(D) „We need to talk about where we go from here in your long-term care.“
(E) „What do you think?“

18. A man brings his 45-year-old wife to the emergency department. He states she has been ill for
3 days and has had a temperature of 99.8 to 100.5 F. Today she is having difficulty staying
awake, is talking to persons who are not there, and at times appears to be frightened of some-
thing. She is restless and somewhat combative when restrained. What is the most likely diagnosis?

(A) Acute stress disorder


(B) Bipolar I disorder, manic type
(C) Brief psychotic disorder
(D) Delirium
(E) Dementia

19. A 35-year-old woman lives alone. She has never been married, has a Masters of Business Ad-
ministration and has been employed as a stock broker. She just got fired from her work because of
irresponsible work decisions on three separate occasions over the past 2 years. On these occa-
sions she had sold all of the securities of clients, without their authorization, and invested the
money in securities that had glossy portfolios but turned out to be worthless. On these three occa-
sions she worked 22 to 24 hours per day for 10 days at a time, gorged herself on „junk food“ and
drank alcohol excessively. Which of the following is the most likely diagnosis?

(A) Bipolar disorder, type I


(B) Bipolar disorder, type II
(C) Cyclothymic disorder
(D) Schizophrenic disorder, paranoid type
(E) Substance-induced delirium

The examples
Nr. 10 (USMLE Step 1 Q Book page 431, Nr. 36), Nr. 15 (page 409, Nr. 36),
Nr. 16 (page 427, Nr. 15), Nr. 17 (page 431, Nr. 39), Nr. 18 (page 432, Nr. 42) and
Nr. 19 (page 433, Nr. 50) are taken from Daugherty, S. R., Ph.D., (2006). USMLE Step 1 QBook
(third edition). New York, Chicago: Kaplan Publishing.

83
ANSWER KEY
1. B
2. B
3. A
4. C
5. B
6. C
7. E
8. C
9. B, C + E
10. D
11. A
12. C
13. C, D, E, G + I
14. D+I
15. A
16. D
17. B
18. D
19. A

84
Answers and Explanations:

1. The correct answer is B.


Due to the rapid weight loss and the physical risks associated with it (decompensation of
electrolyte balance, etc.), in-patient care is recommended. The girl can identify the negative
effects of the eating disorder, however has not developed any concrete changes in her eating
habits. As hospitalization is primarily the mother’s idea, a scheduled admission in terms of
problem orientation and as a resource for problem solving is possible. In-patient supervision
(Unterbringungsbereich) would only be necessary in a life-threatening situation.

2. The correct answer is B.


You bring the case to the child protection group of the hospital to discuss further procedure.
Additional information: Ultimately, the accused is a close relative of the girl. Therefore, press-
ing charges should be prevented until the well-being of the child is served through victim
protection services or a child protection group.

3. The correct answer is A.


According to Medical Law (Ärztegesetz), you have to bring charges against the 23-year-old.
Additional information: However from a psychological point of view and for the child’s well-
being, a referral to the child protection agency, in terms of facilitation, would be more fa-
vourable. The girl could decide herself if she brings charges against him, whereby the burden
of litigation would be easier to cope with and the quality of the testimony would be higher.

4. The correct answer is C.


(2) Family counselling: Due to the extreme stress of the situation, the family members are
provided psychological support right from the start. The sessions are often conducted by both
the doctor and the psychologist. Moreover, it is important in the first phase of the treatment
to obtain information about the patient. Attention should be paid to the patient’s previous
history, the patient’s life concept, preferences, experiences and interests as well as the social
framework to be able to establish links to the patient’s resources and capabilities.

(5) Sensory stimulation: This is the acute stage in which the neuropsychologist makes early
contact with the patient for improved arousal and for the development of a simple, reliable
communication code (e.g. yes-no-communication through winking). In early rehabilitation
and coma stimulation the family is consulted about creating sensory stimulations as well as
intimate surroundings (room arrangements) for the patient. The main goals of stimulation
therapy are to steadily extend the length of wakefulness and the focused attention perfor-
mance of the patient.

(1) Orientation training: The goals are to have structure in surroundings and daily aspects
and to restablish daily routines through implicit learning.

(3) Neuropsychological diagnostic: Neuropsychological diagnostic includes a comprehen-


sive clarification of cognitive capacity, affective disorders, personality changes as well as the
psychological processing of the consequences of illness or accident. The neuropsychological
results serve as a basis for the continuation of individual therapy, treatment and counselling
in regard to occupational reintegration of the patient.

85
(4) Cognitive training, occupational reintegration: Attention performance, concentration,
memory function as well as cognitive flexibility and cabability are trained. The highest goal
of cognitive training is the long-term recovery of independence in daily life and if possible,
the occupational reintegration of the patient.

5. The correct answer is B.


Dementia is an advanced, chronic, pathological change in the brain and leads to impairment
of the higher brain functions like: memory, thought, orientation, conception, judgement,
learning, language and problem solving. The indications for dementia in the existing case
are more than six months of memory defect that had been observed and confirmed by family
members. Furthermore, an orientation defect, emotional instability, inclination to fall, denial
and lack of judgement had also been described. The case against (A) cerebrovascular acci-
dent (Insult) is the lack of hemilateral symptoms. Generally, imaging procedures should be
applied for further diagnosis. A delirium (C) is unlikely because one of the main symptoms
of a delirium is a change in vigilance. Depression (D) is unlikely because of the slow devel-
opment, the orientations defect, the emotional instability and the attempt to conceal the
deficiency.

6. The correct answer is C.


(A) is not a good question, because you don’t know if this is the most important problem for
the patient at the very moment, or if he is not ready to talk to you about this issue. Answer
(B) may be perceived as downplaying the problems. Answer (D) and (E) are certainly no
invitation to talk about the patient’s current problem, and therefore do not contribute to
building a therapeutic relationship. Answer (C) is most appropriate, because it does not imply
any presumptions about the inner experiences of the patient, so the patient can feel free to
talk about what is currently on his mind, the flow of thought and emotions are not interrupted.
This contributes to the therapeutic relationship. In addition, the question is an attempt to
focus on the most painful experience, thus the multiple problems are prioritized, which is
crucial for developing goals in crisis intervention.

7. The correct answer is E.


From all known suicide risk factors, a previous suicide attempt – even in the sense of a cry
for help - is the most important predictive risk factor for future suicides and suicide attempts.
All other answers are, however, also important risk factors, especially borderline personality
disorder and bipolar disorder.

8. The correct answer is C.


All the information concerning the daughter is speculative. In detail: far away (definitely not
indicative) – supposedly has hardly any friends („hardly“ is no indication of loneliness and
too unspecific) – in preparation for a big exam (not indicative as it contradicts suicide risk –
she is future oriented) – no time for a visit (plausible reason, no indication) – high expecta-
tions, highly motivated (absolutely no indication, we find no signs that the daughter herself
is afraid of not reaching her goals) – enormous demands (speculation from the mother, no
indication that the daughter has „enormous“ demands). Mrs. Meier is active on behalf of her
daughter’s well-being (an indication that suicide is not likely as she is not focused on herself
but on another person – in addition, she is focused on the future of another person) – is
nervous and anxious (in context is understandable, but in the case of suicide there would be
no emotion apparent, „emotional numbness“) – house, grandchildren, garden,… (describes
clearly the presence of a high, not low, level of future plans). Although there are very few
concrete indications concerning Mr. Meier, the only right answer is (C).

86
The question is „who is most likely“ and only with Mr. Meier relevant factors are unknown.
We do not know if he is thinking only of himself, or if his thoughts are with other relevant
people (doctor, wife, daughter), if his non-emotional behaviour is totally normal or possibly
an indication of emotional numbness, and if the future (quite a big plan) has meaning for
him or not.

9. The correct answer is B, C und E.


The answer (A) is wrong as further invasive methods would increase somatisation. The answer
(E) is correct because the patient’s symptoms are a reaction deriving from the loss of her
relatives.

10. The correct answer is D.


The Thematic Apperception Test is a projective test that uses pictures depicting ambiguous
interpersonal situations that the examinee is asked to interpret. Psychodynamic theory sug-
gests that since the stimuli are vague, the patient projects his or her own thoughts, feelings
and conflicts into his or her responses, providing the examiner insight into the patient’s
thought and memory content. The Minnesota Multiphasic Personality Inventory (MMPI;
choice A), which uses true and false items, is the most popular objective personality test. The
Myers-Briggs Personality Inventory (choice B) is based on Jungian theory and assesses basic
dimensions of personality (extraversion); it is used extensively in occupational counselling.
The patient selects preferred adjectives from groups of choices. The Rorschach Test (choice
C) is another projective test that involves asking patients to describe what they see when
presented with a series of black and white inkblots. The Type A und B Behavior Patterns Test
(choice E) assesses the amount of „driven quality“ a person has to their life. Type A’s are
always „running out of time.“ This is a verbal test that resembles an interview.

11. The correct answer is A.


Acute stress disorder is a temporary disorder with a substantial degree of severity, which
through a psychological manifest could develop as a reaction to serious mental and physical
burdens also in mentally sound people. The symptoms occur within minutes after the dis-
tressful incident and last normally hours or maximum a few days. For the diagnosis of a post
traumatic stress disorder (B) it is too early. This occurs with a latency of weeks to months after
the distressful incident. Often catastrophes, serious accidents, witnessing a violent death or
being the victim of torture or rape can trigger the reaction. One cannot rule out that the
symptomatology stated above could lead to an (C) Adjustment disorder but would first be
diagnosed at a later time (usually within a month from the triggering effect) particularly if
depressive tendency, fear and above all, anxiety combined with the feeling that the patient
cannot manage are present. Few symptoms regarding a reactive depression (D) are present,
at least not in the initial diagnosis. Panic attacks (E) are diagnosed in situations in which no
objective danger exists.

12. The correct answer is C.


Adjustment disorder with depressed mood. The disorder in this category include those where
the primary symptom is a disturbance in mood. In other words, inappropriate, exaggerated,
or limited range of feelings. To be diagnosed with a mood disorder, you must have extreme
feelings. In other words, crying, and/or feeling depressed, suicidal frequently. Or, the oppo-
site extreme, having excessive energy where sleep is not needed for days at a time and during
this time the decision making process in significantly hindered. The disorder is time-limited,
usually beginning within 3 months of the stressful event, and symptoms lessen within 6 months

87
upon removal of the stress or when new adaptation occurs. Panic Disorder (answer A) is
characterized by sudden attacks of intense fear or anxiety, usually associated with numerous
physical symptoms such as heart palpitations, rapid breathing or shortness of breath, blurred
vision, dizziness and racing thoughts.

Often these symptoms are thought to be a heart attack by the individual, and many cases
are diagnosed in hospital emergency rooms. The symptoms of this disorder come on rapidly
and without an identifiable stressor. Major Depressive Disorder (answer B) – Over a two week
period, the patient has consistently experienced five or more of the following symptoms, and
these behaviors must be outside the parameters of the patient's normal behavior. The symp-
toms of depression (2-weeks-period) include the following: depressed mood (such as feeling
of sadness or emptiness), reduced interest in activities that used to be enjoyed, sleep disturb-
ance (either not being able to sleep or sleeping to much), loss of energy or a significant
reduction in energy level, difficulty concentrating, holding a conversation, paying attention,
or making decisions that used to be made fairly easily, suicidal thoughts or intentions. So-
matization Disorder (answer D) includes a history of physical complaints prior to age 30
which occur over a period of several years. There must be a significant impairment in func-
tioning or history of resulting medical treatment. After appropriate assessment for the re-
ported symptoms or for at least the severity of the complaints.

13. The correct answer is C, D, E, G and I.


After some months of psychotherapy, the student managed to reduce her alcohol consump-
tion and caught up on her neglected obligations. In a meeting with the Dean of Student
Affairs she was granted a delay for re-examinations. She experiences more pleasure and
success in her studies and in her relationship and is less exhausted than before. She no longer
withdraws from her fellow students, the suicide fantasies are less frequent and she no longer
fears dropping out. The approach (G) in this case proved to be successful as under the
circumstances rapid action was necessary to avoid tragedy. After examination of the symp-
toms and willingness for psychotherapy was present, approach (F) would have been too
massive. With such symptomatology, clarification by a specialist is recommended – especially
in respect to a depression. A medical therapy for example should be implemented until it is
proven that depression no longer exists. Autogenic training and physical exercise (H) are very
probable remedies for relieving stress and exhaustion however in this case as first and only
treatment it would be unsuitable.

14. The correct answer is D and I.


In a 2 hour consultation it is clear that the patient has made his father his rolemodel. His
father was also „always there for his patients“. The father recently died after a second heart
attack. The patient acknowledges the connection between his inclination to constantly being
on call for his patients and his half conscious desire to do justice to his father. He accepts
the recommendation for burnout prevention treatment. He has agreed to report again when
the need arises. After three months he has another consultation. He reports that he has lim-
ited his workload and reduced the amount of house calls. Financial ruin has been avoided,
he has time again for his family and his wife has hope again for the marriage. With the
appropriate insight into the cause of the symptomatology, it is recommended to begin with
gentle measures. As first measures F, G and H would have been too substantial in this case
as the patient was able to put the recommendations from the consultation to use with re-
markable initiative.

88
15. The correct answer is A.
These eight researchers are probably under the influence of LSD, which causes hallucinations
notable for their brilliant colours. LSD also shows activity at serotonin receptors and can
activate the sympathetic nervous system, resulting in symptoms such as diaphoresis, blurred
vision (due to pupil dilation) and palpitations.

Methadone (choice B) is a synthetic opiate used to treat heroin addiction. It has analgesic
properties but does not ordinarily induce hallucinations.
PCP (choice C) can cause hallucinations marked by alterations of body image and distortions
of space and time. PCP can also cause a dissociative anaesthesia and analgesia. Common
side effects of PCP use include hypersalivation, muscular rigidity, hypertension and nystag-
mus. Highly coloured visual hallucinations are not as commonly seen with PCP intoxication
as with LSD intoxication.
Phenobarbital (choice D) is a long-acting barbiturate that acts as a CNS depressant. It is
used in the long-term management of seizure disorders.
THC (choice E) is found in marijuana and produces a euphoric high followed by subsequent
relaxation and sleepiness. Marijuana use can result in visual hallucinations, delusions and a
toxic psychosis, but generally only at extremely high doses.

16. The correct answer is D.


Depersonalization occurs when an individual perceives that his „person“ or „self“ becomes
detached from his body. He maintains his contact with reality and level of consciousness, as
opposed to derealization (choice E), in which the environment seems distorted or takes on
an unreal quality.
Delirium tremens (choice A) occurs as a withdrawal symptom in chronic alcoholics abstaining
from alcohol. It is characterized by anxiety, tremor, sweating and vivid hallucinations.
Delusions (choice B) are false, fixed belief systems that exist in the face of no evidence to
support them and are not shared by most others.
A patient with dementia (choice C) has a normal level of consciousness with gradual onset
of cognitive symptoms (e.g., memory disturbance) and small potential for recovery. In con-
trast, delirium is characterized by impaired consciousness (diminished alertness and disturbed
orientation to time, place and person), acute onset of symptoms (e.g., confusion), excessive
daytime sleeping, diffuse slowing of brain waves on EEG and full recovery in most cases.
Hallucinations (choice F) are perceptions that have no basis in the environmental stimuli
present. Hallucinating patients are often observed to respond to internal stimuli (e.g. patients
who appear to be talking to themselves are actually replying to voices heard in their heads).
An illusion (choice G) is when a real environmental stimulus is misperceived as something it
is not.

17. The correct answer is B.


It is important that the patient knows the association between his self-destructive behavior
and the physical sequelae. However, it is also important for cooperation in long-term man-
agement to not make the patient defensive by labelling him an alcoholic (choice C).
Equivocating with „I’m not sure“ (choice A) or avoiding the patient’s question (choice D and
E) does not provide a basis for cooperative and mutual trust in the doctor-patient relation-
ship.

89
18. The correct answer is D.
This is a psychotic level disorder (the patient is hallucinating): she has a fluctuating level of
consciousness and is disoriented. Also, there is a clear history of a febrile condition that
developed rather rapidly, all of which suggests delirium.
In acute stress disorder (choice A) a traumatic event occurs that precipitates an anxiety-type
reaction, not a change in the sensorium.
In both bipolar I disorder, manic type (choice B) and brief psychotic disorder (choice C),
patients may reach a level of behavioural disruption of psychotic proportion. They do not,
however, demonstrate changes in level of consciousness or major disorientation.

Persons with dementia (choice E) demonstrate a clear sensorium with no fluctuations in the
level of consciousness. Additionally, persons with dementia predominantly show symptoms
of impairment of cognitive functions (e.g. memory impairment).

19. The correct answer is A.


Bipolar disorder, type I, is the appropriate diagnosis because she has had repeated manic
episodes. The inappropriate grandiose activity with her clients´ accounts (without the benefit
of consultation), decreased need for sleep and involvement in potentially self-destructive be-
havior (e.g. excessive alcohol consumption) support this diagnosis.
There is no history of depressive episodes, which is mandatory for the diagnosis of bipolar
disorder, type II (choice B).
Since her behavior is of psychotic proportion and there is no history of depressive episodes,
cyclothymic disorder (choice C) is incorrect.
Persons with schizophrenic disorder, paranoid type (choice D), have a major thought and
affect disorder and characteristically do hallucinate. During an episode, they are unable to
function in reality, e.g. selling and buying securities on the stock market. She demonstrates
no such behavior.
The hallmark of delirium (choice E) is a fluctuating level of consciousness. There are no
indications in the history that she is manifesting this symptom.

The examples
Nr. 10 (USMLE Step 1 Q Book page 431, Nr. 36),
Nr. 15 (page 409, Nr. 36), Nr. 16 (page 427, Nr. 15),
Nr. 17 (page 431, Nr. 39), Nr. 18 (page 432, Nr. 42) and
Nr. 19 (page 433, Nr. 50) are taken from Daugherty, S. R., Ph.D., (2006). USMLE Step 1 QBook
(third edition). New York, Chicago: Kaplan Publishing.

90

Vous aimerez peut-être aussi